Jump to content

Wikipedia:Reference desk/Science: Difference between revisions

From Wikipedia, the free encyclopedia
Content deleted Content added
Line 894: Line 894:
:::Could they ? Perhaps, although probably not with current technology and knowledge (and, in any case, the public water supply would be a much more effective means of distribution). Would they ? No - not because of any ethical scruples, but because it would be economic and political suicide. Monsanto, like any commercial company, have economic incentives to exploit their market as much as they can get away with. A country that introduced non-voluntary reversible sterilisation of all of its citizens would see its economy collapse immediately. [[User:Gandalf61|Gandalf61]] ([[User talk:Gandalf61|talk]]) 11:43, 26 October 2009 (UTC)
:::Could they ? Perhaps, although probably not with current technology and knowledge (and, in any case, the public water supply would be a much more effective means of distribution). Would they ? No - not because of any ethical scruples, but because it would be economic and political suicide. Monsanto, like any commercial company, have economic incentives to exploit their market as much as they can get away with. A country that introduced non-voluntary reversible sterilisation of all of its citizens would see its economy collapse immediately. [[User:Gandalf61|Gandalf61]] ([[User talk:Gandalf61|talk]]) 11:43, 26 October 2009 (UTC)
::::Ever hear about the fog that jumped out of the boiling pot of water while his companion fell asleep in the ever increasing temp of the water in the pot he was in? [[Special:Contributions/71.100.9.185|71.100.9.185]] ([[User talk:71.100.9.185|talk]]) 12:22, 26 October 2009 (UTC)
::::Ever hear about the fog that jumped out of the boiling pot of water while his companion fell asleep in the ever increasing temp of the water in the pot he was in? [[Special:Contributions/71.100.9.185|71.100.9.185]] ([[User talk:71.100.9.185|talk]]) 12:22, 26 October 2009 (UTC)

:::::Yes, we [[Boiling frog|have an article on it]], actually. But hey, if you're convinced it's likely, don't let us dissuade you. But you're probably not going to convince us here. It sounds very science fiction and far-fetched for a number of reasons. It presupposes much closer cooperation between the government and large biotech companies than exists, where both would mutually be willing to let the other one take on activities of such risk that they could kill millions and lead to political revolution. This is rather improbable. --[[Special:Contributions/98.217.71.237|98.217.71.237]] ([[User talk:98.217.71.237|talk]]) 12:33, 26 October 2009 (UTC)


== ascetic acid ==
== ascetic acid ==

Revision as of 12:33, 26 October 2009

Welcome to the science section
of the Wikipedia reference desk.
Select a section:
Want a faster answer?

Main page: Help searching Wikipedia

   

How can I get my question answered?

  • Select the section of the desk that best fits the general topic of your question (see the navigation column to the right).
  • Post your question to only one section, providing a short header that gives the topic of your question.
  • Type '~~~~' (that is, four tilde characters) at the end – this signs and dates your contribution so we know who wrote what and when.
  • Don't post personal contact information – it will be removed. Any answers will be provided here.
  • Please be as specific as possible, and include all relevant context – the usefulness of answers may depend on the context.
  • Note:
    • We don't answer (and may remove) questions that require medical diagnosis or legal advice.
    • We don't answer requests for opinions, predictions or debate.
    • We don't do your homework for you, though we'll help you past the stuck point.
    • We don't conduct original research or provide a free source of ideas, but we'll help you find information you need.



How do I answer a question?

Main page: Wikipedia:Reference desk/Guidelines

  • The best answers address the question directly, and back up facts with wikilinks and links to sources. Do not edit others' comments and do not give any medical or legal advice.
See also:



October 20

Vacuum Ablation

I've heard of a concept called Vacuum Ablation in which given enough time the atoms in a substance boil away into space. Is there such a thing? Josh Parris 02:54, 20 October 2009 (UTC)[reply]

Googling "vacuum ablation", it looks like the term refers to a situation where there is a substance in a vacuum, and the substance is ablated by pulsing a laser on it. It's not a matter of just waiting for the ablation to occur, without supplying any source of energy. However, see also Outgassing#Outgassing in a vacuum, which is closer to the situation you're thinking of. Red Act (talk) 03:17, 20 October 2009 (UTC)[reply]
You might also want to read Pulsed laser deposition, which is what vacuum ablation is used in. Red Act (talk) 03:21, 20 October 2009 (UTC)[reply]

So, in summary: No, Vacuum Ablation is not something extremely long-lived space structures need to worry about due to the high energies required to cause it. Josh Parris 03:34, 20 October 2009 (UTC)[reply]

Resolved

largest possible Black Hole

What is the largest possible Black Hole and how long would it take to form? 71.100.9.185 (talk) 03:23, 20 October 2009 (UTC)[reply]

I don't think there's any theoretical limit on the upper size of a black hole. Josh Parris 03:34, 20 October 2009 (UTC)[reply]
Black holes have a minimum size, in that if they're too small, Hawking radiation will make them evaporate. But the only limit on maximum size is how much matter in the universe gets close enough to get sucked in. The supermassive black hole in OJ 287 is currently the largest black hole known, with a mass of about 18 billion times the mass of the sun. But AFAIK there's no theoretical reason why a black hole couldn't be much larger than even that. Red Act (talk) 04:35, 20 October 2009 (UTC)[reply]
Size in terms of mass is one thing. Another is size in terms of diameter. But given the nature of black holes, is "diameter" a meaningful term? If so, do scientists have any clue about the "widest" black hole? ←Baseball Bugs What's up, Doc? carrots04:42, 20 October 2009 (UTC)[reply]
For a black hole (or more pedantically precisely, a nonrotating, uncharged black hole), the mass and "diameter" are equivalent. The Schwarzschild radius of a black hole, which is the radius in Schwarzschild coordinates of the black hole's event horizon, is directly proportional to the black hole's mass: r=2Gm/c2. So the aforementioned black hole in OJ 287 has a Schwarzschild radius of about 5x1013 m. There is no more a theoretical maximum on the Schwarzschild radius of a black hole than there is a theoretical maximum on a black hole's mass. Red Act (talk) 05:09, 20 October 2009 (UTC)[reply]
I don't follow how they're "equivalent", but "proportional" makes sense. 5 times 10 to the 13th meters is apparently on the order of an Astronomical Unit, yes? (Distance from sun to earth) That would be sufficiently large. ←Baseball Bugs What's up, Doc? carrots05:15, 20 October 2009 (UTC)[reply]
They are equivalent because they are essentially the same thing just measured in different units. --Tango (talk) 10:39, 20 October 2009 (UTC)[reply]

What about the amount of time it would take to form the largest possible Black Hole and how long did it take to form the supermassive black hole? Also after the largest possible Black Hole sucks up all the matter in the Universe could it then suck up all of the empty space? 71.100.9.185 (talk) 04:51, 20 October 2009 (UTC)[reply]

That's kind-of one hypothesis about the end of the universe, but there's a lot of distance between black holes. ←Baseball Bugs What's up, Doc? carrots05:15, 20 October 2009 (UTC)[reply]
That is one theory. However the currently accepted theory is that the expansion of the universe is accelerating, despite gravity pulling everything together. Under that theory a black hole could conceivably suck up all the matter nearby given enough time, which could be quite a lot of stuff, like a galaxy cluster, but not everything in the universe. Rckrone (talk) 05:30, 20 October 2009 (UTC)[reply]
Not a single black hole, maybe, but all of them together could theoretically eventually consume everything that wasn't a black hole, just through random drifting? Or most of it. Then all you would have is drifting black holes, which might eventually drift into each other - or might not. ←Baseball Bugs What's up, Doc? carrots05:35, 20 October 2009 (UTC)[reply]
"Or not" is more likely, due to the expansion of the universe. And all the black holes will eventually evaporate due to Hawking radiation, starting as soon as the background temperature of the universe drops below their blackbody temperature. Just imagine - a vast, dark, empty, universe, but occasionally some points light up, first very slowly in the long radio frequencies, but culminating in an incredible flash. Possibly the last survivors travel in search of these explosions to harvest the entropy - there should be a (somewhat depressing) SF story in there... --Stephan Schulz (talk) 10:56, 20 October 2009 (UTC)[reply]
"An incredible flash..." the next Big Bang? I thought one theory was that space doesn't exactly exist by itself, that it's somehow connected with the objects in it. If there are no objects in it, maybe there's no space either? Not a vast nothingness - but a singularity - and the possibility for the cycle to start over? ←Baseball Bugs What's up, Doc? carrots13:59, 20 October 2009 (UTC)[reply]
Well, intriguing as it is, Hawking radiation will only release the energy equivalent of the matter that formed the hole in the first place. In the last second, it will release the energy of about 1 million Hydrogen bombs. Impressive, yes, but not Big Bang stuff. Moreover, this will be a conventional explosion into space, not a metric explosion of space. --Stephan Schulz (talk) 14:53, 21 October 2009 (UTC)[reply]
Also, the Hawking radiation would have fairly high entropy, the big bang had very low entropy. (You mentioned harvesting entropy above - I think you were a little confused, you want to harvest the absence of entropy.) --Tango (talk) 15:09, 21 October 2009 (UTC)[reply]
Yes, point taken about the entropy. But my way sounds better. ;-). --Stephan Schulz (talk) 17:49, 21 October 2009 (UTC)[reply]
There isn't a supermassive black hole; there are a bunch of them. It's believed that most galaxies have a supermassive black hole at the center of them.
Black holes have existed and have been growing since shortly (in cosmological terms) after the big bang. So they've been around, and growing, and occasionally merging, for about 13 billion years now.
A big crunch, in which the universe recollapses down into one black hole singularity, is one possible ultimate fate of the universe. But it's not really known whether that will ever happen. If that does happen, it will be billions of years from now. Red Act (talk) 05:29, 20 October 2009 (UTC)[reply]
And, possibly, create another big bang, yes? With a new universe and possibly new laws of physics? ←Baseball Bugs What's up, Doc? carrots05:36, 20 October 2009 (UTC)[reply]
Well, there is the Big Bounce possibility. It's highly speculative, as would be any theory that extrapolates past singularities in both directions. And thinking that there might be different laws of physics in each bounce is even much more wildly speculative.
Again, heat death is the currently preferred theory, although I wouldn't take heat death as being engraved in stone, since predictions of the fate of the universe depend heavily on the nature of dark energy, about which we know next to nothing. The field of physical cosmology has changed drastically in the last few decades, and I don't think there's any good reason to expect that it won't continue to change drastically in the next few decades. We just don't have all the answers yet, which makes it an exciting time to be watching the progress as it's being made. Red Act (talk) 06:31, 20 October 2009 (UTC)[reply]
P.S.: Although collapsing into one big black hole hasn't been completely ruled out, the preponderance of the evidence at this point is that that probably isn't what will happen. Instead, the ultimate fate of the universe appears more likely to be heat death. Red Act (talk) 05:37, 20 October 2009 (UTC)[reply]
Meaning essentially a vast nothingness? ←Baseball Bugs What's up, Doc? carrots05:43, 20 October 2009 (UTC)[reply]

an inherently flawed workup to a Grignard synthesis?

So ... my lab synthesis prescribed to me worked like this. First we form phenylmagnesium bromide in ether in situ, then react it with methyl benzoate (in more ether). We hydrolyse the alkoxide salt with dilute sulfuric acid, discard the aqueous layer then dry the organic layer with brine and magnesium sulfate, then evaporate, then recrystallise the crude product with hexanes.

How likely is it that an explanation for a yield of say, 34%, comes from the fact that the magnesium (soft) alkoxide salt ends up forming a carbocation that is then even more soluble in water? The carbocation escapes into another phase, driving carbocation formation to the right ...

Secondly, I notice there isn't a lot in this workup that separates methyl benzoate and bromobenzene from the product ... is it reasonable that they all get consumed to near completion (I had an MP depression of about 5 C); their BPs are really high but the product is a solid; maybe bromobenzene and methyl benzoate form an azeotrope that can be evaporated easily...? (But on a totally different scale than evaporating ether.) Would acid-catalysed hydrolysis of the starting reagent into methanol and benzoic acid help significantly? John Riemann Soong (talk) 04:22, 20 October 2009 (UTC)[reply]

Does elevated oxygen allow adjusted respiratory rates?

People who are suddenly at unusually high altitudes tend to feel short of breath. Is the opposite true if we hyper-oxygenate a room?

Let's say respiratory rate is R, and respiration volume is V. Is it as simple as saying that in a room with an oxygen level X times normal at sea level a human will be able to breath at a rate of R/X? Or would the blood oxygen levels fall too low between breaths? Assuming constant respiratory rate, could we instead inhale V/X - taking shorter breaths to satisfy the body's need?

I'm envisioning a dude sitting in a room with 3X average oxygen and breathing *really* infrequently!218.25.32.210 (talk) 05:00, 20 October 2009 (UTC)[reply]

Breathing serves two purposes; the intake of oxygen, and actually more importantly, the exhaling of CO2. The body produces CO2 at roughly the same rate, regardless of how much oxygen you are taking in, so there is a minimum required breathing rate necessary to expel CO2 from your system before it builds up to toxic levels. Even in a pure-oxygen atmosphere, you'd still need to breath at roughly the same rate you do in standard 20% oxygen atmosphere, in order to get the CO2 out of your body. --Jayron32 05:16, 20 October 2009 (UTC)[reply]
Interesting! In such a situation would you be introducing potentially harmful levels of oxygen into your bloodstream, or can the lungs autocompensate by reducing uptake per inhalation?218.25.32.210 (talk) 06:54, 20 October 2009 (UTC)[reply]
It's a bit more subtle than stated above. If you are untrained, your reflex to exhale will kick in long before CO2 becomes critical, and your breathing rate is unaffected by elevated oxygen levels. If you go SCUBA diving, you are breathing an atmosphere that contains increased levels of oxygen in absolute terms - every 10m under the surface doubles increases the atmospheric pressure by one atm and leads to a corresponding increase in oxygen content. Untrained divers will use up their air supply at correspondingly increased rate, but trained divers can slow down their breathing somewhat and hence stay underwater much longer. And indeed, if the oxygen partial pressure exceeds certain values, it does become toxic. IIRC, the upper limit for recreational diving is 1.4 atm. You cannot reach this with plain air under normal depth restrictions, but you can with Nitrox, easily - with Nitrox 36, you can only go 29m down. Military divers seem to dive reasonably safely with up to 2 atm of oxygen partial pressure, but that leaves little room for errors. --Stephan Schulz (talk) 07:41, 20 October 2009 (UTC)[reply]
While SCUBA diving, you are supposed to keep your breathing regular, because holding your breath can allow pressure-differences to build up in the lungs if you change depth. However, I have noticed that at depths of ~ 50 or 80 feet, I can easily go a full minute or two minutes on Nitrox without feeling the need for breathing. It's a very unusual feeling - there is no shortness of breath at all, just a feeling of not needing any more air. As Stephan Schulz pointed out, this can help preserve the air supply - using it up slower. Divers at deeper depths need to really be cognizant of these effects to make sure that they are actually getting a safe level of oxygen. To help mitigate this, at great depth, Nitrox is usually supplanted with Trimix (breathing gas) (usually helium, nitrogen, and oxygen). Attempting to breath trimix at atmospheric pressure can be dangerous - there isn't enough oxygen partial pressure - so technical divers usually take the tank down to depth with them and switch out at the bottom. Nimur (talk) 14:14, 20 October 2009 (UTC)[reply]

IFT TRANSFORMER

WHY IF=455 INTERNAL CIRCUIT DIAGRAM & WORKING..........???????????

First, it's considered rude to type in all caps becuase it seems to be shouting.
Second, I understand if you haven't mastered the English language. But, even presuming (and since I don't know the technical terms I will guess and give you the benefit of the doubt) that an "IF=455 internal circuit diagram" is your subject, you have not provided us with even a verb to go with it, let alone a question. Yes, "work" can be a verb (action word), but to be understood, you must use the form "work," and ask "why does this circut diagram work in this way?" Then, you need to give some context to your question - what about the way it works confuses you?
If it's another question that you have, you will need to phrase it the same way - with a subject (what you're asking about) and a verb or verb phrase (what it does/doesn't do that you're unsure about.)
So, phrase it that way, so we can understand and help you. Oh, and below "save page" you'll see a list of things you can insert - at the right are the tildes you should sign your question with. Thank you.209.244.187.155 (talk) 12:32, 20 October 2009 (UTC)[reply]
The questioner likely wants to know why 455 kHz is used for the intermediate frequency in superheteropdyne AM radio circuits, the most common type of Amplitude modulation radio since the late 1920's. The use of an intermediate frequency allows improved frequency selectivityThe radio frequency signal, which can vary over a wide range, is converted to one constant intermediate frequency (typically 455 khz). It is easier to amplify the constant IF frequency and to detect the signal than is several stages of amplification must function at the original radio frequency. Does the questioner want a simplified verbal explanation or a technical mathematical explanation? Read the articles linked above, and come back with any additional questions. Another explanation is found at "Consumer electronics for engineers pages 12-13. Edison (talk) 15:04, 20 October 2009 (UTC)[reply]
IF=455 is someone's shorthand for "Intermediate Frequency = 455 kHz". That frequency is usually chosen in small AM radios because it is a fair compromise between 1) ease of constructing high-Q resonant transformers in the radio IF amplifier; 2) high enough for acceptable image frequency rejection by the radio front-end tuned amplifier; 3) by being a de-facto standard there should be no strong transmissions at 455 kHz and components are standardised e.g. fixed-frequency ceramic filters are now often used instead of wound coils. In my experience 455 kHz is the commonest IF in Japan and US while in UK (when they made radios) it was 465 kHz. Cuddlyable3 (talk) 16:16, 20 October 2009 (UTC)[reply]

Neutronium

Science fiction has loved to use neutronium for all sorts of purposes, plating on shields for instance. Is it possible, even in principle, to use neutronium as an engineering material in small components? Or will it immediately return to "normal" matter in a spectacular reaction if by some magical means it is suddenly removed from a neutron star? SpinningSpark 07:01, 20 October 2009 (UTC)[reply]

I don't think this is a scientifically meaningful question. Neutronium is almost exclusively a science fiction concept, so answering questions about it would basically be a science fiction answer. As the neutronium article states, the term "neutronium" is rarely if ever used in the scientific literature, and there is no agreed-upon definition for the term. Science fiction uses the term to mean the material present in the cores of neutron stars, but the composition of the material in the cores of neutron stars is uncertain. Supposed bound clusters of neutrons like tetraneutron are not supported by current models of nuclear forces, and the one experiment that supposedly suggested its existance can't be replicated. So trying to answer questions about neutronium's nature scientifically is almost as hard as trying to come up with scientific answers about kryptonite. Red Act (talk) 07:39, 20 October 2009 (UTC)[reply]
Sorry, but I think that's evading the question (but I'm happy if the answer is simply we don't know). There is no known source of kryptonite, or, as you say, any meaningful definition of what it is. However there is such a thing as a neutron star and it must be made of some material, and there are at least theories as to what that is. If it makes you happier let's not call this material neutronium. If by some magical (ie unspecified) means I place 1 kilogram of neutron star material on my desk, what will happen? My guess would be a rather nasty explosion since the material is no longer under the pressure required to maintain that density, but I would rather like a scientific answer. SpinningSpark 14:09, 20 October 2009 (UTC)[reply]
You might want to read about Neutron temperature to bring yourself up to speed on what is scientifically known about neutrons. The rule of thumb I got from a physics professor (which is very approximate), is that "thermal (slow) neutrons are nearly impossible to detect", and "fast neutrons are for bombs." (...because of the role they play in the chain reaction of nuclear fission). The key point to remember is that a very large chunk of our sensory perception of materials comes from electrostatic and electromagnetic interaction - color, volume, texture - these sensory properties are the result of macroscopic averages of microscopic interactions with the electromagnetic force. Neutrons, categorically, are uncharged - so it will be hard to define their volume; they probably will not have a well-defined color; (though they have a magnetic moment, and certainly can interact with electromagnetic waves, there probably will not be spectral lines like those that yield color in other materials). They'll probably be more like an invisible noble gas (but not quite). If you could imagine pulling a 1 kg lump of neutrons out of a giant source of neutrons, you'd need to know their temperature to estimate their thermal diffusion rate - eventually, they would dissipate. Keep in mind that neutrons can pass right through most matter, excepting the statistically small fraction that collide with a nucleus. If the neutrons have low energy, they will just bump their way through all the atoms they encounter and "fall" through your container (if you tried to store them in a box). If they have high energy, they can really muck up the atomic nuclei of the container (fissioning them, for example, but probably not yielding a chain reaction unless the material is conducive to that). You'd have to be pretty creative to confine the neutrons - they're fairly elusive particles. Nimur (talk) 14:29, 20 October 2009 (UTC)[reply]
That's all very interesting Nimur, but it's not an answer to the question asked. So let me address that. Neutronium matter wouldn't be stable without the assistance of the gravitational atraction and your kilogram of neutronium would likely explode. Dauto (talk) 16:04, 20 October 2009 (UTC)[reply]
Thermal neutrons do not explode. They drift. The qualitative behavior will depend entirely on the neutron temperature. That is why I suggested that the starting place might be the neutron temperature article. Nimur (talk) 17:01, 20 October 2009 (UTC)[reply]
I think that thermal neutrons at high density (which is basically what the OP is implying by "neutronium") drifting apart would constitute an explosion. Think of what happens when a tank of compressed gas gets broken open. Rckrone (talk) 17:51, 20 October 2009 (UTC)[reply]
All those decaying neutrons would be producing fast moving protons and electrons, a hydrogen plasma at an extreme temperature, it could be like trying to contain a piece of the sun, just for the decay products. Graeme Bartlett (talk) 20:43, 20 October 2009 (UTC)[reply]
According to this version of the Neutron star article:
And this version of the Degenerate matter article has:
  • "Neutron degeneracy is analogous to electron degeneracy and is demonstrated in neutron stars, which are supported by the pressure from a degenerate neutron gas."
Which is interesting because it describes "the Pauli exclusion principle" (and, more specifically, "neutron degeneracy") as if it constitutes a "Force." The question I have, then, is: how does this "force" then relate to the "four fundamental forces"...?
But, to restate the original question above, I think: would this "force" cause the neutrons to accelerate away from each other if a small amount of the matter at the core of a neutron star were somehow liberated from it and removed to some region of less gravitational intensity? Wouldn't that then, in a simplified sense, be just a matter of the "potential (gravitational) energy" of the neutrons in our sample being suddenly converted to kinetic energy? Or just like a hot molecular gas being released suddenly from containment to expand into a larger volume, I suppose, as I far as what I understand Graeme Bartlett to be saying in the remark above. But I'm not sure the analogy really holds here; for example, given the "quantum degeneracy" of the material, isn't it possible for that kinetic energy to perhaps be expressed in the same direction for all the neutrons?! (LOL!;)
I guess it doesn't really make much sense to try to imagine what would happen in a case like this in the absence of any plausible way for it to happen -- but, then again, that didn't stop Einstein, right?! Wikiscient 00:31, 21 October 2009 (UTC)[reply]

Featured Article - Diamond

In todays featured article it explains that synthetic diamond is used in heat sinks. But in the heat sink article, under 'construction and materials' there is no mention of diamond. When would synthetic diamond be used for a heat sink?91.109.234.25 (talk) 08:20, 20 October 2009 (UTC)[reply]

Diamond makes a good heatsink since it has good thermal conductivity and poor electrical conductivity. However, it has the disadvantage that it's rather more expensive than copper plus a thin insulator. So it is used as a heat sink where the components themselves are expensive - we used it in the 1970s to heat sink semiconductor lasers, which themselves cost around £1,000 even then. --Phil Holmes (talk) 09:18, 20 October 2009 (UTC)[reply]
There is currently a lot of work being done on ways to manufacture diamonds in a (relatively) cheap and predictable manner for purposes such as that. It's hard figuring out the best shapes, but they're beginning to make progress. ~ Amory (utc) 17:53, 20 October 2009 (UTC)[reply]

Artificial "gills" - getting oxygen from water on the fly...

Reading the helpful links about SCUBA and various gas mixes up above led me to wonder about the current hurdles yet to be cleared in creating what would essentially be artificial gills - a device that pulled oxygen straight from the surrounding water allowing a human to breathe submerged for extraordinarily long periods of time. Surely there is tremendous financial potential for such an invention. Does this fall into the category of "as yet impossible, but not theoretically so" or is it lumped in with time travel as "theoretically impossible" ? 218.25.32.210 (talk) 08:50, 20 October 2009 (UTC)[reply]

See Artificial gills (human). Red Act (talk) 09:07, 20 October 2009 (UTC)[reply]
It's probably a matter of economics. There usually isn't a need to put a human underwater for such a period of time. Compressed gas is well-known, easy to use, and economical. You might want to note that nuclear submarines (which can stay submerged for a long time) are limited by their foodstocks, not their air supplies. Nimur (talk) 14:34, 20 October 2009 (UTC)[reply]
Also note that, as mentioned in the item above, pure oxygen is not what you really want to be breathing. Just extracting and breathing oxygen might be safe very near the surface where you aren't under significant pressure, but maybe not for long periods, and certainly not at any depth. I presume the water also contrains large amounts of nitrogen in solution, but you'd need a device that extracted nitrogen and oxygen and produced the correct mixture. And below a certain depth nitrogen isn't safe either, and then you need to bring rare gases like helium to mix with our breathing air, because you'd never find enough of those in the water. --Anonymous, 02:50 UTC, October 21, 2009.
The problem, far as I recall, is that there's less oxygen per volume in water than air. This is why air-breathing whales have been able to grow larger than any fish ever did. So an artificial gill wouldn't just have to be efficient at extracting water, it would have to circulate a lot of water to keep a warm-blooded mammal like our selves happy. That takes power. What we're offering the scuba diver is then a contraption with heavy batteries that run out of power, instead of a tank of air that runs empty. Not an attractive proposal. Nuclear submarines as far as I know, do not extract dissolved oxygen from the water, but splits the water into oxygen and hydrogen by electrolysis. This is also currently the case onboard the international space station. The needed oxygen can be recycled as humans do not consume atmospheric nitrogen. EverGreg (talk) 08:41, 21 October 2009 (UTC)[reply]
It's surprising that a fish can grow as large as the whale shark or the basking shark. But if cold-blooded creatures require less oxygen, the question is how much less? Or maybe the answer is obvious - is the need in the same ratio as the amount? That is, is there an upper bound on how large a fish could get before it would suffocate due to being unable to get enough oxygen from the water? ←Baseball Bugs What's up, Doc? carrots12:26, 21 October 2009 (UTC)[reply]
yeah the thing is that gills have a surface and bodies have a volume. The gill surface tells you how much oxygen you get in, the body volume how much you need. The trouble is that when you increase the size of something, the volume rises more sharply than the surface area. (try out the math with a cube or a sphere if you like) Insects also depend on their surface to get oxygen. So the reason we don't have bluewhale-sized fish is actually the same as why we don't have dog-sized insects: geometry. EverGreg (talk) 14:11, 21 October 2009 (UTC)[reply]
Mere speculation. I think the basking shark example cited is blue-whale-like enough. And dog-sized insects would likely be a greater issue of lack of sufficient support from an exoskeleton rather than an issue of respiration -- more spiracles could provide an adequate tracheal system. DRosenbach (Talk | Contribs) 03:40, 22 October 2009 (UTC)[reply]

Michelson-Morley Experiment Simulation

Can you please check this Flash Animation carefully? Does it simulate the light propagation response with respect to the assumed aether movement precisely?--Email4mobile (talk) 10:48, 20 October 2009 (UTC)[reply]

Only in regards to the somewhat arbitrary units they are using (pixels per second, "frames", etc.). The concept is correct (although regarding light as "particulate" to such a degree is not historically valid for the actually M-M experiment, but it doesn't matter much), and it's only meant to illustrate the concept. --Mr.98 (talk) 12:23, 20 October 2009 (UTC)[reply]
I had some difficulty rotating the table because its angle jumped unexpectedly. At one point when I wanted 45 it was changing 44.5, 45.5 degrees etc. It seems the "aether evidence" one needs to see is the staggered emergences of the red and blue arrows. In the real experiment one would see an interference pattern of rings at the target and be looking for any drift in the rings as one rotates the table. I suggest that it would be more instructive to have the table rotate when one clicks on Play. Cuddlyable3 (talk) 15:57, 20 October 2009 (UTC)[reply]
It looks wrong to me. In the default orientation the horizontal speed of the light should be (where v is the wind speed), but it looks like it's just c. In fact, when v is set higher than c the light shouldn't be able to move horizontally at any speed, but the applet still shows it going at c. -- BenRG (talk) 23:06, 20 October 2009 (UTC)[reply]

Gargling salt water to prevent swine flu

I got a forward that says gargling warm salt water twice a day will help prevent swine flu because the virus takes 2-3 days in the throat to proliferate. Is this true? It also says drinking warm liquids has the same affect except it takes the virus to your stomach where it can not survive. Thanks. StatisticsMan (talk) 12:35, 20 October 2009 (UTC)[reply]

The answer is the same as with any fact given in a chain email - no. I have yet to see a single true statement in a chain email, it just doesn't happen. --Tango (talk) 12:57, 20 October 2009 (UTC)[reply]

::If you inhaled the salt water and died it might reduce the chances of you passing the swine flu onto someone else? --BozMo talk 13:00, 20 October 2009 (UTC)[reply]

While you should not take information from a chain email at face value, they are also telling us this over the radio. Lovely media, passing on unverified information as truth. The CDC does not have this listed under their suggestions to prevent H1N1 spread, so I would take that information with a grain of salt. Actually, if I can find an audio of them saying it, we can include it in our article as it would be a reliable source! Googlemeister (talk) 13:03, 20 October 2009 (UTC)[reply]
Must... avoid... grain of salt... salt water gargling... pun... --Mr.98 (talk) 23:56, 20 October 2009 (UTC)[reply]

I feel confident in saying that method should work - provided you've also had your H1N1 flu shot. :) ←Baseball Bugs What's up, Doc? carrots13:08, 20 October 2009 (UTC)[reply]

I would say that there is probably a grain of truth in the salt, but I wouldn't like to rely on the method. Dbfirs 13:23, 20 October 2009 (UTC)[reply]
Gargling salt water can theoretically kill bacteria, but it's only really effective for cleaning pus off of the back of your throat and for making a sore throat feel better. Warm liquids will help you, but not for the reason given - it helps because drinking liquids is always good, and most people are usually slightly dehydrated anyway. Drinking water is essentially the first step in the cure for nearly everything that can happen to you. ~ Amory (utc) 13:36, 20 October 2009 (UTC)[reply]
Also, different influenza stains bind at different parts of your respiratory tract. Avian flu is so deadly (partially anyway) because it essentially binds in the lungs, and causes pneumonia. Your average flu usually binds to the throat, so since salt water won't cure the flu this remedy therefore can't be true. ~ Amory (utc) 13:39, 20 October 2009 (UTC)[reply]

Minimum population to restart the human race?

Assuming we want to minimise inbreeding what's the minimum amount of people you'd have to have to restart the human race, after say... a nuclear war or something? A source or computer model study or something would be useful. I've read wikipedias article on Minimum viable population but didn't really understand it, and couldn't find a solid year 0 number. Gunrun (talk) 13:10, 20 October 2009 (UTC)[reply]

The solidity of a "year 0 number" will depend entirely on the solidity of your definition of inbreeding. Go ahead :-)
DVdm (talk) 13:17, 20 October 2009 (UTC)[reply]
(edit conflict) The absolute minimum is 2 (or even 1 with modern technology), but, to minimise inbreeding, I would guess at about 100 because I know of populations around a hundred that have mainly inbred without significant consequences. You will get different answers depending on the amount of inbreeding you are prepared to tolerate. Dbfirs 13:21, 20 October 2009 (UTC)[reply]
(edit conflict) Well, obviously 2 wouldn't do, because that would require mating of siblings (and/or mating between parent and offspring), which would probably constitute inbreeding. Bus stop (talk) 13:23, 20 October 2009 (UTC)[reply]
Generation ship#Biology and society discusses this issue, although in a different context. It has references too. --Tango (talk) 13:33, 20 October 2009 (UTC)[reply]
This is horribly useless information, but a friend of mine spent a number of months performing research in Australia and in her spare time would explore the Outback. Long story made short, whether by meeting them or otherwise, she learned that the natives have, over the years, developed a system of reproduction that allows for safe breeding. Moreover, it is supposedly the/an efficient way to sustain a viable population given the absolute minimum of human inhabitants. I have never found any proof outside of her, so you may have to travel around Australia to prove this. ~ Amory (utc) 13:58, 20 October 2009 (UTC)[reply]
But Amory, you didn't tell us what the system actually consists of. SpinningSpark 14:29, 20 October 2009 (UTC)[reply]
Jah, I don't know. The there's some plan for what kind of cousins can marry, etc. It sounds fascinating and I'd love to know, but "a friend heard from an aboriginal" isn't exactly a good source. ~ Amory (utc) 15:44, 20 October 2009 (UTC)[reply]
Unless you can tell use something about how the system works or what evidence there is that it does work, it is entirely useless and I don't know why you mentioned it... --Tango (talk) 17:16, 20 October 2009 (UTC)[reply]
Perhaps they mentioned it in the hope of inspiring some else to think "aha" and find a source they already know exists? Franamax (talk) 18:42, 20 October 2009 (UTC)[reply]
According to Engels, among hunter-gatherer societies, the gens system was used and I assumed the Australian system referred to by Amory was some variant of this. In its purest form, women stayed within the gens, whilst men were obliged to marry outside the gens and moved into their wife's gens. This system results in some strange rules on incest to modern eyes. Intercourse with any woman within ones own gens is incestuous, cousins no matter how many times removed are forbidden if they are in the same gens. On the other hand a cousin in another gens is perfectly legal, as would be ones paternal grandmother (although probably not first choice in most cases). SpinningSpark 20:18, 20 October 2009 (UTC)[reply]
I would take anthropological information from 19thC armchair researchers like that with a huge heap of salt. Did he ever go to Australia? See an Aborigine? 19th century stuff of that type was almost completely hypothetical; they were working within the conceit of people existing in the so-called state of nature (i.e. wandering around after leaving the garden of Eden). I don't know if the information is correct or incorrect, but would bet the house that it was baseless. Matt Deres (talk) 22:25, 20 October 2009 (UTC)[reply]
475 according to this book, although what the odd one is supposed to do after all the other 474 have paired off I'm not at all sure. SpinningSpark 14:55, 20 October 2009 (UTC)[reply]
That is because you are assuming an even M/F ratio. If it was 1M/4F, things would be even. And work out very nicely for the males by the way. Googlemeister (talk) 15:02, 20 October 2009 (UTC)[reply]
Gosh, that did not even occur to me! I believe the original paper the book is referring to is this and there is also this by the same author SpinningSpark 15:05, 20 October 2009 (UTC)[reply]
Pitcairn Island is a good case in point - their population is essentially composed of descendents of some female islanders from Tahiti and a few men who survived the Mutiny on the Bounty. They coped for many generations before outsiders joined the community - and even now, it's likely that few if any outsiders have added to their gene pool. So the answer is considerably less than 100 individuals. As others have pointed out, modern technology could solve a lot of potential problems if the people involved were prepared to produce children to a prescribed plan to ensure that the gene pool were used to it's utmost. SteveBaker (talk) 14:56, 20 October 2009 (UTC)[reply]

Maybe population bottleneck and Small population size might be useful articles on the subject? Also, they say the 120,000 moose in Newfoundland are descended from 4 brought over in 1904. TastyCakes (talk) 15:30, 20 October 2009 (UTC)[reply]

I find the question what is the minimum number of people needed to sustain a modern technological society, I would assume that there is a a bifurcation point (catastrophical collapse) when nobody can make anything due to the lack of someone else's expertise (wikipedia hates how-tos so we can't help!) --Squidonius (talk) 17:35, 20 October 2009 (UTC)[reply]
Mice are very different when it comes to breeding than humans. You can breed siblings with each for generations without worrying to much about all the deleterious effects humans see after even just one generation. In fact, modern biological research largely depends on that fact. ~ Amory (utc) 17:56, 20 October 2009 (UTC)[reply]
Is that because mice are genetically simpler (or have fewer recessive gene problems) or because we are much more sensitive to "problems" in inbred humans than mice? TastyCakes (talk) 18:11, 20 October 2009 (UTC)[reply]
Both, definitely, although much more of the former. The human genome is full of recessive alleles that would all but kill us after a generation or three of pure inbreeding. In all honesty, we probably know much more about murine genetics than human, so we can probably spot a problem with them faster. However, as you say, it's a lot easier to visually identify an abnormal human than a mouse, although we are pretty good at it; besides, the different speed of aging makes it easier. ~ Amory (utc) 18:34, 20 October 2009 (UTC)[reply]
That is only true of laboratory mouse strains (which are essentially clones of each other). Wild mice (and most other non inbred animals) are just as likely to suffer from inbreeding as humans are. Rockpocket 01:24, 21 October 2009 (UTC)[reply]
Wouldn't it just be the descendants with two recessive alleles that died? Franamax (talk) 18:55, 20 October 2009 (UTC)[reply]
I would suggest that a) labmice have been pretty well culled of individual with recessive genetic defects by many generations of inbreeding and b) that we possibly tolerate 5% of mice with heavy genetic illnesses, but we do not find that acceptable in humans. --Stephan Schulz (talk) 19:08, 20 October 2009 (UTC)[reply]
My understanding is that inbred mouse lines are maintained precisely for the purpose of maintaining specific recessive defects. They are outcrossed to preserve the recessive allele, then incrossed to produce the syndrome of interest. Am I wrong there? Franamax (talk) 20:11, 20 October 2009 (UTC)[reply]
Properly inbred mice strains (such as BALB/c) are homozygous at all loci, therefore they have no recessive alleles (since there are no other alleles for them to be recessive to!) But it is true that some of the recessive alleles that were originally in the founding population are now fixed in the strain. During the inbreeding process, the mice with two copies of the harmful recessive alleles would have died. It was the ones that survived that were bred on. Therefore, by a type of natural selection (in the hands of the breeding scientists), the nasty lethal alleles would be weeded out and the non-lethal ones remained. The same thing would likely happen with a human bottleneck. The issue is whether there are either enough people, or few enough lethal recessives in the founding population, that the right assortments occurred to ensure there were individuals that dodged that bullet. A healthy dose of luck would help too since we probably wouldn't know if the right mating choices were made until it was too late to do anything about it. Rockpocket 01:19, 21 October 2009 (UTC)[reply]
Assuming a simple case, yeah, but you've got the right idea. That's why it's so deadly - once something negative gets expressed there's no chance for new genes to come in. Looking at a really simple case of an autosomal recessive deleterious gene, if only one parent carries the gene then the chance of a child getting a copy is 50%. That means there's a 25% that both siblings have the gene after only one generation. ~ Amory (utc) 19:27, 20 October 2009 (UTC)[reply]
Side note: I'm still trying to figure out how this sub-thread went from moose to mice. Shouldn't we be talking about "meese", the plural of moose? :) Franamax (talk) 20:19, 20 October 2009 (UTC)[reply]
I'm pretty sure it's mooses. Or possibly Moosae. TastyCakes (talk) 20:50, 20 October 2009 (UTC) [reply]
It's about to change again. All the billions of Rabbits in Australia, a major national pest, are descended from 24 rabbits released by a well-meaning Englishman, Thomas Austin, in 1859. -- JackofOz (talk) 20:30, 20 October 2009 (UTC)[reply]
If you buy the Toba catastrophe theory, then the human population recovered from a bottleneck of somewhere between 2,000 and 20,000. Rockpocket 01:42, 21 October 2009 (UTC)[reply]
Searching for minimum human population finds an assortment of material. Including, in the case of a project to populate a planet, the minimum is two females if they have sufficient quantities of stored fetuses, eggs, and sperm. -- SEWilco (talk) 16:19, 26 October 2009 (UTC)[reply]

Inbreeding

In a case like described above, would it really be a problem for siblings or offspring or whatever? I'm talking strictly biological/genetic problems. I know that long-term inbreeding can cause problems like Charles II, but with a continually expanding population, the severe inbreeding wouldn't occur after a few generations. So with an initial population of 2, would it be nearly as bad as the Habsburgs? —Akrabbimtalk 18:45, 20 October 2009 (UTC)[reply]

If you are starting with only 2 genomes, you are by definition inbreeding, no matter what the eventual population size. The only "new" genetic material would be novel single-nucleotide polymorphisms and syntenic reshuffling / gene duplications introduced by retrotransposons. These processes introduce genetic diversity but act slowly over many generations. Franamax (talk) 19:01, 20 October 2009 (UTC)[reply]
A population with so little genetic diversity has difficulty adapting to changes in environment. For example, if a new disease pops up, chances are good that nobody in the population will have any natural immunity and the whole population will be wiped out. Usually you get a few people with immunity that can repopulate, thus avoiding extinction. Alternatively, if everyone is the same height and suddenly the short trees all die and you need to pick your fruit from taller trees there won't be any tall people to pick them and the species will die out, usually it would just evolve to be taller. (Obviously, both those issues can be overcome with technology, which is why people above are mainly talking about recessive congenital problems, which are harder to deal with technologically. For non-technological species, it is a bigger problem.) --Tango (talk) 20:25, 20 October 2009 (UTC)[reply]
Adam and Eve? ;) Rockpocket 01:04, 21 October 2009 (UTC)[reply]
What Franamax and Tango have said is valid, but that's not what "causes" problems like that of Charles II (or the well-known hemophilia suffered by many other inbred royals).
The real problem there is (very poorly) discussed in Inbreeding#Allele exposure. Basically, in inbred populations individuals are more likely to inherit a maladaptive "recessive" allele from both their parents -- and so, consequently, those individuals are more likely to suffer from the maladaptive traits encoded by those alleles and "more likely" (of course) to pass on those same recessive alleles to any offspring they produce (if any...). Note that in the larger population from which that inbred group came, individuals are much more likely to inherit at least one "dominant" allele (and will in turn pass that on to at least about half their offspring on average) and therefore will not suffer (as much) from such maladaptive traits (even if they inherited the maladaptive "recessive" allele from the other parent).Wikiscient 01:49, 21 October 2009 (UTC)[reply]
Indeed, as I said the problem of bad recessive alleles is more significant for a technological society (or when we are only considering a small subset of the population, as we are with royal families). For other species, lack of genetic diversity can be much more of a problem since it can cause complete extinction rather than just a quarter (if the good and bad alleles are equally prevalent, which is unlikely to be true, but nevermind) of the population having a congenital defect. --Tango (talk) 10:31, 21 October 2009 (UTC)[reply]

storing freshly charged batteries

What is the best way of storing ( safely and ready for use) all those charged AA, AAA cells freshly out of the recharger?--79.67.32.188 (talk) 17:18, 20 October 2009 (UTC)

(Question restored, because without it the rest of the thread is incomprehensible. Please NEVER delete a very sensible and reasonable question while leaving the responses. Per WP:IAR) Edison (talk) 04:49, 21 October 2009 (UTC)[reply]
I use these cases. Keep in mind that NiMH rechargeable batteries will self-discharge in storage. Some (typically the highest capacity versions) will discharge significantly within days. Others are designed to discharge more slowly, and may be usable for months. So keep this in mind when storing them. -- Coneslayer (talk) 17:26, 20 October 2009 (UTC)[reply]
It depends what you mean by 'ready for use'. Keeping rechargeable batteries in the freezer drastically lowers the self discharge rate but you should not use them until allowing them to warm up to about room temperature. [1] [2]. Also you need some way to store them avoiding short circuits, for example the containers Coneslayer showed. Also as Coneslayer mentioned, many NiMH have a high self discharge but while it's currently true low self discharge batteries are lower capacity then the highest capacity versions, the best way to tell if they are low self discharge is if they are sold as such. It's likely any battery not sold as a low self discharge battery will not be a low self discharge. As low self discharge batteries were only commercialised about 4 years ago, it's save to assume any battery older then that is not one no matter the capacity. Also because fakes are not uncommon, look out for known reliable brands and from decent retailers. Nil Einne (talk) 18:04, 20 October 2009 (UTC)[reply]
Re: "It's likely any battery not sold as a low self discharge battery will not be a low self discharge.": Actually, "pre-charged" batteries are marketed under a variety of names; see Nickel-metal hydride battery#Low self-discharge cells. — Sebastian 19:17, 20 October 2009 (UTC)[reply]
I think we need to distinguish between marketing terms and useful information. There's little point IMHO giving a list of low self discharge battery brands names since it's likely to be incomplete. But any low self discharge battery will mention or imply its low self discharge characteristic even if it doesn't call it self a low self discharge battery or use the specific terms. E.g. Sanyo Eneloop "Once charged, eneloop retains its charged capacity even after 6 or 12 months of storage" Maha Imedion "IMEDION can be stored for an extended period of time without substantial loss of power and are ready to use out-of-package" Panasonic Infinium "365 days Standby Power: “Use them, Charge them, Store them, Use them”". I admit, I should have been clearer and in particular said that they would likely either describe a low discharge or mentioned them retaining charge for a long period of time (I considered it but have a tendency to write long and confusing answers so decided to avoid it) and apologise for any confusion but I would also hope anyone reading the article and using the RDS understands that if a battery mentions that it has a low discharge, keeps a high charge over 1 year or whatever is a low self discharge battery even if it doesn't specifically call itself a low self discharge battery. Understanding and cutting through the bullshit of marketting hype is IMHO an important life skill. Incidentally precharged is an exceptional poor name. You don't need to have a low self discharge to be sold precharged and I'm pretty sure if you went back to before low self discharge batteries you will find batteries that mention how they are precharged. The charge may not last long so even if they are precharged it may not be of much use. (In any case I'm not convinced many people even care if they are pre-charged, the only case would be if someone finds they urgently needs batteries and decides they could do with some new rechargables.) Some companies may use the term to mean they remain charged when you want them (if you charged them before hand) however it's still a rather confusing terminology which is IMHO best avoided particularly on the RDS. In any case I stick with my main point. Any battery which does not mentionr imply it has a low self discharge characteristic, whatever way it may market the feature is unlikely to be low self discharge. If you come across some random low capacity battery, don't expect it to be a low self discharge just because it has a low capacity. Nil Einne (talk) 20:36, 20 October 2009 (UTC)[reply]

Um, we seem to have had the question deleted but at least some of the answers are still here. Under what circumstances could that be correct? (Maybe this belongs on the talk page, but I'll ask it here.) --Anonymous, 02:56 UTC, October 21, 2009.

The question was asked by a banned user, but didn't get deleted before several well-intentioned editors had provided detailed answers, and, bleeding heart that I am, I couldn't bear to delete their work along with the question. (And, yes, any further discussion should probably be on the talk page.) —Steve Summit (talk) 03:55, 21 October 2009 (UTC)[reply]

Human's scent

I am looking for general information about human's personal scents...of the type a dog or cat would recognize one by or like you might smell on someone's clothes. I looked in my psychology book and found a whopping one line on the subject: "As any dog or cat with a good nose could tell us, we each have our own identifiable chemical signature." Our Wikipedia article has even less information (unless I'm missing something?). What exactly is this chemical signature? Why is it not possible to smell your own scent? Has any psychological/biological research been done on human scent? Finally, in theory, would it be possible to take a shirt that's never been worn, have someone sleep in it, run a piece of the shirt through some chemical analysis process (perhaps Mass spectrometry?), discover the composition of the chemical signature/scent, and (this is probably stretching it) perhaps synthesize it? Ks0stm (TCG) 18:57, 20 October 2009 (UTC)[reply]

Well, you can run the t-shirt past a woman's nose. Scents apparently reveal details of the human immune system. Google "mhc scent human compatibility" and you will see lots of results. Franamax (talk) 19:21, 20 October 2009 (UTC)[reply]
Its easy to smell your own scent. Rub your finger on the scent glands behind/above your ear (for example). Smell your finger. As a student I seem to remember you could easily tell the gender of the person from the scent rubbed on a cotton bud that way and quite a lot else besides (often you can identify the person) --BozMo talk 20:53, 20 October 2009 (UTC)[reply]
Ahh, so that's why they tell us to wash behind our ears? They're teaching us to erase the signs of our own unique existence? Franamax (talk) 22:41, 20 October 2009 (UTC)[reply]
My mum teaches 4 year olds and apparently she often has someone in the class with really sensitive smell who can smell a jumper found in the classroom, say, and immediately say whose it is. --Tango (talk) 21:35, 20 October 2009 (UTC)[reply]
Richard Feynman describes his skill at this particular art in Surely You're Joking, Mr Feynman!. Our noses are actually fairly good, we just rely on our eyes for practically everything, so we're not usually aware of scents unless they're extremely strong. Tevildo (talk) 22:32, 20 October 2009 (UTC)[reply]
Individuals do have their own gestalt of odors that are the combined product of exocrine secretions. Once could ask someone to wear a t-shirt then do mass spec in the manner you suggest and get a spectrometric read out. However, the chances are if you did the same experiment the following day with the same person, you would get a different molecular profile. Why? because the content of out odor secretions will vary with diet, time, health, body flora, temperature, stress etc. So we do not each have a unique, permanent odor signature. That said, it is likely that different individuals would have greater variation than the variation within an individual. So there is some element of individuality.
There is quite a lot of information out there on molecules hypothesized to be involved in human odor identity. The major histocompatibility complex (MHC) has long been suggested to be involved, though a direct link has yet to be shown in humans (see Claus Wedekind, for some indirect evidence). Other molecules have been claimed to be human pheromones, such as estratetraenol and androstadienone, though again, direct evidence is lacking. In reality, the biggest factor in human odor variation is likely to be diet; to put it crudely - what comes in largely comes out, you change what you put in, you change what you get out.
In contrast, genetically encoded olfactory signatures are much better defined in animals, and we know of numerous chemicals that transmit specific olfactory information about others (I'm actually writing an article on some of these at the moment, its incomplete, but you there are plenty of sources you could read ). If you want to learn more, there are interesting chapters in Olfaction, Taste and Cognition by Catherine Rouby et al [3] and Pheromones and Animal Behaviour by Tristram Wyatt [4] Rockpocket 00:55, 21 October 2009 (UTC)[reply]
Incidentally, there has been a lot of research on the best ways to store human scent. In the pre-DNA days, it was used by police forces to put individuals at crime scenes, as specialized dogs could determine whether a given scent at a crime scene matched up with a given scent sample (sometimes taken surreptitiously with specialized chairs). The Stasi had a large scent archive for this reason. Apparently the dogs were quite good at it, and could track people based on very old scents that had been stored (on specialized cloth, in mason jars). --Mr.98 (talk) 15:52, 21 October 2009 (UTC)[reply]

The last of its kind...

Is there a specific term used to describe species which, whilst not technically extinct at present, are only comprised of a single individual and thus inevitably will be (barring cloning or that hypothetical genetic manipulation of a closely related species)? Something more specific than functional extinction? --Kurt Shaped Box (talk) 21:49, 20 October 2009 (UTC)[reply]

I have heard read the term "non-viable population" for a scenario where an extinction pro futuro is but a matter of time. --Cookatoo.ergo.ZooM (talk) 22:08, 20 October 2009 (UTC)[reply]
I've heard the term "genetically dead" being used about an animal that will not be able to reproduce. That would usually be the case for the last member of a species. EverGreg (talk) 08:26, 21 October 2009 (UTC)[reply]
A similar situation occurs in a very different field: that of disappearing languages. This [site] states that

"According to Unesco's Atlas of the World's Languages in Danger of Disappearing, a language is considered endangered when it is no longer spoken by children, moribund when only a handful of elderly speakers are left, and extinct when it is no longer spoken. The numbers vary by source, but even the most optimistic estimates are alarming, with half of the world's languages struggling to survive. Some sources declare 5,000 of the 6,000 total in some state of endangerment."

Michel M Verstraete (talk) 19:22, 21 October 2009 (UTC).[reply]

The word you need is relict. HTH, Robinh (talk) 21:37, 21 October 2009 (UTC)[reply]
You nailed it! :-D EverGreg (talk) 09:06, 22 October 2009 (UTC)[reply]

Dishwashers

Y'ever had a question you wanted to ask, but you knew that doing so would make you sound like an insane person? This is where I am now. Please bear with me a minute. We have a GE dishwasher that came with the house, we have hard water, but use a water softener. Both items seem to work just fine. We use the regular ol' Cascade dishwasher powder. Here's the thing: when we wash something in the dishwasher it takes much, much longer for it to dry than if we washed it by hand. Like, days versus hours longer; I've literally picked up tupperware bits with water still in them after 36+ hours have elapsed. On the other hand, items washed by hand tend to air dry in a couple of hours; we can put them away the same day we wash them. Now to top it off, here's the twist: if we take the items just washed by the dishwasher and give them a quick rinse, they dry more quickly than if we don't rinse them. If we open the door at noon and rinse a few of the items out at 6pm, those rinsed items will be dry by morning (say) while the other items will not be despite having six extra hours to dry.
I thought my wife was crazy when she said this was going on, but I've seen it time and time again. And yes, I've read the article on confirmation bias. If anything, I wanted to prove her wrong and had to eat my words! So after all that, my question is this: what on earth could case this set of observations? Could something in the detergent be preventing evaporation? That stuff should all get rinsed off anyway; a quick spritz of tap water shouldn't make a difference, right? Matt Deres (talk) 22:15, 20 October 2009 (UTC)[reply]

Are you putting the rinsed items back in the dishwasher to dry or on a draining board? Evaporation rates depend heavily on humidity and it will be more humid in your dishwasher than in the rest of the kitchen, even with the dishwasher door open (although with the door open the difference should be fairly small, so I'm not sure if that can completely explain it). --Tango (talk) 22:22, 20 October 2009 (UTC)[reply]
Water can "pool" in low spots in Tupperware, etc., and that could take awhile to evaporate. That stuff I take out, shake off the excess, and let it dry in the sink rack. Otherwise, the stuff in my dishwasher is pretty much dry after the drying cycle, so perhaps there's a problem with your machine. ←Baseball Bugs What's up, Doc? carrots00:09, 21 October 2009 (UTC)[reply]
That is a good point - most dishwashers I've used blast hot air at everything to dry it before the cycle finishes. There shouldn't be any need to wait for things to dry at all. --Tango (talk) 00:11, 21 October 2009 (UTC)[reply]
I don't usually take the dishes out right away except if I suspect there could be overturned (i.e. right-side-up) Tupperware and such to be extracted and air-dryed. Otherwise I leave it alone, except to let things cool off. ←Baseball Bugs What's up, Doc? carrots00:15, 21 October 2009 (UTC)[reply]
I'm answering a few things at once, so outdenting. We put the stuff back into the dishwasher, essentially back where it was originally. We don't use any kind of drying cycle on the machine and leave the door open (and trays out) when we're waiting for things to dry. The crevasses in the tupperware are unwaveringly the last things to dry regardless of which method we use, but I believe they exhibit the same behaviour (not sure about that one; will have to pay attention). I haven't tried pinning down which items are worst for drying, but plastic tumblers and ceramic mugs seem repeat offenders (ignoring the tupperware). I know it sounds barmy; I wouldn't didn't believe it either except that, as the guy who normally unloads the washer, it kept on getting hammered home. Imagine opening your dishwasher first thing in the morning (say, 7 am) and finding stuff still too wet to put away 12 hours later - and some stuff still too wet 24 hours later. That's not the case when I do dishes by hand - even, and I know this sounds even less believable, even if I use the dishwasher racks as drying racks for what I've hand washed. Matt Deres (talk) 00:37, 21 October 2009 (UTC)[reply]
Are you sure that both the water from the kitchen-sink-tap and the water-supply for the dishwasher are equally "water softened" by whatever system you use for that?
My experience has been that adding a product called "jet dry" to the dishwasher along with the detergent drastically decreases drying-time (whether with or without also using a "dry cycle" (ie., applying someheat after the final rinse)...). Not sure how it does that (and I don't have any on-hand at the moment to see what's in it) but I suspect that the presence or absence of "water softening" might have a similar (ie., chemical) impact on drying-times...Wikiscient 01:13, 21 October 2009 (UTC)[reply]
I'm not sure I 100% understand your question and subsequent clarification. To be absolutely clear, EITHER:
  1. The stuff washed in the dishwasher stays inside the dishwasher to dry - but the stuff that's hand-washed sits out on the draining board or counter-top or something. If that's what you're saying then the answer is easy - it's a lot more humid inside the dishwasher than outside - and that strongly inhibits drying.
  2. Dishes from within the washer are brought outside and still dry slower than hand-washed dishes. If that's what's happening then we have to look at what chemical goop is still dissolved in the water - that's a tougher answer. Detergents and dishwasher goop may be designed to either cause a reduction in surface tension (like hand-washing liquid detergents) - or to increase surface tension to cause 'beading' (like wax on your car) to allow the water to run off or be blasted off with air jets inside the dish-washer. With lower surface tension, the water spreads out over the surface of the dish - you get a larger surface area - so you get faster drying. With higher surface tension, you get spherical droplets - minimal surface area - slower drying.
A proper controlled experiment requires that you wash two identical dishes by hand and two more by machine. Take one of the machine-washed dishes and place it on the countertop where you normally dry hand-washed items - and take one of the hand-washed dishes and place it inside the dishwasher as if it had been machine-washed. Now - observe which ones dry first - let us know what happens!
SteveBaker (talk) 04:26, 21 October 2009 (UTC)[reply]
I got the impression from the OP that they use the racks of the dishwasher as the usual place to let dishes dry, whether hand- or machine-washed (I know I do).
Either way, another important control to run would be to hand-wash a set of dishes using machine detergent (and ideally vice versa though N.B. I do not think it advisable to put regular dish-washing soap in a dishwashing-machine -- it might make a mess or even damage your machine if you do!). I'm pretty sure it's got to be either the difference in the detergents used for hand- and machine-washing, respectively, or (less likely) a difference between sink and machine in the "softness" of the water supply for each.
Still curious to know what you find out! Wikiscient 17:37, 26 October 2009 (UTC)[reply]


October 21

How does the speed of light speed limit jive with physics shortly after the big bang?

I heard on a TV show that a microsecond after the big bang, the universe was about the size of our galaxy. That's a lot more than 3*10^8 m/s. 71.161.59.133 (talk) 01:43, 21 October 2009 (UTC)[reply]

Two answers. One is that the fabric of the universe itself is not limited to the speed of light. That is, special relativity does not prohibit the universe expanding faster than the speed of light, even if objects within the universe are prohibited from doing such. (Relevant article: metric expansion of space.)
But wait! If that's the case, why is the universe basically homogenous? That is, if it is expanding faster than the speed of light, then how did the initial light get spread throughout the existing universe pretty evenly? (Which it is.) For the currently accepted as the tentative answer, see the article on inflation (cosmology). Basically the most popular theory is that for a brief period, the universe expanded extra fast, and then slowed down again. This seems to jive with experimental evidence, though why it inflated, and why it stopped inflating, are not really understood (and there is a certain ad hoc aspect to the theory in general—it looks a lot like a "patch", even though it is pretty well accepted these days). This allowed the universe to stay rather small for awhile longer, becoming homogenous, and then to expand real quick-like. --Mr.98 (talk) 02:01, 21 October 2009 (UTC)[reply]
Just by the bye, I think the word you want is jibe. Jive is something else. --Trovatore (talk) 02:05, 21 October 2009 (UTC)[reply]
Hmm, you're right. Though it's funnier if they are really jiving. --Mr.98 (talk) 02:10, 21 October 2009 (UTC)[reply]
The Wiktionary article usage notes state that jive and jibe are interchanged in the U.S. - I have never heard the term "jibe" - so regardless of the authoritative "correct" usage, it seems that "jibe" doesn't jive around here... Nimur (talk) 13:35, 21 October 2009 (UTC) [reply]
It says "most sources consider [jive] an error". I am American, and also consider it an error. It's a mis-hearing, like "boldface lie" and "you've got another thing coming". --Trovatore (talk) 19:38, 21 October 2009 (UTC)[reply]
I see, the balloon analogy at metric expansion of space helped. So two objects on the surface of the "balloon" of space can move apart from each other at a rate faster than the speed of light if the "balloon"'s expansion is what's causing (or contributing to) their displacement. Too bad we don't know of a way to contract the fabric of space between two destinations so we can travel a long distance while still obeying the cosmic speed limit of c. 71.161.59.133 (talk) 22:03, 21 October 2009 (UTC)[reply]
Indeed. If we had some matter with negative mass (which we don't) we could make something like an Alcubierre drive, which is basically what you've described. --Tango (talk) 22:07, 21 October 2009 (UTC)[reply]
The Alcubierre drive is pure pseudoscience, though. All that Alcubierre did was write down a warp drive metric, plug it into the field equations of general relativity, get out a stress-energy tensor that, not surprisingly, makes no sense (the negative energy is the least of its problems), and then say "if we could create this stress-energy tensor that makes no sense, then we'd have a warp drive". That's the same as coloring the pixels on a computer screen so that they make a picture of the starship Enterprise and then saying "if we could make that in real life, we'd have a warp drive". You can write down any metric just like you can make any picture, but relatively few of them describe things that can happen in the real world. The Alcubierre drive definitely doesn't; the evidence for that is roughly as strong as the evidence against fairies in Dawkins' garden. -- BenRG (talk) 00:57, 22 October 2009 (UTC)[reply]
So what you're saying is, if you want to get to, say, Alpha Centauri in one week, your plan is to shrink the whole universe so that Alpha Centauri is right on our doorstep, pop on over, and then re-inflate the universe? At least no one can accuse you of thinking small.... --Trovatore (talk) 22:08, 21 October 2009 (UTC)[reply]
The way I was imagining it, you wouldn't have to deflate the whole balloon if you could just locally pull the rubber together in an area. But you'd probably want to contract an area of space without much mass in it, because that mass, if contracted a whole lot into a very small volume, might collapse into a black hole. 71.161.59.133 (talk) 22:18, 21 October 2009 (UTC)[reply]
The balloon analogy is perilously close to being outright wrong. When cosmologists say that the universe is expanding they just mean that the stuff in the universe is moving apart. Space, as such, doesn't expand. There is not and never has been (even during inflation) any "faster than light" expansion in any sense that matters. Faster-than-light speeds show up simply because the speeds used in cosmology are defined differently. Special relativity has "coordinate velocity" which is limited to c, but it also has "proper velocity" which has no upper limit. It's not a different kind of motion, it's just a different definition of the word "velocity". (The difference is that coordinate velocity is measured by synchronized clocks at the starting and ending points, while proper velocity is measured by a clock you take with you. Because of time dilation the clock you take with you measures a shorter time, so the velocity you get that way is higher.) The velocities in cosmology are analogous to proper velocities. Hubble's law says that galaxies at the edge of the visible universe are moving away at about 3 times the speed of light in every direction, and there are probably galaxies much farther away (that we can't see) moving at a billion times the speed of light in every direction—but you can nevertheless fit all of that inside a light cone, with the outermost galaxies moving slower than the boundary of the cone. It's just a different definition of speed. There's no violation of the light speed limit when the universe expands. There's nothing special going on at all. It's just stuff moving away from other stuff. -- BenRG (talk) 00:57, 22 October 2009 (UTC)[reply]
I'm not completely sure what you mean but as far as I can tell SR doesn't allow for anything like this. If you're moving at speed v relative to another object, you will perceive the object to be moving at speed v relative to you, and if that's how you measure your "proper velocity" then it still can never be greater than c regardless of time dilation. Cosmology requires some much bigger guns than just SR. Rckrone (talk) 16:19, 22 October 2009 (UTC)[reply]
Sorry, I didn't realize that proper velocity was a thing. Still, I'm pretty sure that when it's said that the edge of the universe is moving away from us at some speed greater than c, they aren't talking about proper velocity. The universe is expanding faster than SR on flat space-time would allow. Rckrone (talk) 18:42, 22 October 2009 (UTC)[reply]
It's not exactly the same as coordinate and proper velocity, but the difference between superluminal and subluminal motion in cosmology really is just a matter of definition. The Milne universe is the zero-density limit of big bang cosmology. In Minkowski coordinates the Milne universe is finite in size, spatially flat, and the edges are expanding at the speed of light, but in cosmological coordinates it's infinite in size, spatially hyperbolic, and recessional speeds are given by Hubble's law. (Also, in Minkowski coordinates the redshift is given by the SR redshift formula, while in cosmological coordinates the same redshift is given by the cosmological redshift formula.) When the matter density isn't zero you can't embed the universe in Minkowski space any more because spacetime isn't flat any more, but you can still take any finite spherical portion of the universe and put a Schwarzschild vacuum around it with the Schwarzschild m equal to the total enclosed mass. The edges stay inside the Schwarzschild light cones even if they have speeds higher than c in cosmological terms. -- BenRG (talk) 20:44, 24 October 2009 (UTC)[reply]
Then, of course, there's the possibility that c is actually slower than it was immediately after the Big Bang. Given 15 billion years, and the further possiblity that the rate of deceleration is itself also declining, the speed limit may have been considerably higher way back when. Perhaps a century from now, a light-year may be a proton's width shorter. B00P (talk) 07:00, 22 October 2009 (UTC)[reply]

Blood pressures & winter jackets

This question appears to be a request for medical advice. It is against our guidelines to provide medical advice. You might like to clarify your question. Thank you.

Responses containing prescriptive information or medical advice should be removed and an explanatory note posted on the discussion page. If you feel a response has been removed in error, please discuss it before restoring it.

We probably should not answer that here. If we told you one thing and it was wrong, that could affect the effectiveness of your work. Ask the people you work with or a doctor. Falconusp t c 11:29, 21 October 2009 (UTC)[reply]
Hmm, I don't see how it's medical advice, but I managed to find two studies [5] [6] that examine the difference between bare arm and clothed arm blood pressures. My question was more about how exactly the sphygmometer measures the pressure its exerting and whether or not having clothing in the way would mean some of the pressure is being dispersed. -- MacAddct1984 (talk • contribs) 16:18, 21 October 2009 (UTC)[reply]
It is clearly medical advice because it is seeking advice about carrying out a medical procedure, the failure or success of which could affect a patient's condition or treatment. Further, what the heck an Emergency Medical Technician is doing asking for technical advice on the RD is beyond me. OK, I do understand s/he may not be what they appear to be. But any intelligent responder should realise that medical technicians have recourse to professional advice within their work ambit. Richard Avery (talk) 17:34, 21 October 2009 (UTC)[reply]
It's not a medical advice question in the usual sense, which is where someone asks about what they should do for their own health (for example it basically satisfies kainaw's criterion). There is still a liability issue here and it's the same sort of concern that motivates the rule about medical advice questions. I don't think this is the right place for the question to be answered. Rckrone (talk) 18:20, 21 October 2009 (UTC)[reply]

WWII era German HEAVY tank

Hello. One of my friends mentioned a heavy (truly heavy) tank designed by some German scientist in the late WWII. The tank was designed to be inpenetrable, and could withstand huge amounts of enemy fire because of it's thick armor. My friend also said that the tank was so heavy, it would've had to use submarine engines as it's propulsion device, and it could've withstood even artillery shells or bomb hits. The production never begun because of the tank's heavy weight - it would sink slowly in the ground (or quickly if it was muddy) and become immobile very soon. The entire story sound a lot like an urban legend or a alternative history description to me, but I just want to be sure - was there such a tank or plans to build one? 88.112.62.154 (talk) 07:11, 21 October 2009 (UTC)[reply]

What you describe is pretty much true. See Landkreuzer P. 1500 Monster and the smaller Landkreuzer P. 1000 Ratte. For more crazy tank ideas, some of which were actually built, see Super-heavy tank. Someguy1221 (talk) 07:15, 21 October 2009 (UTC)[reply]
Thanks a lot! It was that Ratte tank. He did mention the name too, I just forgot it. Thank you so much! 88.112.62.154 (talk) 07:20, 21 October 2009 (UTC)[reply]
While not WWII, there's always Killdozer -- MacAddct1984 (talk • contribs) 17:15, 21 October 2009 (UTC)[reply]
As tanks go, that's not very impressive. A foot of concrete gives about the same protection as an inch of armor steel, so at a guess, the Killdozer has about as much armor as a late-model Panzer IV. --Carnildo (talk) 23:53, 21 October 2009 (UTC)[reply]

Coiled rope

I had a homework question that went as follows: a uniform rope of linear mass density λ is coiled on a smooth horizontal table. One end is pulled up with constant speed v. Find the force exerted on the end of the rope as a function of the height y.

At first I decided to look at the total energy of the system, which I found to be (λyv^2)/2 + (λgy^2)/2. Then I said that F=dE/dt, getting (λv^2)/2 + λgy as my answer.

But then I took a different approach, instead looking at the net force acting on the rope: ΣF = dp/dt, F - Mg = (dm/dt)*v, F - λyg = λv*v, F = λv^2 + λgy.

Each answer seems plausible, but which is right?

Neither is right. The speed is constant, so what it actually is is irrelevant. You just need the weight of the rope that isn't on the table to equal the upward force in order to have zero resultant force and thus maintain constant velocity. --Tango (talk) 10:52, 21 October 2009 (UTC)[reply]
I disagree with Tango, a segment of the rope is accelerating all the time, from being static on the table to being in upward motion so v is needed and the second answer is correct. In the first I assume you mean Fv=dE/dt in the above. Anyway E=(λyv^2) + (λgy^2)/2 not (λyv^2)/2 + (λgy^2)/2. You have a mass of λy moving at v m/s so the second /2 is in error. --BozMo talk 10:56, 21 October 2009 (UTC)[reply]
I agree with your disagreement, and actually I had meant F=dE/dy...but for the total energy, we both agree on the potential energy, so that's not an issue; but I don't see why there wouldn't be a /2 for the kinetic component. Isn't K=(1/2)mv^2=(1/2)(λyv^2)? —Preceding unsigned comment added by 24.202.172.103 (talk) 11:03, 21 October 2009 (UTC)[reply]
Ok I was reading the ^2 as a half in the formating I am not used to keyboard notations. But where does F=dE/dy come from? That's not physics. Force times velocity is energy expended rate of change of force with height is garbage. Which I think answers your question, the second is right. --BozMo talk 11:06, 21 October 2009 (UTC)[reply]
W = ΔE, for small displacements dW = dE, dW = Fdy, leading to F=dE/dy, right? —Preceding unsigned comment added by 24.202.172.103 (talk) 11:16, 21 October 2009 (UTC)[reply]
Ok. In fact Fv=dE/dt gives the same answer so I will have to look at the second version more closely. Trying to do too much at the same time--BozMo talk 11:20, 21 October 2009 (UTC)[reply]
SO to be clear on notation in (2) you write net force is rate of change of momentum with time, which is λv of rope accelerated to v. ?--BozMo talk 11:27, 21 October 2009 (UTC)[reply]
Had to use a bit of paper for this one. I think the energy version works and the net force one does not. The reason is that there has to be some tension in the rope at the point of contact. With gravity you do not notice, it just pushes slightly less hard down on the table. In zero gravity free space I think the whole coil would start to move towards you.--BozMo talk 11:45, 21 October 2009 (UTC)[reply]
I don't think so. I solved the problem a totally different way, and I got the second answer, F=λv2 + λyg. The way I did it was to use the equation Fnet=macm, where Fnet is the net force on the rope, Fnet=F-λyg, m is the total mass of the rope, including the part of the rope on the table, and acm is the acceleration of the center of mass of the entire rope. The center of mass of the entire rope is at (y/2)(λy/m) = λy2/(2m), where again m is the total mass of the rope, so acm=λv2/m. Red Act (talk) 12:36, 21 October 2009 (UTC)[reply]
Hmm. But that uses the net force again, in effect? In your terms the force upwards on the rope by the table is actually unknown. You do not know how much rope is off the table but not travelling upwards at V and it may matter. I think this curved segment of rope which is being accelerated is significant (as in non zero) even for a ultra thin rope. --BozMo talk 12:48, 21 October 2009 (UTC)[reply]
Agree with Red Act. Second answer, F = λv^2 + λgy, is correct. You can't use conservation of energy because energy is "lost" to thermal energy when each part of rope is "instantaneously" accelerated from rest to speed v - this is like an inelastic collision in reverse. The total work done in lifting the rope can be found by integrating F dy from y=0 to y=L, the length of the rope - you get λLv^2 + λgL^2/2. Of this, λgL^2/2 is potential energy of the rope, λLv^2/2 is kinetic energy of the rope, and the remaining λLv^2/2 is lost to thermal energy. Gandalf61 (talk) 12:53, 21 October 2009 (UTC)[reply]
Surely not? You can get ropes which are pretty elastic and in a real physical system there is no need for the acceleration to be instantaneous, the rope would curve upward at the elbow. The upward curve would reduce the amount of rope on the table hence the missing force. --BozMo talk 13:02, 21 October 2009 (UTC)[reply]
But this is an ideal problem with an ideal inextensible rope. In real life, yes, the rope would stretch and the acceleration would not be instantaneous - but then you would have to account for the energy stored as elastic energy in the rope as it extends under its own weight. Gandalf61 (talk) 13:16, 21 October 2009 (UTC)[reply]
Forgive me but I still think this is wrong. Even for an inextensible rope the proper net force solution requires introducing tension as a function of height. As long as the rope has a non-zero mass per unit length this is needed. You could solve the equation for the curve shape of the rope assuming it was completely flexible and its instantaneous acceleration around the curve was given by the tension and curvature. You would find that a significant part of the rope was not in contact with the table which would be the missing force (do you remember deriving Catenary curves that way at school)?. You could force the elbow to be on the table e.g. by fitting a pulley but then that would exert net force up on the table. I don't think anything here introduces an energy loss to the first order in flexibility, etc. The curvature of the rope at contact with the table scales like v^2/g which is a length. --BozMo talk 13:26, 21 October 2009 (UTC)[reply]


And I still think that energy is lost and you are wrong - but rather than go back and forth on this, let's wait to see what other folks think. Gandalf61 (talk) 13:44, 21 October 2009 (UTC)[reply]
I agree with RedAct. Regardless of tension or elasticity or internal energy or anything else, the net force on the rope has to equal the change in momentum per time. The force applied to the end of the rope has to counter gravity on the dangling end of the rope, which is λgy (technically the upward force is transmitted by the normal force at the corner of the table). Then as the rope rounds the bend, the vertical change in momentum is caused by gravity (the force on the end of the rope is in the wrong direction to have an effect) and the horizontal change in momentum which is λv2 must be supplied by the force on the end of the rope. Rckrone (talk) 16:54, 21 October 2009 (UTC)[reply]
So how did you calculate y for the length of a non-straight bit of rope with a v^2/g curve at one end? If the rope is thin and inelastic then it will not descend to a right angle bend. Nor would a small link chain for example. If it has some marvellous non Newtonian properties (e.g. a very rusty chain) it might descend to a right angle bend and then basically Gandalf61 would be correct, it could lose energy in a corner. However in general the acceleration upward will be along a curve of radius order of v^2/g giving rise to an additional weight of rope off the floor which has to be supported by the force. This means if you do the net force calculation properly you have a smaller acceleration than expected and you end up with the energy version of the equation. --BozMo talk 17:21, 21 October 2009 (UTC)[reply]
The spirit of the problem is that the rope is coiled very tightly (think of a very dense rope), such that any curvature at the end can be neglected. Or even with a less dense rope, you can always specify a large enough value of v that the rope at the end can't possibly have as large of a radius of curvature as you're requiring of it. If the diameter of the rope coil isn't considered negligible, then there's energy going into making the rope oscillate, which ultimately turns into heat as the oscillation gets damped. Red Act (talk) 20:27, 21 October 2009 (UTC)[reply]
The curvature is not to do with the rope thickness but to do with the challenge of accelerating a finite mass in zero distance. Anyway, the OP gave too different calculations which were apparently inconsistent. A system which conserves energy resolves the inconsistency by a pick up on the rope, the rope joins the ground at a curve of about v^2/g. A system which maintains no pick up has to introduce an energy loss associated with flexing the rope or an extra downwards force to stop a pick up. Either way it breaks energy conservation or adds more force. It seems to me that we have no inconsistency (but actually I do not believe there is a force free way of suppressing the pick up. --BozMo talk 20:51, 21 October 2009 (UTC)[reply]

I find it dubious that energy loss is the solution. For simplicity, let's say we aren't in a gravitational field, and that instead of pulling on a macroscopic rope, we are pulling on an infitessimal rope (a point particle in effect). What is the force necessary to accelerate this point particle to a speed v? Again, using either forces or work yields each yields different but plausible solutions, but with a point particle, how is it that energy can be lost? —Preceding unsigned comment added by 24.202.172.103 (talk) 21:00, 21 October 2009 (UTC)[reply]

Does this not resolve in exactly the same way when you introduce the fact that the force has to be applied for a time and a distance. It is only if you think you can apply the force without the particle moving at all that you get a different answer. Just as for zero curvature above--BozMo talk 21:13, 21 October 2009 (UTC)[reply]
The key difference between a rope and a point particle is that a point particle is rigid. Treating the rope as a point particle would be as if the rope was superglued into a rigid shape, so that the mass being accelerated is always the entire mass of the rope. Eliminating the variable-mass aspect of this problem would turn this into a completely different problem. Red Act (talk) 22:32, 21 October 2009 (UTC)[reply]

The fundamental flaw with the first "solution" is that it relies on the work-energy theorem. But the work-energy theorem only applies to a rigid object. "If the object is not rigid and any of the forces acting on it deforms the object, then the Work-Energy Theorem will no longer be valid. Some of the energy transferred to the object has gone into deforming the object and is no longer available to increase or decrease the object's Kinetic Energy."[7] Turning a coiled rope into a straight rope is deforming the rope, and some energy is generally required to do that. The only way to straighten out the rope without expending any energy in the process would be to move the rope at an infinitesimal speed. Straightening out the rope in a finite amount of time requires a finite amount of energy. So the work applied to the rope in this problem must be greater than the kinetic energy plus potential energy that the rope gains. Additional work must be applied in order to supply the energy required to straighten out the rope in a finite amount of time. Red Act (talk) 23:14, 21 October 2009 (UTC)[reply]

To better see where the problem lies, change the experiment to eliminate gravity, and eliminate kinetic energy at the end. Supposed you're floating in space, with a coil of rope at rest relative to your spacecraft. Your goal is to straighten out the coil of rope in a finite amount of time, such that at the end, the rope is still at rest relative to your spacecraft. No matter how carefully you plan how you nudge parts of the rope, you're going to need to apply some energy to straighten out the rope. The minimum amount of energy required to straighten out the rope is dictated by how little time you have to complete the task, and the initial shape of the rope. But it's a nonzero amount of energy, and it's energy that the first "solution" above neglects to consider. Red Act (talk) 23:34, 21 October 2009 (UTC)[reply]

Cool, thanks.

Just a little caution. There are a number of ways of idealising physical situations in order to model them. Using deformation energy to consider a "chain" (a flexible rope with linear density) is not a standard one and not one a maths examiner would typically recognise. Generally the work energy theorem is considered to apply to flexible chains, which are not considered to require energy loss in deformation. --BozMo talk 07:14, 22 October 2009 (UTC)[reply]
To sum up: in mathematical modelling we can only take something as effectively zero if it can be made arbitrarily small as the control parameters (rope thickness, deformation viscosity etc) become arbitrarily small. In this problem there is no choice of control parameters which make the curvature as the rope reaches the ground arbitrarily small so you have to consider the curved part of the rope. For a given geometry the energy loss flexing a real rope is linearly proportional to several physical constants in the rope and so this does tend to zero as the control parameters tend to zero (provided we are confident that the limit is well behaved and the geometry doesn't change, unlike for turbulent flow as viscosity tends to zero say). Therefore, in the standard treatment we would take a completely flexible rope with no energy loss but consider the fact the the rope accelerated over a finite corner. The finite corner gives more hanging rope for the force to support, so the net force argument does not work. There is no energy loss so the energy argument does. --BozMo talk 08:35, 22 October 2009 (UTC)[reply]
I found a problem very similar to this one in this textbook. The only difference between that problem and this one is that in that problem, the top of the rope is being pulled upward by being attached to a mass that's been given some initial velocity, instead of the top of the rope being pulled up at a constant velocity as in this problem. But both problems involved a coil of rope, whose mass is not taken to be negligible, being pulled upward. Notice several things about the problem in the textbook:
1) The problem has the student start off with F=dp/dt. That is, the textbook points the student in the direction of the second solution above, the solution that I believe is the correct one.
2) The problem states that "This is an example of a variable mass system for which the principle of conservation of energy does not hold...", and also points out that the work-energy principle does not hold here because this is a variable mass problem. That is, the textbook leads the student away from the first solution above, the one that you believe is the correct one.
3) The problem assumes that you can solve the problem with the information given, without mentioning anything about having to mess around with the curvature of the rope. Indeed, insufficient information is given in the problem to figure out what the curvature of the rope would be, information like what the diameter of the coil of rope is. The tacit assumption is that the portion of the rope that's off the table is simply straight. Red Act (talk) 11:13, 22 October 2009 (UTC)[reply]
It is clear in the "flexible chain" formulation that the shape of the rope becomes independent of the thickness of the rope as the thickness of the rope tends to zero, so that is not a problem. You also do not need to know the exact shape, its existence just explains why the net force approach gives a different answer, and why there is a well posed problem which conserves energy. I agree with you that if the portion of the rope off the table is simply straight your answer is the correct one. However it is rather unclear to me that any physical circumstances would approximate to that (they might, I just cannot think of any), whereas treating the problem as a flexible chain does correspond to a real physical limit. If there is any real physical limit of parameters capable of yielding a right angle in the rope (as some internal property tends to zero) I would be interested. --BozMo talk 13:02, 22 October 2009 (UTC)[reply]
BozMo - I don't understand why you continue to insist that work done on rope = kinetic energy of rope + potential energy of rope when it clearly isn't. Applying Newton's second law (and ignoring gravity and p.e., since we all agree on the λgy term) then F dt = dp = λv dy, so F dy = Fv dt = λv2 dy, and so work done on rope = λv2y, whereas k.e. of rope is only λv2y/2. If you insist that there is no "lost" energy then you have to explain why Newton's second law does not apply to the rope. Gandalf61 (talk) 13:51, 22 October 2009 (UTC)[reply]
Hmm. I have done too many problems treating chains as point weights and rigid connectors I guess. There just isn't a friction to absorb any heat (of course is it idealised but the limit is a good one in mathematical terms, for a given set up you can halve the friction by changing materials and that cannot change the geometry). If you take the obviously geometry with the chain ascending upwards above the coil centre and the coil feeding round in a circle the rising chain gets horizontal rotational kinetic energy and angular momentum. That might disipate into your heat in time but there is absolutely no way it is plastic deformation at a corner. --BozMo talk 14:25, 22 October 2009 (UTC)[reply]
The alternative way to see this geometrically is to lay the end of the rope flat along the ground and pick it up at v m/s but this time you have to run sideways at v m/s. Your upwards force is still F dt = dp = λv dy, so F dy = Fv dt = λv2 dy, and so work done on rope = λv2y, but now the k.e. of rope is λv2y. Are you really claiming that I am missing an energy loss in the corner?
One more note. If your rope stays exactly above the point in the coil where it instantaneously is unwinding then the whole rope is travelling around in a circle of radius r (which cancels) and at a velocity v. Therefore there is λv2y/2 in vertical kinetic energy and λv2y/2 in horizontal kinetic energy. --BozMo talk 14:42, 22 October 2009 (UTC)[reply]
Hmmm ? Energy is energy - it is not vertical or horizontal. But if you impart a horizontal component of velocity to the rope then you have to change both force/momentum and energy equations, not just one of them, so there is still lost energy. I think you are tying yourself in even more knots than the rope - I am done with this nonsense. Gandalf61 (talk) 14:57, 22 October 2009 (UTC)[reply]

Anyway to recap when this has got to. Flexible chains are energy conserving so there is no internal loss to plastic deformation and the like. However in this case some half of the energy has to turn into horizontal kinetic energy as the initial movement of the rope has to be not only v upwards but also v along the coil so the rope can continue unwinding without snapping. The upward force is F dt = dp = λv dy, so F dy = Fv dt = λv2 dy, and so work done on rope = λv2y, and the k.e. of rope is λv2y . Further up in the air this additional horizontal energy (and angular momentum) presumably disipates. I happily concede I was wrong about which equation gave the right answer. However it is quite clear that everyone else was wrong about what happened to the energy. It cannot be dissipated in the corner as it still has λv2y energy just above the corner. --BozMo talk 15:02, 22 October 2009 (UTC)[reply]

Who said that all of the energy dissipation was necessarily right at the corner? What I said yesterday was "If the diameter of the rope coil isn't considered negligible, then there's energy going into making the rope oscillate, which ultimately turns into heat as the oscillation gets damped." In any realistic rope, that circular oscillation with the extra λyv2/2 of kinetic energy will get damped, and that energy will be converted into heat. In the limit as r->0, the initial oscillations will have a smaller amplitude but a higher frequency, which I think would result in the oscillations getting damped into heat faster. In fact, if you get down to a small enough value of r, the oscillations would even have a high enough frequency that they would already count as heat initially. Of course, it's impossible for a real rope coil to have a radius anywhere near that small, so it does take some time for the oscillations to get damped into oscillations that have a frequency high enough to count as heat. At any rate, that extra energy does eventually wind up as heat, so an equivalent way of saying why solution #1 is incorrect is to say that it fails to take into consideration the heat that's produced as a result of deforming the rope. Saying otherwise would require a rope that can sustain energy in the form of a macroscopic oscillation forever, without it getting turned into heat. Such a rope does not exist. Red Act (talk) 15:43, 22 October 2009 (UTC)[reply]
"Who said that all of the energy dissipation was necessarily right at the corner?" Who? see above "energy is "lost" to thermal energy when each part of rope is "instantaneously" accelerated from rest to speed v - this is like an inelastic collision in reverse" was not your comment I agree. Then you were right all along I just misunderstood you, also when you said in your illustration "No matter how carefully you plan how you nudge parts of the rope, you're going to need to apply some energy to straighten out the rope." I thought you were implying the energy loss was somehow in straightening the rope. All clear now, you did not mean that. --BozMo talk 18:33, 22 October 2009 (UTC)[reply]
Yes, I said that. I assumed an ideal inextensible rope, and with that assumption my statement is correct - each part of the rope is accelerated instantaneously and energy is lost to thermal energy instantaneously. You objected that the inextensible rope is not a reasonable model of the real world. Red Act points out that even with a deformable rope, where energy goes initially into transverse or longitudinal oscillations of the rope, this energy is eventually lost to thermal energy anyway. In either case the assumption that work done on rope = kinetic energy of rope + potential energy of rope is incorrect. Gandalf61 (talk) 09:53, 23 October 2009 (UTC)[reply]
No, you are still in error about this. With an ideal inextensible rope as each element unfurls it has a vertical velocity of v and a horizontal velocity of v, to match the horizontal speed at which the pick up point has to travel sideways. An ideal inextensible rope is incapable of spontaneously losing energy. --BozMo talk 10:47, 23 October 2009 (UTC)[reply]
"An ideal inextensible rope is incapable of spontaneously losing energy" - oh really ? Attach an ideal rigid mass to one end of an ideal inextensible rope; attach the other end of the rope to an ideal perefctly rigid support; release the mass from a point vertically below the other attached end of the rope. When the mass comes to the end of the rope it stops dead - instantaneously - because the rope cannot extend. Where has the kinetic energy of the mass gone ? Gandalf61 (talk) 11:47, 23 October 2009 (UTC)[reply]
Sorry am I missing something? With an ideal inextensible rope the mass would bounce upwards, same as with any rigid elastic rod. --BozMo talk 12:12, 23 October 2009 (UTC)[reply]

I agree with Red Act where he has written that

I arrived at this conclusion by considering the force in two components. Firstly there is the component of force necessary to support the weight of the rope. That is:

Secondly there is the component of force necessary to supply the steady increase in momentum:






In this question, mechanical energy is not conserved. Half the power provided by the rising force goes into kinetic energy of the rope, and half is lost to irreversibilities and ends up as heat. Dolphin51 (talk) 10:50, 23 October 2009 (UTC)[reply]

Yes, as above Red Act (and you) are correct about this. The issue is how the half is lost to irreversibles and the claim that this could be limited to a tiny area around the kink for a (near) perfect chain. It could not. Initially the rope picks up at the point with v up and v sideways and the v sideways gets lost in galloping some time later. --BozMo talk 10:57, 23 October 2009 (UTC)[reply]

I am now having second thoughts about this. Perhaps half the power goes into kinetic energy, and half goes into gravitational potential energy. I need to think about it. (I agree that the idea that half goes to irreversibilities is not intuitively convincing.) Dolphin51 (talk) 11:02, 23 October 2009 (UTC)[reply]
Half instantly into irreversibles is obvious nonsense as a perfect chain can be modelled by stiff light rods and elastic weights. That was my starting point but to avoid you making the same mistake which I made above, draw a little picture of the rope at lift off. You can see immediately as the lift off point has to move sideways at v so does the rope, otherwise it would snap. I have seen a video of a helicopter picking up steel cabling (to lift a tree trunk at the far end of the cabling) and the galloping is massive. As you I thought this must to take the extra energy into PE but actually it is to absorb the sideways kinetic energy. --BozMo talk 11:10, 23 October 2009 (UTC)[reply]
Y'all are forgetting that the lost energy doesn't need to be instantly transformed in heat. For an ideal rone no heat is produced but the extra energy can be radiated away as waves on the rope that (for an inextensible rope) will propagate at infinte speed and (apparently) disapear.
PS: Please BozMo, stop talking about sideways energy. It's getting on my nerves. Dauto (talk) 14:52, 23 October 2009 (UTC)[reply]
Agree completely that it doesn't need to be, and certainly isn't instantly transformed in heat... that is the point. Sorry you do not like the phrase "sideways energy" but as kE is quadratic, the two directions are orthogonal and Pythagorus was right it does seem to me to be a shorthand with a unique and obvious meaning... FWIW the whole problem for a flexible elastic chain is far from solved here (and does not need to be). There is still the question of whether the rope induces tension in the coil or not and what the pick up curve actually is. I would need a long car journey to do those. --BozMo talk 14:59, 23 October 2009 (UTC)[reply]

Acid attack neutralization spray?

In certain backward cultures, acid attacks are a problem for women. Could a spray can or face cream be devised to neutralize the acid immediately after contact and limit tissue damage without being corrosive or toxic itself?80.1.88.12 (talk) 10:50, 21 October 2009 (UTC)[reply]

Certainly something which was a form of Buffer solution would help.--BozMo talk 11:04, 21 October 2009 (UTC)[reply]
Bicarbonate, since it can either gain or loose a proton at physiological pH (bicarbonate is actually the buffering agent in blood), is frequently used for neutralizing both acids and bases. Sodium Bicarbonate is readily available to most people as baking soda, and can be used either as a solution in water or directly as the powdered solid. -- 128.104.112.179 (talk) 14:41, 21 October 2009 (UTC)[reply]
I think the real difficulty here is in being able to apply the neutralizer within a useful amount of time. I suspect a technical solution is not the answer to this particular problem. --Mr.98 (talk) 15:54, 21 October 2009 (UTC)[reply]
Note also that neutralization of an acid typically involves an (often violent) exothermic reaction - i.e. heat. Simply washing the acid off with large amounts of water is almost certainly preferable. Using a thick, inert cream layer before a suspected attack may protect somewhat, I'd think. Does e.g. Vaseline react with commonly used acids? --Stephan Schulz (talk) 16:12, 21 October 2009 (UTC)[reply]
It's a tricky problem because even washing it off with water will initially spread the acid over a larger area before it becomes dilute enough not to be harmful. As Stephan says - you need large amounts of water - like a bucketful. SteveBaker (talk) 17:56, 21 October 2009 (UTC)[reply]
And I imagine it also requires some knowledge of what the scarring substance in question is. If it is lye, you don't want to apply water to it. --Mr.98 (talk) 17:30, 22 October 2009 (UTC)[reply]
Finding something to neutralize the acid isn't hard -- as noted, baking soda will do quite nicely -- the problem is that it needs to be applied immediately, within a few seconds of the attack. That's why anyone working with acids will have an emergency shower in the same room. --Carnildo (talk) 23:57, 21 October 2009 (UTC)[reply]

Lotions and aluminum starch

I've noticed many hand and body lotions contain "aluminum starch" (usually within the top 5-10 ingredients). What is it and why is it added to lotions? I see the word aluminum and I'm a little worried about rubbing into my skin every day. --68.103.141.28 (talk) 13:44, 21 October 2009 (UTC)[reply]

It's a binding and anti-caking agent (full name Aluminum Starch Octenylsuccinate and is a modified starch). Here is a safet report from PubMed. It is also approved for use in food. SpinningSpark 16:02, 21 October 2009 (UTC)[reply]

Experiments

What are the five steps for doing an experiment?--Mikespedia (talk) 15:56, 21 October 2009 (UTC)[reply]

I suspect whatever it is you are looking for (homework, whatever) is covered in scientific method, even though the idea of there being a defined five steps that are actually used by practicing scientists is kind of absurd. Actually, the definition at Simple Wikipedia is probably closer to what you want than the long philosophical/historical exegesis at the regular English article. --Mr.98 (talk) 15:58, 21 October 2009 (UTC)[reply]
1) Do your own homework
2) Read the textbook provided by your science teacher
3) Pay attention during class when your science teacher lists the 5 steps
4) Write what your teacher says down in a notebook so you can refer to it later
5) Read our article on scientific method, which contains several different ways of conducting experiments. There is no one single "5 step" method, but rather many different ways of thinking about the overall set up of an experiment. That's why # 1-4 are important; it is likely that your teacher has a specific 5 steps in mind, and there is absolutely no way that people who did not sit in your class will know which 5 steps your teacher taught you. --Jayron32 16:03, 21 October 2009 (UTC)[reply]
Quite right assuming you mean some variant on Aim/Apparatus/Method/Results/Conclusion --BozMo talk 16:05, 21 October 2009 (UTC)[reply]

Particles in Brand New Water Bottle

This MON I bought me a 32 oz. bottle of EVIAN water, left it in the freezer to chill while at work. Left work early and forgot about the water. TUES I take solid ice bottle out of freezer, and place at my work station to thaw. I come in this WED morning to see a sealed bottle filled with white flakey floaties. Looks like dandruff. After the initial gross out period, I concluded that the freezing of the water expanded the bottle a bit, and this expansion stretched the thin plastic, and after the thaw, a think layer of weak plastic (skin like) broke apart inside to make flakes--A sort of Evian Snow Globe. Do you concur? If not, explain. Thanks. --i am the kwisatz haderach (talk) 16:02, 21 October 2009 (UTC)[reply]

Was the bottle still factory-sealed on WED ? Most water bottles have a break-to-open ring under the screw top. Cuddlyable3 (talk) 22:08, 21 October 2009 (UTC)[reply]
Yes, Factory Seal still unbroken. I'm putting back in the freezer, see if I can incubate more in a Freeze/Thaw/Repeat sequence. --i am the kwisatz haderach (talk) 22:38, 21 October 2009 (UTC)[reply]
Most plastic bottles I'm familiar with use wax or a wax-like substance to improve the seal between the cap and the bottle. It could be that part of this flaked off when you froze the bottle. --Carnildo (talk) 00:00, 22 October 2009 (UTC)[reply]
From the ice cube article: "Melting ice cubes sometimes precipitate white flakes, commonly known as 'floaties'. This is calcium carbonate which is present in many water supplies and is completely harmless." Red Act (talk) 00:38, 22 October 2009 (UTC)[reply]
Red Act got it right. Same thing happened to me. I called the number on the bottle to ask about the white flakes, and got basically the same answer. Other than the "yuk factor", you have nothing to worry about (add usual disclaimers here). Bunthorne (talk) 19:55, 26 October 2009 (UTC)[reply]

Oil-based scum in me coffee

My last question made me think of STARBUCKS or COFFEE BEAN, just regular BLACK COFFEE from any/random cafes. I don't use milk or sugar, and like all Americans, I get my coffee in their togo cups w/plastic lids. Sometimes if I sit down and relax in the cafe, I'll take the lid of, and blow on the hot coffee, take sips, the usual. I've noticed that there's this thin layer of oily shine on the surface water of the coffee. What could this be? I'm thinking some sort of chemical cleanser used by the cup/lid manufacturing company. That's just speculation. What do we think? Cheers, --i am the kwisatz haderach (talk) 16:08, 21 October 2009 (UTC)[reply]

It's most likely the oil from the coffee itself. That oil is a large part of the flavor/aroma of the beverage. --Sean 16:13, 21 October 2009 (UTC)[reply]
Or something from your saliva perhaps? Googlemeister (talk) 17:44, 21 October 2009 (UTC)[reply]
If the water is hard (contains a lot of calcium ions), both coffee and tea will form thin layers of something that looks like oil, but is some scum formed from calcium, coffein, and some other substances. It's harmless, but ugly, and the reaction spoils the taste of tea and, to a lesser degree, coffee. --Stephan Schulz (talk) 19:20, 21 October 2009 (UTC)[reply]
o/~I had a dream there were clouds in my coffee...o/~ Tevildo (talk) 00:11, 22 October 2009 (UTC)[reply]

How does resistance change on increasing the tempreture of the conductor?

From the ohmslaw i hit a doubt. Law states: the electric current flowing through a mettalic wire is directly proporstional to the potential difference 'v' across its ends provided its tempreture remains the same. Sescond part of the the law states that temp must be same, but if tempreture is changed conductor's resitance will increase or decrease. frfom the formulae R is directly proportional to the Heat so Heat is directly proportional to R this gives that resistance should increase on heating but after heating there will be more free ions to conduct electricity, so resistance should decrease. —Preceding unsigned comment added by Myownid420 (talkcontribs)

  • Read Electrical_resistance#Temperature_dependence. Toodles. --Jayron32 16:16, 21 October 2009 (UTC)[reply]
  • Current in a metal is not carried by ions, but by electrons. What makes a metal a metal is the fact that the crystal has an energy band that is partially empty and partially full. That means that the electrons in that band can essentially move freely through the metal. Thus metals are good conductors. As the temperature increases, atoms in the lattice increase their vibration and hence interact more often with electrons, slowing them down. This increases resistance. Semi-conductors, on the other hand, have completely filled or empty energy bands, but have only a small band gap between the highest full and lowest empty band. Increased temperatures allow electrons to jump that gap and move into the conducting band. Thus, for semi-conductors resistance drops with temperature. --Stephan Schulz (talk) 16:27, 21 October 2009 (UTC)[reply]
The OP first stated Ohm's law correctly. However there is no "Sescond[sic] part of" Ohm's law concerning temperature. Resistance, the 'R' in Ohm's law, being proportional to temperature is a property of the conductor material. The OP should not confuse Heat with Temperature. Heat is the energy that must be moved in order to raise or lower the temperature of any thing. Cuddlyable3 (talk) 21:57, 21 October 2009 (UTC)[reply]

why isn't sodium chloride that hygroscopic

What makes magnesium sulfate, calcium chloride and lithium chloride more hygroscopic than sodium chloride? I'm confused, sodium chloride appears to be in the middle of them (w/regard to periodic trend)... John Riemann Soong (talk) 17:23, 21 October 2009 (UTC)[reply]

It's a complicated property that has little to with the sort of thing that can be explained easily via periodic trends. Hygroscopicity (is that a word? it is now...) is likely related to the binding energy between the ions in question and water molecules. My guess is that the +2 ions will create stronger bonds with water than will the +1 sodium; while the smaller Lithium +1 ion will likewise bind tighter than the larger Sodium +1 ion; but that is just an educated guess. There are a LOT of factors to consider, solvation is an exceedingly complex process, and so while that is one factor to consider, it likely is not the only one. The solvation article contains a rough overview, and shows exactly how many things need to be considered when looking at a problem like this. --Jayron32 02:55, 22 October 2009 (UTC)[reply]

CNS & PNS cell types.

I was wondering if someone would be kind enough to assemble a list of the main cell types found in the Central and Peripheral Nervous Systems, and what it is that each type does? Many thanks, Colds7ream (talk) 17:59, 21 October 2009 (UTC)[reply]

Sensory afferent --> pseudounipolar, motor efferent --> multipolar. ANS --> both sympathetic + parasympathetic would be multipolar. Bipolar exists only in the retina eyes and a few other places. DRosenbach (Talk | Contribs) 18:15, 21 October 2009 (UTC)[reply]
Ahh...forgot about ancillary cells. Glial in the CNS, consisting of various types of astrocytes. Then there would be Schwann cells in the PNS and oligodendrocytes in the CNS. DRosenbach (Talk | Contribs) 18:16, 21 October 2009 (UTC)[reply]
Thanks very much! :-) Colds7ream (talk) 21:48, 21 October 2009 (UTC)[reply]
I was sort of in a hurry when responding earlier today -- so I just threw together a bunch of info without properly organizing it. But if you check the links I included above, as well as some of the other, unlinked-to "buzzwords" of neurophysiology, you should be able to get all that you need from Wikipedia. If you have any other specific points, though, you can restate them. DRosenbach (Talk | Contribs) 02:47, 22 October 2009 (UTC)[reply]
It is easily subject to injury because the ball of the upper arm is larger than the shoulder socket that holds it.

What does this mean? How could the ball be larger than the socket and still be contained inside? DRosenbach (Talk | Contribs) 19:04, 21 October 2009 (UTC)[reply]

The implication is that the socket embraces less than half of the ball, in the same way (though not to the same degree) that a golf ball is larger than the cup of the tee supporting it. 87.81.230.195 (talk) 19:47, 21 October 2009 (UTC)[reply]


The hip joint socket is more than a hemisphere
An idealised hemispherical socket
The shoulder joint is much less than a hemisphere
The pictures (right) explain this better than words can. The ball is spherical - the inside of the socket has the same curvature as the ball - they both have about the same radius - so the ball fits perfectly into the socket - but unlike normal industrial/mechanical ball-and-socket joints - and unlike the hip joint, the shoulder socket is less than a hemisphere. If there were no muscles, tendons and whatnot holding it in place, it would just fall off. SteveBaker (talk) 00:24, 22 October 2009 (UTC)[reply]
Wonderful! Thanks Steve. DRosenbach (Talk | Contribs) 02:44, 22 October 2009 (UTC)[reply]
The analogy used in the anatomy classes I took is that the shoulder is like a teacup sitting in a saucer. Yes, it's a ball and socket joint, but like SteveBaker wrote, the socket is not a hemisphere. In fact the glenoid fossa of the scapula (where the ball of the humerus articulates) is surrounded by a lip of collagenous material called the glenoid labrum which increases the concavity of the socket. The muscles and tendons of the rotator cuff are critical in maintaining shoulder integrity, along with the tendons of the surrounding larger muscles. -- Flyguy649 talk 02:45, 22 October 2009 (UTC)[reply]


To maintain joint integrity and stability,the center of rotation of the ball in any ball & socket/cup joint must remain relatively static in translation(horizontal movements) with respect to the socket/cup. The large socket with ligaments at the hip maintains this better than the less stable shoulder joint. It is a bit like one of those compression and tension mechanical equilibrium setups.Sjschen (talk) 03:57, 22 October 2009 (UTC)[reply]

Sunrise and sunset on a perfect sphere

I was reading an article about the International Space Station, and it got me thinking about how astronauts perceive sunrise and sunset from various orbits, and how you might figure this out for a general case of an observer on a sphere. After musing over it for a while, it seems like there are quite a few things you'd have to know.

In the simplest case, we might have the following:

  • There is an observer .
  • is standing on the equator of a sphere with radius .
  • is a point source a distance away from .
  • is rotating in space about its axis and completes a full revolution every time units.
  • exhibits no precession in its rotation.
  • The axis of rotation of is perpendicular to the plane containing the center of , the center of , and .
  • does not rotate around .

Here the only variables seem to be , , and . If starts out on the point on closest to , is there an equation that describes when will experience sunrise and sunset?

Moving to a more realistic real-world case, you can see how there might be a number of additional different variables:

  • is some height above in geosynchronous orbit (as if O were standing on a mountain with height ).
  • is not on the equator of , but rather at an angle above the equator. (Is this equivalent to simply being on the equator of a sphere of size ?)
  • rotates around with a period of time units.
  • has an axis of rotation which is tilted at an angle from the vertical.

That's four more additional variables now: , , , and . You could get even more complex (for example, might not be in geosynchronous orbit and instead completes a rotation around every time units), but I think this covers a lot of useful cases.

Is there an equation which completely describes when O experiences sunrise and sunset in this case? —Preceding unsigned comment added by 216.30.180.104 (talk) 19:48, 21 October 2009 (UTC)[reply]

You'll likely find answers to your question by digging in two directions (though I doubt a single equation will answer each and every one of the issues you raise):
  • Astronomy: Astronomers have been preoccupied with this problem for centuries, though accurate solutions have been found only after the laws of motion and gravitation have been mastered. Check, for instance, the [Horizons] on-line system.
  • Remote sensing: Some Earth orbiting space platforms embark limb scanning instruments that aim to characterize the vertical distribution of chemical compounds in the atmosphere by measuring absorption processes during occultation. See, for instance, the article on Upper Atmosphere Research Satellite.

Michel M Verstraete (talk) 20:39, 21 October 2009 (UTC).[reply]

The generic term here is occultation, and it occurs whenever the nearer two bodies' positions are closer (by angular distance) than their angular diameter. In general, you can set up an equation to define the angular position of both bodies (as viewed from whatever stationary or moving point of reference you like), and then solve for all times when the angular distance is less than the angular diameter (for a partial occlusion). In your case, you seek the angular diameter of earth and sun, as well as their relative angular positions, while viewed from the moving frame of the ISS (or some other orbit). In the most general case, even the angular diameters of both objects are time-varying (as their relative distances might change). Nimur (talk) 20:53, 21 October 2009 (UTC)[reply]
(For a grazing occultation, angular distance is less than the sum of the two angular diameters). A total occultation is when the angular separation is less than the nearer body's angular diameter. Nimur (talk) 20:58, 21 October 2009 (UTC)[reply]
The geometry here isn't too bad. For someone on the Earth, the sine of the apparent angle of the sun above the horizon plane is
where ψ is the angle corresponding to the time of day, with ψ = 0 at noon. Note also that φ here is not strictly the tilt of the Earth's axis, but the tilt of the axis toward or away from the sun. For Earth this value varies over the course of the year between 23.5° in December and -23.5° in June (I arbitrarily chose φ to be positive in the direction of the north pole pointing away from the sun). To find the time of sunrise and sunset on Earth for a given θ and φ, you would find the values of ψ that satisfy s(ψ) = 0.
In orbit we can use the same formula, except now ψ is the angle through the satellite's orbit, with ψ=0 at the closest point to the sun, θ = 0 since the plane of the orbit has to pass through the center of the Earth, and φ is the tilt of the satellite's axis of revolution toward or away from the sun. So now .
Above the Earth, the sun is still visible when it passes below the plane of the horizon. The angle at which it passes out of view is at -cos-1(r/rorbit) so to find the angle ψ for sunrise and sunset solve for s(ψ) = -(1 - (r/rorbit)2)1/2. In either case, to get from an angle ψ to a time t, t = ψ*T/2π where T is the period. Rckrone (talk) 21:17, 21 October 2009 (UTC)[reply]
In the relatively low orbits of the ISS and Shuttle, the atmosphere plays a big part in what they see. Rayleigh scattering and Mie scattering dominate what they see - just as they do for us down here on earth.

Stop bleeding

Does anyone know the how small a blood vessel has to be before it can stop bleeding by itself if cut? For example, if I slice and tiny capillary or arteriole it will stop bleeding relatively quickly, however, the same cannot be said of something like, say the aorta. I can't seem to find any information on this... Sjschen (talk) 21:57, 21 October 2009 (UTC)[reply]

The aorta will eventually stop bleeding when pressure drops due to hypovolemia and overload of compensatory mechanisms. DRosenbach (Talk | Contribs) 02:33, 22 October 2009 (UTC)[reply]
And, depending on where the hemorrhaging occurs, formation of a hematoma may have a tamponading effect. In general, though, I don't think any lacerated or incised arterial vessel except a very small one would stop bleeding on it's own. A dentist who cuts into the greater palatine artery needs to apply pressure and may need to suture to the palatal bone to ligate in order to stop bleeding. Similar measures are neccesary if one incises the sublingual artery. If I would cut the artery coming out of the mental foramen and just leave it, I think it could very well lead to life-threatening blood loss. DRosenbach (Talk | Contribs) 02:42, 22 October 2009 (UTC)[reply]
The reason I ask is when a surgeon resects some sort of pathology from say, the brain, there are some vessels that must be cauterized, while others can be left to bleed while the surgeon continues to use the aspirator to suck out more lumps of tissue. Is there a guideline somewhere to say when bleeding should/must be "manually" halted and when it can just be left as is (and it is safe to close up the craniotomy/incision)? Sjschen (talk) 03:19, 22 October 2009 (UTC)[reply]
I'd say that any arterial vessel proximal enough to spurt blood in concert with the pulse should be dealt with, rather than leaving it to clot on its own. DRosenbach (Talk | Contribs) 03:32, 22 October 2009 (UTC)[reply]
No guideline that I know. For any given operation, ligating specific named vessels may be part of the procedure (like ligating the uterine artery during a hysterectomy). Beyond that, if a vessel is actively bleeding in the OR, it simply must be stopped. Hemostasis is constantly on a surgeon's mind; neglecting it can easily be deadly. It's guided more by the surgeon's feel and knowledge of anatomy, not breaking out a little ruler and looking up a guideline. - Draeco (talk) 04:03, 22 October 2009 (UTC)[reply]

Rabbit insertion through human cervix: Reality check

Today's featured article, Mary Toft claims that in 1726 the woman miscarried and for some reason, inserted a cat or rabbit through the cervix into the womb, then delivered it in the presence of a man-midwife. Is this remotely possible? What could a poor woman have used as a speculum or dilator in 1726 to place such an object in the womb, and lived to tell of it? Edison (talk) 23:29, 21 October 2009 (UTC)[reply]

I don't know the answer, but I'd be very suspicious of this account, which (as is stated in the article) is purely Toft's own story. The later "births" observed by people with any sort of medical knowledge were of relatively small pieces of animal which had been placed in her vagina. Warofdreams talk 01:39, 22 October 2009 (UTC)[reply]
You either didn't read the article, or didn't read it closely. An accomplice inserted a few animal parts, not entire corpses, immediately after her miscarriage while the cervix was still dialated. Further "deliveries" were just her (or someone else) cramming odds and ends into her vagina. It's all in the article. 218.25.32.210 (talk) 01:35, 22 October 2009 (UTC)[reply]
I'm kinda dumbstruck by this article.. I'm sorry I know the reference desk isn't for discussion or opinion, but seriously. I thought I'd pretty much heard about the depth of human depravity and then an article comes up like this and proves me wrong yet again.. I've heard of people doing things which are more "sick" for some sort of perverted gratification, but this just seems so perverse yet so remarkably pointless and stupid, it's just stunning! I almost sympathise with the doctors who fell for it because they would have undoubtedly asked them selves: why on earth would someone stick rabbit parts up them self?!? The alternative at the time, that she was giving birth to the animal parts, so completely ridiculous actually made more sense to them then the truth!Vespine (talk) 03:56, 22 October 2009 (UTC)[reply]
I don't know, it's not too different than balloon boy. People do goofy stuff. --Mr.98 (talk) 12:33, 22 October 2009 (UTC)[reply]
antiwikipedia Rant (sorry): I was baffled too! I know getting a science featured article or DYK is nearly impossible as there is too much technical jargon, yet there seems to be an endless list of useless featured articles and DYK such as "did you known that someone with a stupid porn name is actually called something else?" or facts like this, how is this remotely noteworthy? it is not cultural (yet, geek culture gets destroyed dy deletionists) nor entertaining nor scientific! Uncyclopedia describes [8] as "has over 3 million articles, mostly about bands no one has ever heard of, one-time Naruto characters, and rare diseases mentioned in passing on House", which is painfully true... --Squidonius (talk) 13:14, 22 October 2009 (UTC)[reply]
Do you have evidence getting a science related DYK is actually 'nearly impossible'? The requirements for DYK aren't particularly high. Also DYK isn't hooks aren't really meant to be noteworthy. They're more meant to me interesting if possible or at least hopefully catch someone's attention. Ultimately of course DYK hooks can only be whatever is in the article and supported by references so even if it isn't that interesting, there's little choice. Ultimately of course many science DYK hooks are likely to have a similar problem since people have already written an article for the core stuff therefore only the more obscure is likely to be lacking an article. For example I see "that Aliquandostipitaceae members have the widest hyphae in the Ascomycetes" which while I understand it (unlike I expect 99% of visitors to the main page) is not exactly that interesting. In terms of this particular article, the fact that people are still mentioning in in the 1990s and 2000s (as shown by the sources) means it has had more staying power then one time Naruto characters, bands no one has ever heard of, rare diseases mentioned in pass on House, most of 'geek culture' and balloon boy Nil Einne (talk) 17:05, 22 October 2009 (UTC)[reply]
DYK is particularly annoying in it's requirements. The article has to be reasonably long - and well referenced - yet it has to have been created fairly recently. This is a difficult set of rules to match. Hitting any two out of the three is easy. The goal of the DYK project clearly isn't to produce a set of interesting (and verified) oddities for the front page - it's an effort to get new articles pushed rapidly to a particular standard. That goal is a worthy one - but it doesn't help readers of the front page very much! SteveBaker (talk) 20:20, 22 October 2009 (UTC)[reply]
I'm well aware of the requirements. But that's precisely my point. DYK is set up to show off new and recently updated articles. (Whether you feel that's a worthwhile goal or likely to be of interest to readers is somewhat irrelevant to my point.) It's not particularly biased against science related articles and I'm not aware of any great difficulty getting science related articles on it. However it is somewhat biased in favour of the more obscure articles (nowadays anyway) since they are ones most likely to not exist or to be stubs. Having said that, there is still a lot of areas in the developing world particularly the non-anglophile and African world that we are sorely lacking in articles that could fill DYK with less obscure stuff if there were the editors for them Nil Einne (talk) 20:44, 23 October 2009 (UTC)[reply]
The particular hoaxer in question is very well documented and very notable. Any book on 18th century hoaxes would be badly incomplete without a chapter on Mary Toft. When it was happening it was very famous.
You have picked a bad example for your rant. Mary Toft is undeniably notable and worthy of inclusion in the encyclopedia. Entire books have been written about her. APL (talk) 19:37, 22 October 2009 (UTC)[reply]
To APL - for what it's worth ... I don't think that anyone in the above discussion is saying that Mary Toft is not notable or that she is not worthy of inclusion in this encyclopedia. Thanks. (64.252.124.238 (talk) 18:02, 25 October 2009 (UTC))[reply]


October 22

free energy of a substance and temperature

So I'm doing ideal solutions. Can I confirm that generally, the free energy of a pure substance doesn't change as temperature increases, because the increase in dH and TdS cancel each other out? (that is, dH = Cp dT; TdS = T (Cp/T) dT). Basically, what is the temperature-dependence of the equilibrium position of Xa and Xb (concentrations) as temperature increases, with regard to a solution of A and B? Thus, in the free energy equation G = (Xa Ga)+ (1-Xa)(1-Ga) + RT ((Xa ln Xa) + (1-Xa)ln(1-Xa)), I don't have to worry about heat capacities or anything, as only the magnitude of the RT term will change, resulting in a more negative minimum a shift in the concentration position of G_min, but the position of the other critical points don't change. Thanks!John Riemann Soong (talk) 04:11, 22 October 2009 (UTC)[reply]

Wait. What are A and B? Are A and B substances involved in an equilibrium reaction such that A <--> B? Or are A & B two different substances which are dissolving, such that A(s) <--> A(aq) and B(s) <--> B(aq)? Are A and B in the same solution, or are each in their own solution? Are A & B ions, or ionic solids, or molecular solids? I'm not sure I understand the situation you are describing fully here. --Jayron32 04:27, 22 October 2009 (UTC)[reply]
A & B are monoatomic species (well I don't think whether being molecular changes things since this is an ideal solution but anyway) and you can adjust their concentration. Presumably you could take A out of solution and replace it with B and vice versa without changing the free energy of the universe. i.e. you're basically God and adjusting the molar concentrations of A and B and looking what happens to free energy. You're making A and B magically appear and disappear and looking at just the free energy of the solution. John Riemann Soong (talk) 04:31, 22 October 2009 (UTC)[reply]
If A & B are functionally identical, then they are functionally identical. However, if they are real different monatomic species, then each will bind slightly differently with the water molecules during solvation. For example, the size of the atom will affect the absolute entropy of the solvation complex, so exchanging A for B will result in a change in dS. Also, the solvation enthalpy will be different, as A and B will bind with water molecules with slightly different strengths, so dH will be different for the two. The deal is, there is not necessarily any connection between the size factor (affecting dS) and the solvation energy factor (affecting dH), so the exchange of A and B could result in any of 4 possible results (+ or - dS and + or - dH) so dG will change, but not in any predictable manner. Depending on how dS and dH change, dG's temperature dependance is also unpredictable without more information; for example a positive dS will result in a very different temperature dependance profile than a negative dS would. --Jayron32 04:44, 22 October 2009 (UTC)[reply]
Am I missing something here? It's an ideal solution? Enthalpy of solvation is zero, and size of the atom doesn't matter ... John Riemann Soong (talk) 05:05, 22 October 2009 (UTC)[reply]
Ah. Sorry, I missed that. Ideal solution contains a discussion of the thermodynamics of mixing in an ideal solution; free energy is dependent only on entropy (indeed, by definition, Free Energy is basically the inverse of entropy anyways); according to the equations there, dG is most definately temperature dependent, as expected, since dS is also temperature dependent. --Jayron32 05:25, 22 October 2009 (UTC)[reply]
Well there's two parts. There's the G_A and G_B part, and the RT(ln[stuff]) part. What I'm saying, is that except for the RT term, dG is not temperature dependent (let's say they don't mix and they're in separate containers). That is, say before mixing, as I heat two compounds up, their free energies will not change, especially w/respect to each other. That is, as enthalpy of each solution goes up (dH = Cp dT , this change is equally matched by a decrease in TdS = T Cp/T dT = Cp dT). So delta-G of temperature change is zero EXCEPT for the mixing component. John Riemann Soong (talk) 05:50, 22 October 2009 (UTC)[reply]

Uhhh, someone help? What happens to G of a solution as you increase or decrease T, and what happens to its G_min concentration (for the cases G_a >> G_b (and vice versa), G_a = G_b, etc.? It can't be that hard -- I just need someone to sort out the concepts for me. John Riemann Soong (talk) 17:50, 22 October 2009 (UTC)[reply]

Except for the RT term? Thats a pretty big except. Except for not having any money, I am a rich man. Seriously. dG is temperature dependent. Raising the temperature of the system will change the free energy; the T is right there. Look:

You can't just ignore the temperature. If I change the number that T represents, the number that deltaG represents changes too. I don't understand why you just want to ignore that. --Jayron32 20:29, 22 October 2009 (UTC)[reply]

I don't think he's looking to ignore T, rather he's trying to find exactly how G depends on T. Rckrone (talk) 01:06, 23 October 2009 (UTC)[reply]
It's a direct linear relationship. Look at the equation. DeltaG and T are both in there, and they appear on opposite sides of the equal, and on the same level and at the same power. That's a direct and linear relationship. If you plot deltaG vs. T, you'll get a line with thge slope of . This is pretty basic mathematical analysis here. The OP however seems to assert that deltaG is not dependant on T, which is plainly wrong. --Jayron32 01:48, 23 October 2009 (UTC)[reply]
I don't think that's what he was saying though. The formula you're talking about is specifically the free energy of mixing, which isn't the total Gibbs free energy. The OP asked first if ΔG is constant in T (I assume at constant P) for a pure substance (free energy of mixing is not involved). Then he asked in the case of a mixture, if the only in G with respect to T comes from the change in the free energy of mixing (the part you discussed), or if the other components of the total free energy will change as well.
I am really not familiar with this topic, but just going by the formula for Gibbs free energy given in the article, G = H - ST, results in dG = dH - SdT - TdS. At constant pressure dH = TdS as shown, but the SdT term remains, so dG/dT = -S rather than 0. This might be totally wrong but Gibbs–Helmholtz equation seems to corroborate. Rckrone (talk) 06:41, 23 October 2009 (UTC)[reply]
Thank you. I know how to deal with the mixing component -- it's just that I had no idea how to deal with the rise in the free energies of the pure substances, which are no longer constant. Also, how to sort out any possible interaction between the mixing free energy changes and the non-mixing free energy changes. I spent like 5 hours over this without any help. Thanks guys, I guess I'm getting to get a 7.5/10 for my homework. FML. I was looking at a pure substance so then it made it easier to deal with a mixed substance. John Riemann Soong (talk) 09:37, 23 October 2009 (UTC)[reply]

Jayron: I wanted to ignore the mixing component for the time being and talk only about the pure components because the mixing component is the part that's covered by my notes, the part that my group members are confident with, and the part that I essentially know how to do and did lots of laws of logs with. However, I don't know what to do with the rise the free energy of the pure components -- the rise in G_a and G_b!!!!! That's why I wanted to ignore the temperature effects on the RT term, cuz the RT term I have already worked out. Awww, why did you have to do this to me? =( John Riemann Soong (talk) 09:39, 23 October 2009 (UTC)[reply]

Isn't p-chem fun stuff? --Jayron32 12:46, 23 October 2009 (UTC)[reply]
Haha, this is p-chem lite, not taking the real one yet ... I'm taking a materials science (phase transition thermodynamics) thing for my sequence. John Riemann Soong (talk) 15:18, 23 October 2009 (UTC)[reply]

How can I obtain pure sulfur, mercury, lead, and antimony?

Hello,

My friend is giving a presentation on Paracelsus. He's going to talk about how Paracelsus prescribed sulfur, mercury, lead, and antimony for ailments. Is there a way to obtain these substances for the presentation? It's just to show the audience what they look like. We don't need all of them (although that would help).

Thanks,

Drknkn (talk) 04:39, 22 October 2009 (UTC)[reply]

Sulfur and lead are relatively easy to find; sulfur and lead are readily availble from Fisher Scientific; thus they are probably availible from your school's chemistry teacher. I've not seen a stanard educational stockroom carry antimony, but Fisher does sell it. Mercury is readily availible in things like thermometers. Its hard to find a jar of Mercury for sale anymore, but mercury thermometers are still for sale, again from fisher. --Jayron32 04:49, 22 October 2009 (UTC)[reply]
Cool. Thanks.--Drknkn (talk) 06:13, 22 October 2009 (UTC)[reply]
Some of those substances are toxic. Messing with them just for a presentation like that doesn't seem worthwhile. Show some video of them or something like that instead. 69.228.171.150 (talk) 07:19, 22 October 2009 (UTC)[reply]
I agree. Mercury for example is seriously toxic: do not touch it, expose it to the air, or leave it lying around. Best to have nothing to do with it. 78.146.56.118 (talk) 18:37, 22 October 2009 (UTC)[reply]
It is tempting to demonstrate the strange way Mercury rolls around as a shiny and unexpectedly heavy liquid metal and this is probably still done in some schools (it was in mine) in spite of the danger of poisoning. Either follow the advice not to handle Mercury or do so only in a Glovebox. Cuddlyable3 (talk) 19:13, 22 October 2009 (UTC)[reply]
What you can buy, depends on where in the world you live. e.g. In the USA you could probably obtain these chemicals, in the UK - don't bother (except lead), it's not possible for individuals to buy chemicals. I think most of Europe is also very difficult.  Ronhjones  (Talk) 19:45, 22 October 2009 (UTC)[reply]
Pure sulfur would most likely be a powder. Would that make it not a chemical? Googlemeister (talk) 21:23, 22 October 2009 (UTC)[reply]
No, being a chemical doesn't depend on the phase of matter. Rckrone (talk) 07:23, 23 October 2009 (UTC)[reply]
Isn't it somewhat unusual to use the word chemical for a pure element, though? I think compound would, certainly, and the words tend to be used interchangeably. --Trovatore (talk) 07:29, 23 October 2009 (UTC)[reply]
As in medicinal compound. Cuddlyable3 (talk) 15:24, 23 October 2009 (UTC)[reply]
Reading the sulfur article, it appears that ancient medicine (Paracelsus and earlier) used sulfur in creams to treat acne. the article also says that sulfur is still an active ingredient in some modern-day creams against acne. That makes for an interesting link between Paracelsus and modern medicince, as well as a sulfur-compound that's safe and easily available. —Preceding unsigned comment added by EverGreg (talkcontribs) 07:39, 23 October 2009 (UTC)[reply]
See Paracelsus#Contributions_to_medicine. A nice saying about the use of Mercury compounds[9][10] to treat syphilis[11] was "A night in the arms of Venus leads to a lifetime on Mercury". Cuddlyable3 (talk) 15:11, 23 October 2009 (UTC)[reply]

streoscopy

How to make stereoscope ? i have created a movie in a view of creating a 3D picture. Using the technology of two cameras at some distance. now i need a viewing glass. guide me to make it. thanks in advance...

hoping a better result.

yours ravivarma,RAJA.M —Preceding unsigned comment added by Rjravivarma (talkcontribs) 08:22, 22 October 2009 (UTC)[reply]

I would start with our article stereoscopy, especially the references and external links. I also found a page 'Let us Build a Stereoscope', but that may be too elementary for your purposes. stereoscopy.com seems to have a wealth of information and resources. --LarryMac | Talk 14:46, 22 October 2009 (UTC)[reply]
It really depends on what kind of viewing situations you need. One very simple trick is to take the two pictures and print them side-by-side onto a single sheet of paper - or place them side-by-side on your computer screen. Now you can allow your eyes to cross slightly such that you now see two copies of each image overlaid on each other...with practice, you can let the left-hand copy of the right-hand image lay exactly on top of the right-hand copy of the left hand image. When you do this, the image in the middle of the row of three suddenly "pops out" in 3D - while the ones on either side look kinda hazy and transparent.
Slightly more complex - get a pair of those red/blue glasses (they are really red/cyan). Take one image into photoshop or GIMP and delete it's red layer and the other and delete it's green and blue layers - then use the 'composite' feature to put them back together again. Now you have an image that'll pop into 3D with red/cyan glasses.
You could try to make - or buy - a stereoscope...or if you are REALLY cheap - you can print out the image to the right here at an appropriate scale onto some thin cardboard - cut and fold - and you're done!
It can get more and more complex, the fancier you want to get - but that's a good starting point. Here is some other random advice:
  • It's very easy to get the two images switched over and get peculiar inside-out 3D.
  • To get the best results, try to keep your cameras within a few inches of each other and take mostly pictures of things that are less than about 20 to 30 feet away. Beyond that distance, the 3D effect isn't strong enough to give you good results.
  • You can increase the distance between the cameras to maintain the 3D effect out beyond 20 to 30 feet - but what happens is that your brain starts to think the objects in the picture are toys...models of the actual objects. Sometimes this is a fun 'effect' - but it's not what you want for good realism.
  • Make sure that the cameras are at the same vertical height and that the top and bottom edges of their pictures lie in the same straight line. Our brains get very confused - and you can actually make people want to throw up - if you break this cardinal rule.
SteveBaker (talk) 21:02, 22 October 2009 (UTC)[reply]
I've been experimenting with 3D photography. Two significant points you've missed:
  1. You want to use a normal or near-normal lens. If you're using more than a mild telephoto lens (85mm or so on a 35mm camera), the viewer won't be able to fuse the entire scene at once -- they can put the foreground, the subject, or the background in 3D, but the rest will be double images.
  2. You want the camera-to-subject distance to be between 20 and 30 times the camera-to-camera (baseline) distance. None of the one-inch baseline pictures of my desk are online, but I do have a stereopair of Mount Hood taken at about 25 miles with a baseline of almost a mile.
Related to the above, you don't want the depth of the scene to vary too much. You can get away with a high subject-to-background distance (the background will simply look flat), but the foreground needs to be close to the subject or you'll get double images rather than 3D. --Carnildo (talk) 22:57, 22 October 2009 (UTC)[reply]

human electricty

How and where is electricty generated in the human body for muscle adtivation?

Like with neurons, it is not electricity as in electron (negative) flow but it is flow of ions, in muscles being calcium (2x positive) from outside to inside the cells. This however, is just a signal, the driving force are Motor protein which burn ATP. --Squidonius (talk) 14:36, 22 October 2009 (UTC)[reply]
I have a bit of trouble understanding the previous response; here is my version. The membranes of muscle cells contain ion pumps (in the form of specialized protein clusters) that turn them into batteries, with a voltage difference between the inside and outside. The battery does not directly power muscle contraction though -- it drives a flow of calcium ions across the membrane, and the calcium ions activate contractile proteins that produce the muscle power. Looie496 (talk) 19:02, 22 October 2009 (UTC)[reply]
To be more specific:
1) the cell membranes are capacitors, or in other words, charge separators. Charge (in the form of ions) builds up on both sides of a membrane (in the form of positive on one side and A LOT more positive on the other side) and because the ions cannot cross the membrane until certain channels are opened, the charge builds. When the channels finally do open, ions RUSH through them in order to balance both the electrical gradient and the relative ion concentration gradients.
2) calcium ions bind to troponin, forcing tropomyosin to shift from myosin-binding sites on the actin filaments. When the these binding sites become exposed, the myosin heads attach and perform their powerstrokes, thus contributing to muscle contraction.
3) nerve and muscle "electricity" in the form of capacitance and transfer of charge can be measured with an oscilloscope using alligator clips and the like. DRosenbach (Talk | Contribs) 22:52, 22 October 2009 (UTC)[reply]
The idea is that because you have two different species of ions you have both electrical forces and diffusion/osmosis forces at work; sometimes they oppose each other and sometimes they enhance each other. Actually ions don't "gush" into the membrane, in as much as electrons aren't "gushing" out of a battery. In general only a tiny percentage of the potential energy stored is consumed before electrochemical equilibrium is restored. Otherwise a single neuron would get tired pretty quick! John Riemann Soong (talk) 14:46, 25 October 2009 (UTC)[reply]

Alternate Universe?

So besides the universe being everything that is physically existing (the solar system, planets etc.) what are the chances of coming across an alternate universe? Like the one shown here: http://www.youtube.com/watch?v=cryej86SmCk


It just makes me feel so small and less substantial when I see how small I am compared to the rest of what exists. —Preceding unsigned comment added by 139.62.167.223 (talk) 15:16, 22 October 2009 (UTC)[reply]

There are lots of different theories about alternate universes. See multiverse for descriptions of some of them. I would argue that if it is possible to visit somewhere then it is, by definition, part of our universe, but other people define things differently. --Tango (talk) 15:19, 22 October 2009 (UTC)[reply]
OP the chances of you coming across an alternate universe are slim. The chances of you coming across the pseudoscientific scenario where "nanorobots carry our DNA" are zero. Your video link shows five minutes of slick graphics, wordbytes from alleged "experts", "We physicists have calculated" and "Some of the world's leading physicists" who are anonymous or dead. It has a percussive soundtrack to stop clear thinking, such as noticing when COULD changes to CAN at 4:48. The 4:36 spaceships and the 4:56 astronaut were pretty sci-fi props and I suggest you try to enjoy them as such. Cuddlyable3 (talk) 18:41, 22 October 2009 (UTC)[reply]
Nanotech doesn't exist like that yet, but it easily could in the future. However, that wouldn't help with travelling through a wormhole. The only way you could possibly do it is to exploit quantum effects, which means being of at least atomic sizes (which I think the clip actually said) - an atom is on the scale of 100 picometres, a picometre is 1/1000 of a nanometre, so we're talking significantly less than a the scale of nanotech. --Tango (talk) 20:00, 22 October 2009 (UTC)[reply]
...and way WAY smaller than a DNA molecule. Yeah - this was all so much B.S. They've taken a collection of the most speculative hypotheses out there and strung them all together (making it speculative-squared!) and then white-washed all of the difficulties and extrapolated still further from there to a flat out crazy conclusion. There are much simpler ideas that are harder to get your head around and much more believable. Take something as seemingly nutty as quantum suicide - and realise that this only needs one out of the half dozen things that video uses to be true! Or how about digital universeSimulation hypothesis - that could easily be true with nothing more than the laws of physics as we know them. If our relatively mundane universe is infinite (as it very well might be) - then we don't need a parallel universe in order to have another person identical to you, reading an identical post to this one that's different only in that this sentence ends with a colon instead of a full-stop: SteveBaker (talk) 20:11, 22 October 2009 (UTC)[reply]
You don't need to fit a DNA molecule in it - you can send multiple picoprobes if you need to. I'm not sure how to make something that small that can create the cloning technology required on the other side, even if you send trillions of them, but transmitting the DNA information using such picoprobes might well be possible. A traversable wormhole is the more unlikely discovery, IMO, even with quantum effects. --Tango (talk) 21:31, 22 October 2009 (UTC)[reply]

I love being told that I can do what I obviously cannot do because that makes me feel so small and less substantial like the OP. Cuddlyable3 (talk) 13:06, 23 October 2009 (UTC)[reply]

I'm not sure I understand... are you complaining about my use of the generic you? --Tango (talk) 15:16, 23 October 2009 (UTC)[reply]
When one finds oneself addressed on one's personal screen one must consider as I do the implication that one may in fact be the one that that other one is addressing. Or not. You can't be too careful can you?. Cuddlyable3 (talk) 23:11, 23 October 2009 (UTC)[reply]
Sounds like someone has been through the Total Perspective Vortex. I'm sorry. Imagine Reason (talk) 17:43, 24 October 2009 (UTC)[reply]

Fine-tuned Universe?

As I always say when I ask these questions lol, I know very little science and will apologise in advance for the huge display of ignorance I'm probably now about to make.

Arguments about a fine-tuned Universe and the anthropic principle tend to come up a lot when the existence of God is discussed. But to me all these theories tend to rest on the idea that universal constants are arbitrary rather than logically self-evident: we don't look for meaning in the value of pi or e, or even root 2 for that matter, because they are logically self-evident, it cannot be logically conceived that those could have values that are something else.

It's not really treated in the article, so I thought I'd ask here. Is there any scientific consensus about the nature of these constants? Would a scientist work assuming that they were arbitrary (when I say arbitrary I don't mean uncaused, I just mean that there potentially could be alternatives) or assuming that they were logically self-evident and the only possible values of what they are?

Even if there's no consensus, I'd be curious to know what the arguments were on either side of the issue, as it's something that interests me a great deal. —Preceding unsigned comment added by Dan Hartas (talkcontribs) 15:53, 22 October 2009 (UTC)[reply]

That is one of the big unsolved problems in science. The anthropic principle explains the values of the constants pretty well, but there are a lot of scientists working on finding more satisfying explanations. For example, inflationary theory explains why the average density of the universe is so close to the critical density. --Tango (talk) 16:04, 22 October 2009 (UTC)[reply]
In these discussions, one should take care to distinguish the fundamental mathematical constants (including π, e, and so forth) from fundamental physical constants (like α, the fine structure constant). Our articles physical constant and dimensionless physical constant do a pretty good (if cursory) job of defining the differences. Essentially, mathematical constants are the result of specific, arbitrary axioms and conventions we have chosen to use in defining mathematics, whereas physical constants have to be derived from measurements of the actual properties of our universe. The mathematical constants have to be what they are because they're part of the definition of the system. TenOfAllTrades(talk) 16:07, 22 October 2009 (UTC)[reply]

The "Numerological explanations" section of the fine structure constant article might interest you. 69.228.171.150 (talk) 16:10, 22 October 2009 (UTC)[reply]

All this sort of discussion begins and ends with an observer. An observer who is at leisure to ponder the arcane mysteries of the universe, rather than fighting off a king cobra or taking cover from a thunderstorm. Contemplation done at ease will usually end up concluding that the universe is finely-tuned for our human comforts. Vranak (talk) 16:45, 22 October 2009 (UTC)[reply]

It's not really the "fine-tuned for people" bit that I'm wondering about- it's more whether the laws of the Universe could logically ever be any different at all, irrespective of human comforts. —Preceding unsigned comment added by Dan Hartas (talkcontribs) 16:54, 22 October 2009 (UTC)[reply]

Most of our theories require at least some constants to be measured empirically and just plugged in, but work is being done to reduce the number of arbitrary constants. I'm not an expert of string theory, but I believe it could explain the masses of elementary particles, for example, by the resonant frequencies of the strings. --Tango (talk) 17:30, 22 October 2009 (UTC)[reply]
As far as I know, string harmonics have energies that are multiples of the Planck mass, which is far too high to account for any of the Standard Model particle masses (or anything else that has ever been seen experimentally). Certainly no string-theory explanation of anything in the Standard Model is known right now; it hasn't even been shown that string theory is consistent with the Standard Model. -- BenRG (talk) 00:17, 23 October 2009 (UTC)[reply]
I thought the whole point of string theory was that all particles are just strings vibrating in different ways. As far as I know, string theory is reasonably well developed - I think they have worked out how most of the normal particles would work. I think the harmonics you are talking about are the super-symmetric particles - the regular particles are presumably the fundamental frequencies. --Tango (talk) 15:22, 23 October 2009 (UTC)[reply]
As far as I know no one has managed to construct the Standard Model (or rather, something experimentally indistinguishable from it) inside string theory, but if it is possible then it will be by a Kaluza-Klein-like mechanism. The idea of Kaluza-Klein theory (which is much older than string theory) is that all particles (or at least all bosons) are spacetime waves (just like the graviton), but, except for the graviton, the waves involve extra dimensions besides the obvious four. The Kaluza-Klein-like dimensions in string theory are not the famous 6 (=10−4) dimensions that are supposed to have the form of a Calabi-Yau manifold, but the 16 (=26−10) extra dimensions from heterotic string theory, which form a 16-dimensional torus. A 16-dimensional torus should only give you 16 particles, but for some string-specific reason that I don't understand, but that has something to do with winding modes of the string around the torus, they get a much larger symmetry group, either SO(32) or E8×E8, giving 496 bosons. To match the Standard Model you have to match some of those with Standard Model particles and also explain why the others haven't been seen. I don't think that string vibrations are involved in any of this. The particles are vibrations of spacetime, but that's inherited from quantum field theory and Kaluza-Klein theory, where there are no strings to vibrate. At any rate it's a complicated and specific construction, not a simple matter of cellists floating through space as that awful NOVA episode would have you believe. I don't think there are any new unifying principles in string theory; they're all inherited from non-string theories like KK, the Standard Model, and supergravity. People study string theory because it seems more likely to actually work, not because it's philosophically more elegant or simple. -- BenRG (talk) 19:49, 24 October 2009 (UTC)[reply]
There is another point to be made - some people obsess about the actual numerical values - there is a movement amongst some physicists to define our units such that these constants all come out to 1.0. So (for example) instead of the plank distance being some arbitary number of meters - we simply call it 1.0 and let the 'meter' be defined as some horribly large number of plank units. Similarly, the speed of light would be 1.0 - and the 'second' would become some number of (speed-of-light-units x plank-distances). This doesn't really help to answer the question - but I can't help but feel it takes away some of the feeling of arbitaryness. In that view of things - we literally could not ask "What would it be like if the speed of light were twice as big?" because we've defined it as being 1.0 in all possible universes. You'd have to ask yourself a completely different set of hypothetical questions instead. Instead of saying "What would it be like if the plank distance was half as big?" you'd ask "How would the universe have evolved if everything that popped out of the big bang (EXCEPT the plank distance) was twice as large?" - and I can't help but think that some kind of insight lies in that direction. It changes the question from how these fundamental constants could be different to how changing the attributes of the initial state of the universe would cause it to evolve differently. SteveBaker (talk) 20:02, 22 October 2009 (UTC)[reply]
Well, this trick goes only so far. Once you've removed the units from h-bar, c, and G, you're pretty much done; that de-dimensionalizes mass, length, and time, from which all our other units can be derived. So now you're left with dimensionless constants, which can't be redefined away to 1.0 . The fine-structure constant already mentioned, the ratio of the proton mass to the elsectron mass, the charge of the electron when expressed in the Planck units, that sort of thing. Dan Hartas's question about whether these can logically be different is a real question. My provisional answer would be, I certainly see no logical reason why they couldn't. But that's open to revision, if someone comes up with a convincing argument. --Trovatore (talk) 20:11, 22 October 2009 (UTC)[reply]
What you're talking about are systems which employ so-called natural units. TenOfAllTrades(talk) 20:09, 22 October 2009 (UTC)[reply]
They aren't made equal to 1.0, they are made equal to 1. You should only say 1.0 if you mean 1 +/- 0.05. If you mean the integer, just say "1". --Tango (talk) 21:23, 22 October 2009 (UTC)[reply]
I disagree — this is the real number 1, which is a different object from the natural number 1. --Trovatore (talk) 21:31, 22 October 2009 (UTC)[reply]
The real number is still denoted (as you have denoted it) by simply "1". Trailing zeros after the decimal point are only used to denote a particular level of precision, if you are being exact then you don't use trailing zeros. --Tango (talk) 23:02, 22 October 2009 (UTC)[reply]
Strictly speaking, you should use infinitely many trailing zeroes.
The convention of using "1.0" to mean the exact real number 1, as distinct from the natural number 1, comes more from software than from science, but I think it's a useful one in some contexts. It allows a short way of expressing oneself when explaining, for example, that 00 is 1, but 0.00.0 is undefined. To be sure, it does have to be distinguished from the competing convention of using the number of digits reported to give an approximate idea of your degree of uncertainty. --Trovatore (talk) 23:07, 22 October 2009 (UTC)[reply]
How can you be strictly speaking supposed to do something impossible? Yes, in software 1.0 is used to force the computer to treat the number as a real number (so you don't get integer division, for example), but we aren't computers. Outside of the realms of computer programming, trailing zeros after decimal points are only used to denote precision. (Whether the natural numbers are a sub-semiring of the real numbers or just isomorphic to one is purely a question of semantics and isn't worthy of discussion here.) --Tango (talk) 23:20, 22 October 2009 (UTC)[reply]
"Purely a question of semantics?" What's more important than semantics? Semantics is the study of meaning itself. You are descriptively just wrong that trailing zeroes are used only to denote precision. --Trovatore (talk) 23:32, 22 October 2009 (UTC)[reply]
By "semantics" I mean the meanings of words. The meanings of the words aren't important, it is the underlying concepts that are important not the way we express them. --Tango (talk) 15:30, 23 October 2009 (UTC)[reply]
Physicists always write 1 in this situation, never 1.0, and so do mathematicians; the reciprocal of a real x is 1/x, the circumference of a circle is 2πr. For that matter, I write 2 * M_PI * r in C code too, though I've noticed that many programmers do consistently add .0. -- BenRG (talk) 00:17, 23 October 2009 (UTC)[reply]
I am a mathematician, and I occasionally use this notation to make this distinction, on those occasions when the distinction matters. So we have a counterexample. I am not alone in this, I think.
Your examples don't really prove anything; there's only one interpretation for natural-divided-by-real or natural-times-real, and the value equals the one that you get if you first apply the natural embedding from the naturals into the reals. Granted, you could look at values like "3/4", which mathematicians never use to mean "0", but still it doesn't necessarily mean the real value 3/4; it could be the rational 3/4 (which is still in the domain of algebra rather than analysis, unlike the real number ). --Trovatore (talk) 00:26, 23 October 2009 (UTC)[reply]
When we are doing rigorous constructions we distinguish between, for example, the rational number 3/4 and the equivalence class of Cauchy sequences that tend to that rational number. The rest of the time, we just identify the two things. There is rarely any need to distinguish between the rational number 3/4 and the real number 3/4, there certainly isn't any need to do so outside of pure mathematics. --Tango (talk) 15:30, 23 October 2009 (UTC)[reply]
Well, it's good mental discipline, though, and protects you from certain categories of error. Also it's useful pedagogically, in getting across the idea of a real number, which is highly non-obvious to most people. --Trovatore (talk) 20:05, 23 October 2009 (UTC)[reply]
(ec) "1" has significance as the identity element or the concept of unity. Even in a real numbers context it's more than just another value on the real line, which is what's brought to mind, at least for me, when I see "1.0". Water has a density in g/cm3 of "1.0". In natural units c is unity, or "1". There's a conceptual difference there and the choice of notation helps convey it. I don't know if I would go so far as to say that "1.0" here is strictly wrong, but it's definitely not more correct. Rckrone (talk) 00:37, 23 October 2009 (UTC)[reply]
It's true, there is a conceptual difference between "exactly the real number 1" and "rounds to this value within the precision I'm stating". But there's also a conceptual difference between the real number 1 and the natural number 1, and Steve's comment clearly evokes the former. So it's less correct on one dimension but more correct on another one. --Trovatore (talk) 00:41, 23 October 2009 (UTC)[reply]

Question: When counting how many angels can fit on a pin head, should we use real or natural numbers? Dauto (talk) 05:42, 23 October 2009 (UTC)[reply]

Natural numbers, of course. Unless the answer is infinite, in which case the transfinite cardinals are the way to go. --Trovatore (talk) 07:23, 23 October 2009 (UTC)[reply]
This actually is the classical illustration of the difference between countable and uncountable infinities. There are a countably infinite (aleph-null) number of angels on the pin, but each of them has an uncountably infinite space in which to dance... Tevildo (talk) 14:47, 23 October 2009 (UTC)[reply]
Note though that you can't give them all the same amount of space. The same numer of points, yes, but not the same two-dimensional Lebesgue measure. Either some angels are more privileged than others, or they dance on some peculiar fog of points to which a well-defined area cannot be assigned.
This is actually the key idea behind the Vitali set. --Trovatore (talk) 20:02, 23 October 2009 (UTC)[reply]
Could you give them all a space of zero measure? --Tango (talk) 18:01, 24 October 2009 (UTC)[reply]
No, not unless the pinhead itself has measure zero. This is by countable additivity. --Trovatore (talk) 19:59, 24 October 2009 (UTC)[reply]
Take all the laws of physics. Take their Gödel numbers. These are all arbitrary constants. We could have ended up in a four-dimensional universe, or a universe where forces cause jerk instead of acceleration, or any other totally alien laws of physics. Ours are arbitrary. — DanielLC 15:15, 23 October 2009 (UTC)[reply]
That would be a cool universe..
You don't know that. Dauto (talk) 17:18, 23 October 2009 (UTC)[reply]
It's OK - I've just done a Godel numbering of Dauto's reply - turns out it's just an arbitary constant too! SteveBaker (talk) 01:21, 24 October 2009 (UTC)[reply]

Hi, second paragraph, clarification tag, subject :- favourable and unfavourable thermodynamic reactions, a metabolic process couples them to even them out. Can anyone just define those two terms or give a "such as" example? (especially in a manner to add to the article) It's actually GA or FA. What is the character of favourable and unfavourable thermodynamic reactions in a metabolism? ~ R.T.G 16:43, 22 October 2009 (UTC)[reply]

I've reworded this, hopefully it is clearer now. Tim Vickers (talk) 17:21, 22 October 2009 (UTC)[reply]
It is, thank you. ~ R.T.G 17:34, 22 October 2009 (UTC)[reply]

Alignment of the planets

Often, school children are taught the order of the planets as seen in this illustration: solar system. I can imagine that younger children envision that the planets actually "look like that" -- that is, aligned neatly in a horizontal line, one after the other -- in that particular order (Mercury, Venus, Earth, Mars, Jupiter, Saturn, Uranus, and Neptune). However, this is not the case, since each planet follows its own independent orbit. My question is: do the eight independent orbits of the eight planets ever align themselves so that all eight planets actually do appear in a straight horizontal line, as in that illustration? If so, how frequently (or infrequently) does that phenomenon occur? And, is there a name for that? If not, what is the "closest" that the eight planets ever come to achieving that horizontal alignment? Thanks. (64.252.124.238 (talk) 19:12, 22 October 2009 (UTC))[reply]

That would be an extreme case of (strict) Syzygy. Our article describes one such instance with four of the planets. I'm trying some more widespread searches, but that word seems to have been used as the name of a game, a musical act, and at least one corporation, so the search parameters will take some tweaking. --LarryMac | Talk 19:22, 22 October 2009 (UTC)[reply]
It strikes me that the looser definition of syzygy given in our article, which simply requires all planets to be "on the same side of the sun" would allow the appearance of a straight line of planets to an appropriately placed observer. --LarryMac | Talk 19:43, 22 October 2009 (UTC)[reply]
They wouldn't necessarily be in the right order, though. --Tango (talk) 20:23, 22 October 2009 (UTC)[reply]

Assuming the planets follow neat non-interacting Keplerian orbits with well-defined constant "years" (time needed to complete one orbit), the answer to your question is simply the least common multiple of the length of every planet's year.

Also, when I was a child I suffered from the very misconception you talk about. I wondered how it was possible to see Saturn given that Jupiter is both bigger and closer and therefore "stands in the way" to see saturn :-) The grownups never managed to give a satisfactory answer though - perhaps due to their reluctance to clearly say that the picture does not represent the real situation. —Preceding unsigned comment added by 81.11.174.233 (talk) 19:31, 22 October 2009 (UTC)[reply]

The problem is that periods aren't integer years. We can probably assume that the periods don't have precisely rational ratios, which means there's not guaranteed to be a time when any more than 2 planets are perfectly aligned with the sun. However there will be times when 3 or more come arbitrarily close to being perfectly aligned. Rckrone (talk) 20:07, 22 October 2009 (UTC)[reply]
I had thought about the least common multiple (LCM) method (mentioned above by IP 81.11.174.233). But, wouldn't that require that all eight planets actually "start" in a perfectly horizontal line to begin with? And, if so, then every XXX years, they would all return to that "starting point". Right? I can't imagine that, one day, they all "started out" in a horizontal line. Or is my analysis of the LCM method flawed? Thanks. (64.252.124.238 (talk)) —Preceding undated comment added 20:11, 22 October 2009 (UTC).[reply]
Hypothetically, there will be some time when all 8 planets should align as described. It may very well be after the heat death of the Universe, but mathematically it should be possible to calculate how often it will happen. You don't even need to assume that they all had that particular "starting position"; the orbits of all 8 planets should represent a repeating pattern, and you just need to know the time period of one "cycle" of that pattern, and the position in that pattern we are today. Such calculations may require computers to work out all of the math, but it is at least theoretically possible to do it... Of course, that all assumes an "ideal solar system" where the planets ONLY gravitationally interact with the Sun and with no other objects, such as each other. Once you consider that the planets exert gravitational forces on each other, then you have an n-body problem, and the mathematics and physics tell us that the behavior of such systems is chaotic and unpredictable. So then again, there may well be NO way to predict when such an event would occur. --Jayron32 20:19, 22 October 2009 (UTC)[reply]
There is only going to be a repeating pattern if all the orbital periods have rational ratios, which is a big assumption. --Tango (talk) 20:26, 22 October 2009 (UTC)[reply]
Stability of the Solar System says "even the most precise long-term models for the orbital motion of the Solar System are not valid over more than a few tens of millions of years". Plus the Sun is going to dispose of a few planets long before the heat death anyway. Clarityfiend (talk) 02:52, 23 October 2009 (UTC)[reply]
(ec) More likely, they just didn't know the answer. I remember having a supply teacher (or maybe a student teacher) in primary school that insisted Mars was first and Mercury fourth (I'm guessing she knew a mnemonic and got the M's the wrong way around). She wouldn't listen when I tried to correct her... You missed out an assumption there - you need to assume they all started out in alignment. Also, you'll only get a finite lcm if the ratios of the orbital periods are rational numbers. Those two problems mean your method only tells you how frequently they will happen if they happen at all, it won't tell you if they happen. In reality, you need to be a little flexible in your definition - requiring all the planets to be within a 10 degree sector, for example, is far more reasonable and can be calculated (although I can't find anyone that has done it - there might well not be any occurrences within the time period where we can reliably model the interactions). --Tango (talk) 20:23, 22 October 2009 (UTC)[reply]
There have been various well-known close approaches to a grand alignment. On 4 February 1962, the Sun, the Moon, and all the planets from Mercury to Saturn were clustered within a 17-degree area of the sky. There was also a total eclipse of the Sun. Doomsayers had a field day, but catastrophe mysteriously failed to occur. -- JackofOz (talk) 20:35, 22 October 2009 (UTC)[reply]
We are forgetting that the planets are not perfectly in the same plane, so even if all the planets were to be aligned in the 2-D plane, a person standing on Neptune might not have all planets crossing the sun since Jupiter, or Mars, might be higher or lower then the plane right? I mean we have the ecliptic plane, but other planets "ecliptic plane (for the planets)" might very well be different. Googlemeister (talk) 20:49, 22 October 2009 (UTC)[reply]
Indeed. If you include precession in the calculation you might find a time when the nodes of all the orbits were in a line and the planets were all at those nodes - then you would genuinely have them all in a line. --Tango (talk) 21:18, 22 October 2009 (UTC)[reply]
It's not that hard to do a rough calculation. Uranus and Neptune come into approximate alignment about once every 160 years -- we can safely assume that the positions of the other planets are independent random variables during these events. In the 4.5 billion year history of the solar system there have been around 30 million such events. Crunching the numbers yields a prediction that the planets have very likely aligned to within 25 degrees at least once but probably have never all aligned to within 20 degrees. Looie496 (talk) 00:44, 23 October 2009 (UTC)[reply]
Uranus and Neptune are always in perfect alignment - any two points make a line. You need to consider at least three planets for it to be interesting. Did you mean Uranus, Neptune and Earth? --Tango (talk) 15:37, 23 October 2009 (UTC)[reply]
I meant Uranus, Neptune, and the Sun -- sorry, thought that would be taken for granted given the original problem. I used the outer planets because they have the longest periods, 84.3 and 164.8 years. The periods of the inner planets are so short that they are practically randomized for each alignment (wrt to the Sun) of Uranus and Neptune -- even Saturn has a period of only 29.5 years. Looie496 (talk) 21:25, 23 October 2009 (UTC)[reply]
Ok. You can't really take that for granted - people are often interested in two planets and the Earth being in rough alignment because that means they are very close together in the night sky. Two planets and the Sun being in alignment doesn't look interesting at all from the point of view of an observer on the Earth (although if it is close enough it might mean one planet transits across the sun from the point of view of the other planet). --Tango (talk) 18:05, 24 October 2009 (UTC)[reply]

Follow up

Thanks for the input above. Let me ask the question another way, in order to help me understand this concept. Assume that the solar system will exist forever (ad infinitum) and also assume that there will be no changes whatsoever (to the planets or their orbits, etc.). Given these two huge assumptions ... is it a mathematical certainty that the eight planets will some day eventually perfectly align? Or is that not even certain? Thanks. (64.252.124.238 (talk) 15:11, 23 October 2009 (UTC))[reply]

No, it's not certain due to the chance that the orbital periods of the planets may have irrational ratios (which sounds like it should be an oxymoron, but never mind). That is, if one planet orbits once every X seconds and one orbits every Y seconds then there might not be any pairs of positive whole numbers a and b such that aX=bY. That means that they can line up perfectly with a third planet at most once in their lifetime and that once may have been before the planets actually existed. --Tango (talk) 15:36, 23 October 2009 (UTC)[reply]
They will almost certainly never align perfectly. Given sufficient time and no changes in the system they would probably sooner or later align to any desired precision -- it might take 10100 years though. Looie496 (talk) 21:25, 23 October 2009 (UTC)[reply]
Very true. There are always rational number arbitrarily close to any irrational number. --Tango (talk) 18:05, 24 October 2009 (UTC)[reply]

Great! Thank you for all of the input. It was helpful and informative. Thank you! (64.252.124.238 (talk) 16:40, 25 October 2009 (UTC))[reply]

methycobolamin

how does methycobolamin (B12) exectly works?i need full detail. —Preceding unsigned comment added by Klaricidxl (talkcontribs) 20:39, 22 October 2009 (UTC)[reply]

Have you read Vitamin B12#Functions? methylcobalamin is the way to spell it. Graeme Bartlett (talk) 21:07, 22 October 2009 (UTC)[reply]

Cable TV distribution

We have cable TV and it is distributed around the house to various points. A technician from the cable company set that up for us, unofficially. We only have one set-top box (digibox) through which we can get the range of channels we subscribe to. I think the distribution takes place before the signal goes through the box. The other sets in the house until recently were receiving the analogue channels, what in the UK are called the "terrestrial channels" (BBC1, BBC2, ITV1, Channel 4, Channel 5). Now on two of those five channels we have a message that the analogue channel has been switched off in our area. What can we do? If we were to supply the extra TV sets with freeview digiboxes, would they be able to receive the "terrestrial channels"? We don't want to pay more than one cable subscription or to install a Freeview aerial. Any suggestions welcome, or please move the question to another refdesk if more appropriate. Thanks. Itsmejudith (talk) 20:44, 22 October 2009 (UTC)[reply]

AFAIK, there has been a roll out of digital switch overs in the UK for some months now. Some of the channels you are listing presumably are only available as digital services. As a result, your media box will have to be replaced by the provider. I have no idea about the UK, but ours (in Vienna, Austria) was exchanged free of charge. If you want a box with harddrive and HDMI, you may have to pay some minimal surcharge (here it would be around 5 Euros). --Cookatoo.ergo.ZooM (talk) 21:06, 22 October 2009 (UTC)[reply]
No, all the channels the OP names are being broadcast digitally and are free. If you are in a good reception area and the digital broadcasts are coming from the same transmitter site (likely) then your existing aerial may be good enough. However, in many cases it is necessary to upgrade the aerial to get satisfactory reception. You may also find that whatever distribution you are currently using (aerial splitters etc) to get the aerial signal to all your TVs is no longer adequate for digital and will also need to be upgraded. You lose nothing by trying it, you will need the digiboxes anyway (assuming you don't go with the cable solution). If it doesn't work with just the digiboxes then call in an aerial specialist. SpinningSpark 02:08, 23 October 2009 (UTC)[reply]
On re-reading your question you seem to be implying that you do not have an aerial at all, in which case you are relying on your cable company to supply the analog signal through the cable. This is entirely down to them whether they continue to do this or not, you will have to ask them. Freeview digiboxes will not work with the cable companies digital signal. SpinningSpark 02:27, 23 October 2009 (UTC)[reply]
That's right, we don't have an aerial; it would be awkward to fix and reception would be uncertain. You've answered my question, thanks very much, although it was not the answer I was hoping to hear. Itsmejudith (talk) 08:52, 23 October 2009 (UTC)[reply]

temperature

I noticed that one can take a person's temperature from the mouth, under the armpit, the ear, and the rear. What is the difference in temperatures that one would get from those methods if you assume that all temperatures were taken from the same person simultaneously? Googlemeister (talk) 21:14, 22 October 2009 (UTC)[reply]

See Body_temperature#Methods_of_measurement. --Tango (talk) 21:15, 22 October 2009 (UTC)[reply]
That only gives a typical reading. Most people getting their temperature taken are getting it done because it is assumed that they are of abnormal temperature. Is the relationship linear? Googlemeister (talk) 21:21, 22 October 2009 (UTC)[reply]
If the quesion is "is the temperature far from normal", all you don't have to convert to a standard "core temp" scale, you just need to know what is "normal for how you're measuring it". However, the section does also provide some important info about general trends. I don't know if it's literally linear (degree-for-degree), and that is an interesting issue, but "lower temps as you get further removed from the core" seems like a sensible pattern. The section also tells about some serious confounding variables as you get further removed from the core. Especially if the body is trying to change its temperature, those could make the whole idea of a clear relationship a hopeless idea. DMacks (talk) 21:28, 22 October 2009 (UTC)[reply]
Fever#Measurement_and_normal_variation might also be useful, then. We don't seem to have similar information for other types of abnormal temperature. --Tango (talk) 23:24, 22 October 2009 (UTC)[reply]

October 23

On the source of the sound of a fart.

What is the principal cause of the characteristic sound of a typical rip? Is it the flapping of the butt cheeks together or the fluttering of the sphincter muscle? If the latter, what frequencies are we talking here on average? 71.161.59.133 (talk) 00:18, 23 October 2009 (UTC)[reply]

Did you consider checking flatulence for the cause of the sound? As for the frequencies, there must be a wide range or we couldn't have flatulists making money off it. -- kainaw 02:57, 23 October 2009 (UTC)[reply]
You can contact Mr. Methane at his website. Cuddlyable3 (talk) 14:49, 23 October 2009 (UTC)[reply]

ketoconazole/ pets

in doing a check of medicines that were perscribed to my dog(ketoconazole, 200mg capsules) i was unable to see if any precautions exist that apply to animals that might not exist for humans reguarding the use in animals. are there any?24.7.248.73 (talk) 01:56, 23 October 2009 (UTC)[reply]

Wikipedia doesn't give medical advice, and we don't have any more expertise in veterinary medicine. If you have concerns about the appropriateness of the medication, I strongly suggest consulting a vet or other appropriate professional. Warofdreams talk 11:23, 23 October 2009 (UTC)[reply]
Its not clear what you are asking. If you want to know about the published side effects of ketoconazole in animals, see [12]. If you want to know what effects it could have on you as you give it to your animals, then as Warofdreams states, we are unable to help. Rockpocket 18:13, 23 October 2009 (UTC)[reply]

Dry ice advice.

My teenage son is putting together his Halloween costume (he's going as a Borg) - complete with laser-pointer eye-gizmo, LED's on bits of fake circuitboard, mechanical 'claw', heavy face make-up, etc. Since we know lasers don't really shine those cool beams out there - I suggested trying to rig up a dry ice 'smoke' generator to feed a mist out of a thin hose to make the laser visible. So the approximate idea is to take a large coke bottle - built into some kind of back-pack - fill part-way with water & dry ice - run a thin pipe (maybe the ~5mm stuff they use in aquariums) out through the head-gear to squirt a flow of white mist out in front of the costume. We should take it as read that this would be unutterably cool. Sadly, I don't have much practical experience with dry ice. We're going to need to experiment with the design - but it would be good to get some solid suggestions. Hence:

  1. Can you generate enough mist from a 3 liter coke bottle to last an hour or so?
  2. Will the mist travel up a couple of feet of plastic hose without recondensing?
  3. What kind of flow rate would we get? (if there would be enough - we'd like to route several hoses out so that other parts of the costume emit 'smoke' - perhaps turning it on and off as he bends his elbows by kinking the hoses).
  4. Would the mist be ridiculously cold - maybe dangerous to his face? (seems unlikely - and we can maybe build some protective insulation into the gizmo that fits over one eye and that half of his face).
  5. Do we have to worry about it exploding or something ridiculous like that? (I can't imagine it would given there is a 5mm hole in the cap and plastic coke bottles can take a fair amount of pressure).
  6. What's the optimum ratio of water to dry ice?
  7. Someone told me you need warm water to avoid a coating of water ice forming around the dry ice and stopping it from doing it's thing. True or false? (He could, for example take along another tank of hot water to top up the dry ice tank once in a while)

OK! Go!!! SteveBaker (talk) 02:43, 23 October 2009 (UTC)[reply]

IIn college I remember trying to completely enclose dry-ice nuggets in water-ice, but could never accomplish it fully. As the water freezes and gradually covers the dry-ice surface, the area from which sublimated CO2 can escape gets smaller but there's still heat being absorbed (even though water-ice is something of an insulator). That makes it progressively harder to narrow that gap (gas flow and bubbling prevent liquid water from freezing in place). If it did manage to completely enclose, it would surely burst because the capsule can't prevent heat from being absorbed and therefore CO2 gas pressure from building up.
Thinking of which, you're throwing -78°C water vapor through a small hole...if it ever clogs, you can get an ice plug and a nice dry ice bomb. Wouldn't be likely to happen (in my mind, see above) unless the gas flow stopped long enough for ice to form but WP:RD/S/NO_MEDICAL_ADVICE. Warm water probably works better (more visible smoking) because it gives greater water-vapor in the air (smoke is condensed water vapor) and because it causes faster sublimation (greater cold-gas flow). After a while, the water cools and will eventually slush and then freeze.
Maybe you could just hope it rains a little?
OTOH, the whole idea of a laser pointer being directed wherever he faces seems ungood from the "every time you as someone's face, you're shining a laser into his eyes" perspective. DMacks (talk) 07:08, 23 October 2009 (UTC)[reply]
I assume you have looked at Smoke machines? --BozMo talk 14:23, 23 October 2009 (UTC)[reply]
I prefer a low-pressure system using, say, a battery powered hand vacuum cleaner reversed to blow the fog. "Putting dry ice in a bottle and closing it is kinda illegal" according to Dan[13]. Other useful information at that site[14] includes a Japanese video of a Borg secret weapon. Cuddlyable3 (talk) 14:44, 23 October 2009 (UTC)[reply]
Its a very cool idea, but my gut tells me that it may not be practical for what you are trying to achieve. The size of the reservoir you may need to get a strong enough flow for the effect you are looking for may be too large for a little guy to carry on his back. I'm not sure 3 litres will do it, but I could be wrong and it would certainly be worth a try.
  1. Can you generate enough mist from a 3 liter coke bottle to last an hour or so? You would need to top it up a few times with both dry ice and perhaps water. It would probably be a good idea to have a fresh bottle too, as the first one is going to get very cold.
  2. What kind of flow rate would we get? (if there would be enough - we'd like to route several hoses out so that other parts of the costume emit 'smoke' - perhaps turning it on and off as he bends his elbows by kinking the hoses). This is going to depend in part on the dry ice to water ratio. I fear it would be a pretty weak flow rate. Also remember the CO2 mist is heavier than air, so it will tend to settle rather than rise up. A hose outlet lower than the reservoir would assist flow rate.
  3. Would the mist be ridiculously cold - maybe dangerous to his face? The mist will not be too cold, though the reservoir will be very cold and will need to be insulated. The bigger concern is breathing in the CO2. A deep breath of dry ice mist is rather painful (I speak from experience) as it activates the trigeminal neurons in the nose. You need to ensure your child is not breathing it in.
  4. Do we have to worry about it exploding or something ridiculous like that? So long as there is a hole in the bottle you should be fine. You don't want to block the pipe and (obviously) do not use a glass bottle.
  5. What's the optimum ratio of water to dry ice? Trial and error. Too little and you will not get enough mist, too much and you will freeze your water.
  6. Someone told me you need warm water to avoid a coating of water ice forming around the dry ice and stopping it from doing it's thing. True or false? True, when the water begins to freeze, the CO2 production slows. Warm water will help, but it cools down pretty quickly in the presence of dry ice
Let us know how it goes! Rockpocket 18:07, 23 October 2009 (UTC)[reply]

Brain Fluid Behavior?

This technically qualifies as biology, so I will post it here if you do not mind. When I move my head rapidly, I occasionally get a pain on my right side. I was once hit was a dodgeball on the left side of my head, which oddly enough left the left side of my head sore, and I'm not sure if this is related, but it sure is troublesome. The pain is sharp but fades quickly. Note I am not asking for medical advice, only some guesses as to what this may be or may be related to so I can do more research. Thank you. —Preceding unsigned comment added by 69.232.218.255 (talk) 06:23, 23 October 2009 (UTC)[reply]

By asking for "guesses as to what this may be" you are asking for a diagnosis, which does fall within our definition of medical advice (see Wikipedia:Reference desk/Guidelines/Medical advice). The only reliable research that you can do is to consult a qualified medical practitioner. Gandalf61 (talk) 09:37, 23 October 2009 (UTC)[reply]
Really, truly, go and talk to a doctor and tell them everything you said here, and anything else that seems relevant. They will know what other questions to ask, what else to look for, etc. People here on the internet can lead you to frightening ideas or falsely reassuring ideas, either of which is harmful. Ask a medical professional. 86.142.231.199 (talk) 15:56, 23 October 2009 (UTC)[reply]

cdna

How can we identify and isolate tissue specific complimentary dna? —Preceding unsigned comment added by Dinesh11bajaj (talkcontribs) 06:47, 23 October 2009 (UTC)[reply]

To identify if it's present in a particular tissue? That's easy to describe. If you wanted to determine if tissue X has specific gene Y, you could utilize a fluorescent DNA probe to a sample of the tissue and provide conditions for it to hybridize (allow the DNA to unzip and then re-zip in the presence of the probe (probably can all be done in a PCR-type machine) and then perform a Southern blot to streak the sample and check for fluorescence, thereby indicating successful complimentation with the probe and thus presence of the probed DNA in your sample. To isolate it, I'm no molecular biologist, so I'll leave that to someone else. DRosenbach (Talk | Contribs) 13:14, 23 October 2009 (UTC)[reply]
A southern blot is usually used to detect genomic DNA, not cDNA. A tissue northern blot will tell you if the RNA transcribed from a given gene is present, which is what I think the OP is after. Or, you could use RT-PCR as described by Rockpocket below. Technically, our tissues only have genomic DNA or different types of RNA, not complimentary DNA (which is an artificial copy of RNA made using reverse transcription). --- Medical geneticist (talk) 19:22, 23 October 2009 (UTC)[reply]
Dissect out your tissue of interest from your organism of interest, then do an RNA extraction (either following a protocol like the one described here or using a manufactured kit like Qiagen's RNAeasy to extract mRNA). Then do a cDNA first strand synthesis reaction using a reverse transcriptase enzyme (for example, SuperScript) and a Poly-dT primer, to ensure you only make coding DNA. This should give you what you need. Rockpocket 17:42, 23 October 2009 (UTC)[reply]

Change in Velocity without Acceleration

Imagine we're floating in space with no stars or anything else for reference. I'm going to be talking about my velocity, and since you're the only reference I have, I'm going to be measuring it releative to you.

Experiment One

I turn on my jet-pack. I note the distance between us is changing and conclude that my velocity has become non-zero.

Experiment two

You turn on your jet pack. I note the distance between us is changing and conclude that my velocity has become non-zero.

Question

In both experiments, I (correctly) conclude that my velocity has become non-zero, but in the second one this managed to happen without me having a force applied and without feeling any acceleration. Without reference to absolute space or Machian frames of reference, what's going on here?

82.64.139.77 (talk) 08:45, 23 October 2009 (UTC)[reply]


You assume the other guy is at rest. When the other guy turns his jet-pack on, he suddenly feels acceleration, which is indistinguishable from gravity by the equivalence principle. The other guy concludes that the rest of the universe -including you- is being accelerated by this gravity. But you don't feel gravity-caused "free-fall acceleration". —Preceding unsigned comment added by 157.193.173.205 (talk) 09:11, 23 October 2009 (UTC)[reply]

Huh? That doesn't make sense. When someone turns their jet pack on, their conclusion is not that the rest of the universe is being accelerated by gravity. Indeed, it isn't necessary to talk about gravity at all in this problem. The spacetime involved is flat, so gravity here is irrelevant and just confuses the matter.
What's going on is that although all inertial frames of reference are indistinguishable, an inertial frame of reference is distinguishable from a noninertial frame of reference. Proper acceleration can be felt and measured, without any reference to distant objects. So what's really going on is:
Experiment one
We choose an inertial frame of reference, call it A, in which you and I are at rest. You turn on your jet pack, and feel an acceleration, so you conclude that your velocity has become non-zero with respect to inertial frame of reference A. After you've finished firing your jet pack, you may find it convenient to define a different inertial frame of reference B, in which you are then at rest. But you know that frame of reference B is not the same frame of reference as A. You and I agree that it is you that has undergone a (proper) acceleration. If you want, you can also define a coordinate system C in which you are always at the origin, even while you are firing your jet pack. You can define C such that it coincides with A before you fire your jet pack, and coincides with B after you are done firing your jet pack. But coordinate system C is not an inertial frame of reference. At any point in time, there is an inertial frame of frame of reference that's tangent to C, but C taken as a whole is not an inertial frame of reference.
Experiment two
We choose an inertial frame of reference, call it A, in which you and I are at rest. I turn on my jet pack. You, however, do not feel an acceleration, so you conclude that your velocity remains zero with respect to frame of reference A, regardless of the fact that I am moving away from you. You conclude that it is me that has gain a non-zero velocity with respect to frame of reference A. After I'm done finishing firing my jet pack, you may find it convenient to define a different inertial frame of reference D, which is co-moving with me, i.e., an inertial frame of reference in which I am at rest. But you know that haven't accelerated, and that you are still at the same location in frame of reference A. You and I agree that is is me that has undergone an acceleration. Red Act (talk) 10:33, 23 October 2009 (UTC)[reply]
The only points of reference are you and me, so when you choose A in which both you are I are at rest, aren't you making an implicit reference to "absolute space"? 82.64.139.77 (talk) 12:09, 23 October 2009 (UTC)[reply]
No. Purely for convenience in analyzing the situation, A was chosen such that you and I aren't initially moving relative to it, but no statement was made that A itself isn't moving. There are other, equally valid inertial frames of reference, relative to which A is moving. There's no absolute space involved, just differing, equally valid, inertial frames of reference. You could just as validly look at the problem using a different inertial frame of reference, in which both people initially move at the same constant velocity before the jet pack is used. But regardless of which inertial frame of reference is used, the person who uses the jet pack is observed to undergo a non-zero acceleration, and the person who does not use the jet pack is observed to travel at some constant velocity (possibly zero). It's just a little more convenient to choose to analyze the situation in an inertial frame of reference in which some of the velocities involved are zero. Red Act (talk) 12:51, 23 October 2009 (UTC)[reply]

In this case, I (as the "observer") have chosen you as the coordinate system I'm going to be talking about. In that coordinate system, if I turn on my jetpack, I feel an acceleration and I see the distance from me to you is changing - so I know that I'm moving relative to you. When you turn on your jet pack, the distance increases - but I don't feel an acceleration. So my only conclusion can be that our mutual frame of reference accelerated. That makes it non-inertial reference frame and the laws of special relativity don't discuss that situation. To make sense of this, you have to move to general relativity in which acceleration and gravity are equivalent. When I turn on my jet pack, it's no different than before, I feel the acceleration and I see the distance increasing. When you turn on your jet pack, it's as if gravity just turned on in our little universe. If you start getting closer to me - that's equivalent to me falling towards you in free-fall. I don't feel any acceleration (just as I don't if I fall off a tall building). The "force" that's making me fall towards you is entirely equivalent to gravity! If I decide I don't want to get any closer to you - I can fire my jet-pack to counteract gravity and "hover" relative to you. But I have to expend energy to stay still just so long as you are firing your jet pack to provide that "gravity". If this seems peculiar, it's only because I made such a peculiar choice of coordinate system - but the universe doesn't care what coordinate system I choose! SteveBaker (talk) 12:49, 23 October 2009 (UTC)[reply]

Mentioning gravity is not only confusing the issue here, it's incorrect. There is no gravity involved here, because the spacetime involved is flat, not curved. The acceleration of the person who uses the jet pack is not the same as if gravity was "turned on" in such a way that it only pulls that one person. The person using the jet pack undergoes a proper acceleration, which is physically very different from the mere coordinate acceleration that gravity is equivalent to. A person undergoing a coordinate "acceleration" due to gravity of 1000m/s2 can close their eyes, and they won't notice any difference than if they were just floating in space without that "acceleration". A person undergoing a proper acceleration of 1000m/s2, due to a jet pack that's way too powerful, will get squished into a puddle of goo inside their space suit. The person not using their jet pack can also observe the difference, by using a telescope, and observing if the other person's helmet contains a face, or a red puddle of goo. Red Act (talk) 13:46, 23 October 2009 (UTC)[reply]
Less humorously, the person who doesn't use the jet pack will also observe that the center of mass of the person who uses the jet pack, plus the trail of propellant coming out of the jet pack, remains a fixed distance away. Under a gravitational acceleration, that center of mass would not appear to remain a fixed distance away. Red Act (talk) 14:03, 23 October 2009 (UTC)[reply]
To reiterate the point above: Steve, you said "That makes it non-inertial reference frame and the laws of special relativity don't discuss that situation." That's inacurate. Special relativity can deal with non-inertial frames without any problems. See Rindler coordinates. Dauto (talk) 14:19, 23 October 2009 (UTC)[reply]
The jetpack is not required to provide gravity. Gravity is inherent to that non-inertial frame of reference. The jetpack just makes the astronaut using it hover at the origin, rather than falling with the other astronaut. — DanielLC 15:03, 23 October 2009 (UTC)[reply]
Again, there is no gravity in this problem. The acceleration that gravity appears equivalent to at the surface of an object like the Earth is the purely coordinate acceleration of an object that has no proper acceleration, as viewed in an accelerating frame of reference. However, it isn't true that all acceleration observed in an accelerating frame of reference is gravity. Instead, some of that acceleration can be proper acceleration, which is due to forces other than gravity. (Gravity isn't actually a force; it's a pseudo force. Proper acceleration is the acceleration of an object due to real forces.)
The rule is "No curvature, no gravity!" There is no curvature of the spacetime in this problem, so there is no gravity. Regardless of whether you use an inertial frame of reference or an accelerating frame of reference in this problem, the Riemann curvature tensor will be zero everywhere, so the Einstein tensor will be zero everywhere, and the Einstein field equations simplify down to 0=0. (The mass of the objects involved here has a negligible effect on the curvature, and so is taken to be zero.) In other words, in this problem, even if you use an accelerating frame of reference, the Einstein field equations that describe gravity say nothing at all about the observed acceleration. That's because the acceleration involved is not caused by gravity. Red Act (talk) 17:13, 23 October 2009 (UTC)[reply]

electroaromatic substitution in polyaromatic rings

How do the principles change? If I nitrated napthalene, would it still be meta-directing...? John Riemann Soong (talk) 09:55, 23 October 2009 (UTC)[reply]

Directing patterns around substituted benzene derive from the way the directing group affects the electron density patterns in the conjugated pi-system. For polycyclic aromatics, you'd need to figure out how a functional group would affect that electron density pattern in the conjugated pi-system. I suspect the effect to be similar for Naphthalene, but you'd have to consider that unsubstituted naphthalene has 2 locations availible for substitution (due to symmetry) and once substituted, the resulting molecule now has like 6 non-equivalent locations for the next functional group, so terms like "meta-directing" have little meaning. --Jayron32 12:41, 23 October 2009 (UTC)[reply]

help candles??

sir, in my b tech project i m doing a image processing analysis of candles.images are taken under different circumstances.I have processed these images in matlab to get a intensity vs time plot and amplitude vs frequency plot.I can not further more process the results so as to be able to give results with respect to physical significance.

as i was searching over net i found bouancy induced flames are used as model for studing nuclear reactions in supernova(celestial).can i apply my model there.

please suggest

yours sci-hunter 218.248.11.210 (talk) —Preceding undated comment added 11:30, 23 October 2009 (UTC).[reply]

A candle flame is shaped by convection and turbulence in the surrounding air and limited by the flows of liquid wax and oxygen to the wick. None of those factors are found in a supernova. Cuddlyable3 (talk) 13:20, 23 October 2009 (UTC)[reply]
Strictly speaking not turbulence for a candle, the Reynolds number is too low. Buoyance/convection is right though. I don't know of any real link to supernova dynamics but they could be used as an illustration for students I guess. --BozMo talk 14:18, 23 October 2009 (UTC)[reply]


regarding my project,can the fft plots be utilised in some way.indeed i dont know any use of a fft plot please help. sci- hunter 220.225.98.251 (talk) —Preceding undated comment added 08:48, 24 October 2009 (UTC).[reply]

One use of a FFT (Fast Fourier Transform) of data is that one can apply filtering in the frequency domain, such as reducing or boosting certain frequency ranges, then convert back to the original domain via an IFFT (Inverse FFT). Cuddlyable3 (talk) 23:30, 24 October 2009 (UTC)[reply]

Cosmological Constant in Newtonian Gravitation

Is there Cosmological Constant in Newtonian Gravitation? Even if there is then does it even have an effect on gravitation? The Successor of Physics 11:58, 23 October 2009 (UTC)[reply]

No, there is no cosmological constant in Newtonian gravity. The cosmological constant is only a feature of Einstein's model of gravity, general relativity. At any rate, the cosmological constant does not have any appreciable effect on gravitationally bound systems. The cosmological constant only makes a difference at the largest, cosmological, scales. Red Act (talk) 12:18, 23 October 2009 (UTC)[reply]
Within Newtonian or classical mechanics the Gravitational constant in Newton's law of universal gravitation is an empirical physical constant. Cuddlyable3 (talk) 13:15, 23 October 2009 (UTC)[reply]
Yeah, but the gravitational constant is a completely different thing than the cosmological constant. Newtonian gravity only uses the gravitational constant; general relativity uses both constants. Red Act (talk) 14:09, 23 October 2009 (UTC)[reply]
Newton didn't use a cosmological constant, but you can add one to Newtonian gravity. It's just a matter of changing the gravitational Poisson's equation from ∇²Φ = 4πGρ to ∇²Φ = 4πGρ − Λ. That gives you a force of ma = FG + m (Λ/3) (xx0) where FG is the usual gravitational force and x0 is an arbitrary point in space. It looks like the effect depends on which point x0 you choose, but actually it doesn't, for reasons I explained in this old thread. It's easy to see that if the universe is filled uniformly with matter at a density of ρ, then taking Λ = 4πGρ will cancel out the gravitational attraction and prevent it from collapsing. Newton could have introduced the Λ term for the same reason Einstein originally did: to allow for a static universe. But it would have failed for the same reason Einstein's attempt failed: it's an unstable equilibrium. For a more modern example, you can derive a Newtonian version of the Friedmann equations and get a solution equivalent to the ΛCDM accelerating universe, as I showed in that old thread. -- BenRG (talk) 20:14, 23 October 2009 (UTC)[reply]
Thank you, BenRG!!!The Successor of Physics 08:52, 24 October 2009 (UTC)[reply]

what's the WORST thing in egg whites?

what's the worst thing in egg whites, since as far as I know "nothing". If you ate 90g of boiled egg whites every single day (ie the total protein requirement from ALL sources for a hefty guy) for 50 years, would there be any bad effect from that as compared with not doing that?

Note: I am not seeking or asking for medical advice, this is just a point of curiosity. —Preceding unsigned comment added by 85.181.150.192 (talk) 13:43, 23 October 2009 (UTC)[reply]

Egg white has protein, which as you rightfully say is edible. but it is mostly Albumin which (due to its hydrophobic core) collects hydrophobic molecules, which are molecules which are not that water-soluble, say hormones (animal and plant), antibiotics, pollutants, pesticides etc. obviously most of these would be at a level the body can easily handle them. Although hormones and antibiotics are what worries people the most as they are at a higher level. Secondarily, the aminoacid balance is a bit of and it may (very big may) give too much cysteine (kidney stone), which is why eggs smell of sulfur. Thridly, it is an animal product (4 chickens per 50 years), so that guy helped destroy this planet a bit quicker. --Squidonius (talk) 14:08, 23 October 2009 (UTC)[reply]
thanks! what would be an equivalent vegetable alternative, not including soy products? Rice and beans or lentils daily for 50 years (to the tune of 90g of protein per day, let's say it's a 200 pound man)? What would be the worst effect of that? note: still not asking for medical advice :) —Preceding unsigned comment added by 85.181.150.192 (talk) 14:10, 23 October 2009 (UTC)[reply]
(ec) Egg_white#Nutrition says that raw egg whites they bind biotin strongly and thus can create a biotin deficiency if you eat a lot of them over time. Which sounds bad. But cooking them gets rid of that. Mr.98 (talk) 14:12, 23 October 2009 (UTC)[reply]
Squidonius, about the destroying the planet part, could you clarify? These are unfertilized eggs I presume, so how does eating the whites use up 4 chickens over 50 years? And also, how does the deaths of four chickens over that period of time contribute to the destruction of earth? I guess you are probably joking, but I still don't get the 4 chickens part. —Akrabbimtalk 14:15, 23 October 2009 (UTC)[reply]
I think the argument is that you need to keep 4 chickens to deliver those 90gm/day - and if you do this for 50 years, you need to keep them for 50 years. Animals as a food source are highly uneconomical in terms of food calories produced per input. You can keep a lot more people fed if you feed the chicken feed to people without the intermediate agent, who wastes most of the input energy running around, keeping its body temperature elevated, clucking, and fluttering its wings. I don't have the numbers handy, but I would expect orders of magnitude loss in efficiency (and yes, I still eat plenty of meat and eggs despite knowing better ;-) --Stephan Schulz (talk) 17:25, 23 October 2009 (UTC)[reply]
But, this assumes that the food you are feeding to the chickens is suitable for feeding to humans, and that humans could extract the energy from the food more efficiently than the combined efficiencies of the chicken extracting the energy from the food and the human extracting energy from the chicken. This is not always the case. For example, if you live somewhere with scrubby hills, you can graze sheep on this land without costing you a lot of energy. You can't grow crops on it without investing a lot of energy on transforming the land into something suitable, and you can't get as much energy from the grass by eating it yourself as you can by letting the sheep eat it, then eating the sheep. To get the same amount of protein as in 100g of lamb, you'd need to eat about 300g of beans and grains (remembering to eat a mixture to get all your essential amino acids), which can only be grown on land suitable for these.
In order to know which is more efficient, you need to carry out a proper life cycle analysis for the options, taking into account the specific conditions in the specific area the food is raised. The answer for growing the food in Ohio might give a different answer to growing it in Wales, which might give a different answer to growing it in Canada. Eating just the egg whites is a rather inefficient way of extracting the calories, but I'm not sure that all conditions would give an orders of magnitude loss in efficiency. 86.153.168.52 (talk) 20:33, 23 October 2009 (UTC)[reply]
The rule of thumb is that each step in the food chain gets 10% of the energy of the previous step. So, for example, if you're eating lions, you've got (lion eats gazelle eats grass absorbs sunlight) = 0.1% of the available sunlight in the form of lion-derived calories. --Carnildo (talk) 22:53, 23 October 2009 (UTC)[reply]

Apollo photo number?

Hi, I have read in some blogs that this picture was taken by Apollo 11 before they landed on the Moon, possibly 10 to 15 miles (more than 60,000 feet) above the lunar surface. It was part of Dr. Ken Johnston's (which he has now given some of his photos to Moon researchers) personal (former Directer of the Photographic Department of NASA, and trainer of astronauts, of which he has other degrees in science) collection that he had from NASA, and still has the better quality original versions of other pictures. Therefore, one does not need to be an expert, but does anyone know the frame number of this photograph from more than 40 years ago? If anyone can get someone interested in the Moon for help, it would be really appreciated. Note, the link does work, when you go here; click twice in where the link is written, then press enter for it to work. The picture is half over exposed, but you can still see the image, the other tiny white specks are the photographic defects and some stars, but something else that should not be there is in the picture. This is the picture, it's called the "Tower." Thank you for reading.--24.23.160.233 (talk) 14:00, 23 October 2009 (UTC)[reply]

All photos taken during the Apollo 11 mission can be found here at NASA's official site at the National Space Science Data Center. You might want to track down whether this photograph was actually taken on Apollo 11 (or any other moon mission) - because if it was, it will be available in high resolution from NASA. I suspect the image you linked to is the result of many many stages of photocopying and digital post-processing - it's hardly suitable for meaningful scientific analysis in its present state. Nimur (talk) 14:20, 23 October 2009 (UTC)[reply]
Specifically, here are the Apollo 11 Lunar Surface Journal (and other associated non-ALSEP flight- and pre-flight) photographs. Nimur (talk) 14:30, 23 October 2009 (UTC)[reply]
The link doesn't work. --TammyMoet (talk) 15:10, 23 October 2009 (UTC)[reply]
It's important that you see the actual picture. I'm really sorry about the site, when you go there, and as you see the link at the top, click on the link twice (for example, as if your going to change the words there) but don't change the words, just press enter again. Another way to word this... So click on it twice, you know the place where you can see the link written at the top of the internet, and press enter, I guarantee the link will work then.--24.23.160.233 (talk) 15:30, 23 October 2009 (UTC)[reply]
I was able to see the photo and the OP's curious instructions are correct: Click his link and observe the 403 error, then click the URL in your browser, then hit Enter. The photo displayed, oddly. (Windows Vista and Firefox 3.0.13.) Now I'm more interested in why this occurred than the artifacts in the badly photocopied photo. Comet Tuttle (talk) 17:15, 23 October 2009 (UTC)[reply]
(Note - this is a complete guess, but one based on experience). I think the intention of the designer was to prevent the images being displayed other than from a page on the website. The first visit to the URL is aliased to a PHP script that checks for a valid cookie, doesn't find it, and therefore refuses to display the image. However, this script is sending out the very cookie it's checking for, so that, the second time you visit the URL, the system is happy. _Not_ the best of designs, and a brief survey of the rest of the site might lead one to the conclusion that the designer is not among the ranks of the most competent... Tevildo (talk) 18:10, 23 October 2009 (UTC)[reply]
Well that's great, I'm glad you guys were able to see it. From what I know, this photo was enhanced using photo shop, and anyone can do this (this does not mean adding or removing anything from the photograph), to bring out the all the details in the picture, as I'm guessing it was "lighted up," because the dark parts of the craters are gray in color. It was probably originally overexposed by too much light, then it was enhanced, and finally made into copies. As I said before you can see tiny scratches, specks, and stars in the picture (plus the overexposed part). Note, I just want to be clear I am not an advocate of www.enterprisemission.com, which 75% of the site is filled loony unscientific data, and I agree the original site designer was incompetent, as it access denied the link to people who were honestly looking for the truth. However, the "artifact" in the top center of the photograph (which can faintly be seen as rising from lunar surface) is 10 to 15 miles in height, and should not exist (emphasis added)...--24.23.160.233 (talk) 18:25, 23 October 2009 (UTC)[reply]
I'm still not really sure what you mean by "the artifact". Is it the very prominent trapezoidal shape in the center of the image? If so, that suggests to me that the image has been photocopied on a machine with a worn drum and/or dirty corona wires, as that's exactly the sort of effect such a device produces. If it's something more subtle, you'll need to identify it more precisely. Tevildo (talk) 18:55, 23 October 2009 (UTC)[reply]
All I can see is that increased light has been added to the image, as the shadows in the craters should be black, but are gray. True that it has been photocopied, with possibly a dirty corona wires which have produced the lines you see in the picture, but are you suggesting the trapezoidal shape is not a solid object? Careful analysis, and I have downloaded the image on my computer and played with it, shows that it is solid object, and that it touches the lunar surface, therefore it is connected to the ground, and not a floating blob. If it was in the original photo, could this be a natural formation, 10 to 15 miles averaged is just over 12 miles in height?--24.23.160.233 (talk) 19:20, 23 October 2009 (UTC)[reply]
To be clear - contrary to what our OP says - ANY form of "enhancement" whatever (with photo shop or any other image manipulation program) will "add or remove" something. Even loading up a JPEG file, cropping a couple of pixels off of one edge of it and saving it again will introduce artifacts and remove quality from the original. So before we go anywhere - we have to be 100% clear about that. I'm not going to do it again - but a few years ago on this desk, I showed how a speck of dust on an image could be turned into a very believable "flying saucer" by doing nothing more than "image enhancement" to the picture. An awful lot of UFO/conspiracy-theory/Moon-landing-hoax nut-jobs have fallen for this very thing. The trouble is that artifacts from camera flaws (lens flares, etc) - from the transmission technologies of the day (scan-line artifacts) - and from modern data compression systems (JPEG, GIF) - all share the property that they produce fairly sharp-edged geometrical-looking changes to the image at the lower end of the brightness scale. Newspaper and magazine prints add half-toning artifacts. Fax machines and printers make longitudinal streaks. Lens flares are often hexagonal or octagonal and add diagonal lines into the image. Scan line artifacts produce horizontal features and JPEG makes square or rectangular blocks and adds lots of weird colors into the image. Color banding and 'mach' banding is easy to introduce into an image by boosting the brightness and playing with color mapping curves.
When your budding Ufologist gets one of these images - the first thing he thinks to do to "enhance" it is to boost the gamma or the brightness to see what's "hidden" in the dark areas. Lo and behold, there is some kind of a dim, geometrical shape there! So he crops it down to size - maybe rotates it some to get a better 'appearance' - and posts it to all of the other nut jobs. Sadly, saving it that way adds more artifacts from image compression. These other guys get it - and they do the same thing - finding still more amazing details of this 'tower' or whatever it is. Recomposing the image makes noise that was parallel to the screen become diagonal. Repeat this process enough times and you can find amazing looking "structures"! The joke is that all of this "enhancement" has been degrading the image more and more - to the point where pretty much all you're looking at is garbage produced by the processing. The only sure way to know what's really going on is to go back to the most original, UN-enhanced image and look at that - do all of your enhancement from that original image and never, ever store it as a JPEG or a GIF. If the only source image you have is JPEG or GIF, or in a newspaper or magazine or a photograph taken from a video...then all bets are off. :::::::::::SteveBaker (talk) 19:20, 23 October 2009 (UTC)[reply]
Okay, hold on! I'm sorry but you must be tripping, looks like I hit a hot button, and that we got off on the wrong foot here. I never edited the image in the first place. I found the image on the site, and said that I edited the image later on my own computer and found it to some kind of object (because I and everyone here can already see the object without any enhancements). The picture you see on the site is the picture I saw when I first went there. And you suggest conspiracy on my part being person who follows a ufologist enhancing person (which I'm not) is out of line for a reference desk question, I suggest if you can tone it down a little buddy. Also the image has not drastically been made so every find detail has come out, all I can see is that one side is overexposed and the other side looks normal but sides of the photo suggest it is from a worn off picture. That's what causing the image noise, lines, white specks, and scratches. If all the detail was to come out, the hole image would have dots on it. All I'm saying is that it is highly unusual for an object or the trapezoid shape to only appear in one area (why did the astronauts taking the picture from their lunar module position the camera so that the [NONE-existent made by a loony] object should appear exactly in the center of the photograph?) that extends to the ground, and to appear where it does, and that's on the Moon, is even more baffling. If the original picture had the trapezoid tower on it, what can anyone say then?--24.23.160.233 (talk) 19:50, 23 October 2009 (UTC)[reply]
No, no, no! You misunderstand me. I'm not saying that YOU did this with inapropriate image enhancements. I'm saying that dozens of people before you did. The image you pointed us to is already screwed up beyond repair - it's essentially garbage. The only thing I'm disagreeing with you about is your (incorrect) statement that "[enhancement]...does not mean adding or removing anything from the photograph"". That's flat out not true. (I've been in the graphics/image processing business for 35 years - I know of what I speak!) I'm telling you - as clearly and simply as I can - that enhancement (especially done by amateurs) is more likely to turn subtle imaging artifacts into alien towers on the moon - than it is to show you anything useful. Unless you can get a hold of the original NASA image and know exactly how it got from the camera to your desk - and unless you know what you're doing - you are unable to do any kind of image analysis that has meaning because everything you (or anyone else) does to it can only add artifacts that look like geometric objects of one sort or another. Why is this artifact slap bang in the middle of the photo? Well - there are any number of possible explanations. Perhaps a previous amateur image enhancer cropped the image to place the object in the center for a more pleasing composition? Perhaps the astronauts didn't point their camera directly at some "object" - perhaps the thing your seeing is precisely in the center of the image because that was the only point in the image plane where the lens of the camera was perfectly parallel to the window of the spacecraft - so only then did you get a reflection of something inside the spacecraft showing up in the image. I don't know whether that's what happened - but it's all too plausible that some effect like that caused a very, VERY dim artifact in the original image - which has been "enhanced" (screwed up) to the point where it looks like something geometrical in the scene. The point is that you honestly don't know - and in the absence of that, you have to be skeptical of the wild-assed "hidden tower on the moon" hypotheses - and use Occam's razor to seek the simplest explanation...like a reflection of the astronaut himself in the window that's been over-enhanced. But there doesn't even have to be anything in the original photo at all. Multiple stages of inappropriate digital processing can pull junk out of literally nothing at all. 22:20, 23 October 2009 (UTC)

You may also want to know that I consider all the possibilities compare this link and this link. Plus it's critic link, although the person does say not all objects in SOHO can be explained, as I've seem some intriguing images myself that can't be explained. So please don't blame me for putting these here, I put them to show that I have an open mind and look at both sides of the story. I don't believe in everything I see or read, which is not even the case here. The image which I had the question about comes from Dr. Ken Johnston, who is a reliable person, read my first message. So it's best to contact him, though he has lend some of his images (that he originally got from NASA and had been hiding it for more than 40 years) to the guy who is loosing his mind day by day (Richard C. Hoagland), if the original image has the object in it, and it is not a product of Hoagland's photocopying, then we might have something here. Thanks.--24.23.160.233 (talk) 20:20, 23 October 2009 (UTC)[reply]

The picture is of very poor quality; it has probably been printed and scanned, perhaps several times. The "tower" looks to me like a patch where some old physical copy of the photo got scraped and damaged. But, yes, if an undamaged, closer-to-the-original copy could be located it would help to resolve this. —Steve Summit (talk) 22:10, 23 October 2009 (UTC)[reply]
OMG! There is a secret map of an alien city hidden in the period at the end of our OP's post! It's a Wikipedia conspiracy!!!
  1. Take a screen-shot of this very page as a nice, clean '.png' image.
  2. Save it as a JPEG image at poor-ish quality settings so you can post it on your overburdened, underfunded web site.
  3. Someone sees the image, zooms into it and notices "hidden stuff" - all sorts of cryptic and interesting patterns. Is it some kind of a code? I'd better post this on my web site!
  4. Someone sees that image, and notices a subtle yellow structure in the middle of the image - which they enhance with the gamma tool. HOLY CRAP!! There's a bunch of stuff in here. It looks like some kind of alien character set hidden in the message that was posted on Wikipedia! WTF is going on here?
  5. Someone else hears the breaking news and enhances the image with the 'edge' enhancement tool. There is something very weird about that full-stop. Let's zoom in for a closer look!
  6. That looks like a map. No! If you put it into perspective, you can see that shadows are being cast by the "buildings" around the edge of each city block. Maybe it's a secret city belonging to an advanced alien civilization or something?
  7. Discovery channel makes a two hour documentary about how the aliens are communicating with each other using Wikipedia's servers right under our very noses! Look - there are DOZENS of sites out there on the internet telling us about this and NOBODY in the Obama administration are doing anything about it. Jimbo Wales has denied it all! Cronies of his on the so-called "Science Reference Desk" are trying their best to suppress the story and claim that it's just a chance spark from an off-course weather balloon igniting marsh gas reflected in a layer of warm air. Are these mysterious patterns an attempt to indoctrinate humans? Perhaps they are subliminally teaching us the layout of their streets so we'll be better road cleaners when they enslave us all? Our "experts" have looked into other Wikipedia posts - and even some on government web sites, the department of homeland security and even reports denying the existence of Area 51 - and we've found exactly the same kinds of structures!
It's amazing what you can get out of a single pixel period at the end of a sentence...or from a photo of the moon.

SteveBaker (talk) 23:35, 23 October 2009 (UTC)[reply]

Well Summit, all I can say is that you have put a lot of effort into debunking this photo (or the possibility of what this could as suggested by me), and I can see why. I hope you checked out my links before posting your message, as I also included links to show how some people can mistake objects for UFO's in NASA pictures. Therefore, I actually appreciate that you have helped me understand how some pixels can be mistaken for objects. Though I could have just accepted your first paragraph. I don't believe in the invisible lunar glass domes on the Moon, only if they are solid objects appearing in the originals, then maybe I will pay attention to them. I also think that we should keep an open mind, since there are other photos that show similar objects without any enhancements. I could post them for fun later if you like just for you. At this time I wont, but like you said it would be great to have the original so we can see if the object was originally there. My original question sigh... Was what number is this photo so I would have been able to see it in the first place if the object that appears in the edited picture is actually there, I was trying to debunk this photo myself from the beginning, I think your sarcasm is understandable, but I would not be this sarcastic until I see all anomalies photos. I can see that you have had some experience dealing with crack-pots, and I assure you that I'm not one, I'm just an ordinary guy doing some research to do some of my own debunking. A friend had firstly told me of this image, and I looked for it and found it, I was not the explorer as you might think. I encourage you to publish the Wikipedia's Alien Conspiracy as a article in a comedy magazine, you know, so you can make some green off your imagination, which is good in these economic times. Best regards.--24.23.160.233 (talk) 01:55, 24 October 2009 (UTC)[reply]
I just came to a realization, I know that however old the picture is, a trapezoid shape cannot appear in empty space. If Hoax(g)land (just in case you don't who this guy is, he was responsible for making the Face on Mars popular) did do the same thing you did in the image you put in your last message then the whole part of the sky in the image should have specks of gray as they appear in the over exposed right side of the picture, not just one part! They key to finding out how much the original image was played with is looking at the craters, like I have said many times. In my own investigation on photo shop I blacked out (it's interesting to note NASA sometimes blacks out the sky in their Apollo images so much, sometimes the head of a hill is blacked out) the craters as they appear in all space images, and made the sky pitch black as if there was nothing there, but still outline of the object can be seen (the original probably had a black sky too, then ONLY when I adjusted the color temperature to the left or to blue, the object appeared out of nowhere in a pitch black outline - I encourage you to do this, but why does it only appear when adjusted to blue?), I had a scale which measured the histogram of the image (I don't know if this means anything or not, if you can tell me). So this means two things, either the object is really there, or Hoax(g)land purposely edited the image to make it appear as if something was there. There is no middle ground, either this was done on purpose-hoax, or such an object actually exists on the Moon. So what do you think? Any comments would be appreciated if they can shed more light on this subject. Thank you.--24.23.160.233 (talk) 02:15, 24 October 2009 (UTC)[reply]
(EC with below). Not necessarily; the shape could be an "artifact", see Artifact (error), of the original photograph. It could be a lighting effect; it could be some defect in the film, or any of a number of other explanations. I think that's what SteveBaker is trying to show you above; there is a middle ground between "really there" and "deliberate hoax", and that is "artifact of the photographic process which has been misinterpreted, either wishfully or willfully". --Jayron32 02:30, 24 October 2009 (UTC)[reply]
"I know that however old the picture is, a trapezoid shape cannot appear in empty space". Wow! You "know" a lot of things that aren't true don't you!
I think '24 really needs to carefully re-read everything that I and others have already written. I've explained this already. The example above was just one way that artifacts can come about. Here's another one - I just picked an image of the moon totally at random from WikiCommons...then I fiddled with the gamma correction - nothing more than that - and...viola! Alien Rocketships shooting across the lunar surface where previously there was only "empty space". You can produce these kinds of artifacts from almost any image that's been messed with, JPEG-compressed or whatever. It proves NOTHING - except that most non-experts don't know what the heck they are doing when they play with image enhancement tools in an effort to find evidence of conspiracy theories, coverups, etc. SteveBaker (talk) 05:15, 24 October 2009 (UTC)[reply]
Small point of order: Steve Baker put a lot of effort into debunking the photo. Steve Summit just took a look at it and offered his opinion. —Steve Summit (talk) 02:25, 24 October 2009 (UTC)[reply]
It wasn't really a LOT of effort - part of the problem here is that these tools are so easy to mis-use! :-) SteveBaker (talk) 05:15, 24 October 2009 (UTC)[reply]
First of all, I want to thank all of you for almost debunking this photo, as you have put a lot of work into it, and therefore I dearly appreciate the effort. Seeing that the only way to find if the object is real, is to have the original, I might need to contact Ken Johnston himself, so we can forget about this picture for now. However, the reason I got into this is because of another photo, which this time its not edited, just enlarged, which actually you don't need to enlarge it anyways, here it is, it is possibly the best of all. I don't want to sound weird, but the image itself can only be described as that.
The image is from Lunar Orbiter 3, taken in February 1967, frame number 84MIII (LO-III-84H). Here is a list of links, which are mostly from reliable sources that have the enlarged versions (of which only 20% have been enhanced) of the image which contains object(s) in question. Some of these sites may contain conspiratorial information, which I myself am skeptical about, so please ignore that. This has nothing to do with transient lunar phenomenon or fake Moon landing theories (to be clear, the first is mostly natural, and I believe we have gone to the Moon), but this has to do with photographic evidence of something on the Moon. I always apply the scientific method to subjects of this manner. Just to know, I am a person of college level, not perfect, but other than that I prefer to remain an unknown user.
The pictures are not fake, this is the first fact. There are others that I might show later on, but I decided to start with the The Shard. After appearing on two news sites, two works of art, and many mystery sites on the net, I think this is worthy of its own article, or its up to you, we can add it as a section to the Lunar Orbiter 3 article. Nothing has been added to the original image, only a tiny portion has been enlarged and enhancement to show the object(s). The object(s) are connected to the ground and have shadows (they appear in 5 different photographs from other missions), and the other tiny white cross marks, lines, and specks are the photographic defects. A NASA scientist has also quoted on the object(s) (I remember this quote, but have to find it). He was basically saying that you would need water or wind erosion to create it in millions of years (which there is supposedly none on the Moon), it may be referring to another object, but it goes something like this, "No geological phenomenon can explain this object" (the average height of both object(s) is 4 miles, or the size of the Himalayas).
Also the appearance of the object suggests it is a crystalline glass structure (I'm just theorizing here, so you pay attention to that part), also (note, if artificial, it would take hundreds of years to build, and glass weighs 10 times more than steel on the Moon, but I'm not sure about the latter) it looks heavily eroded. This and many other details appear in the links given below. There are 15 links, and I would like to apologize if they take too much time to look at, so thank you for your time. Therefore, I really appreciated and thank you for reading this message. Please, if you can just respond on your own talk page, that would be great, so best of regards. Please, I encourage you to look at these as if your looking at pretty space images, I'm not here to prove or disprove anything, just want to know what the experts think of (the Easter Island statue on the Moon, LOL!) it;
From the biblio-apocalypse site I got this, this is what it say's on the site, if you can confirm this that would be great; These tales are far away from the much more mundane conquest of the Moon. Nevertheless, some mystery surrounding the Moon has always intrigued scientists. The front page of the November 2 1966 edition of The Washington Post read: “Six Mysterious Statuesque Shadows Photographed on the Moon by Orbiter”. The Lunar Orbiter 2 had photographed a lunar area of approximately 30 by 50 kilometers. The photo apparently showed six or seven towers, appearing in a specific geometric pattern, rising from the Mare Tranquilis. Their pointed shadow indicated that they were either conical or pyramid-shaped. (image left) One of the towers measured an impressive 213 meters. NASA countered that the photographs did not show anything of any interest… whatsoever. Perhaps in an effort to merely embarrass the Americans, the Russian magazine Argosy offered the opinion of the Russian space scientist Alexander Abromov. He stated that the Russian Luna 9 had, on landing on the Moon on February 4 1966, taken some bizarre photographs: structures that stood in the landscape in a certain pattern. “The location of these lunar objects is comparable to the location of the pyramids at Giza. The tops of the towers show the same pattern as the tops of the pyramids.” One decade afterward's, in 1976, George Leonard published Somebody Else is on our Moon. Leonard stated he done extensive research in NASA’s archives and had found several photographs, including some of the first, unmanned mission to the Moon. Leonard’s effort was followed by Fred Steckling, who wrote We Discovered alien Bases on the Moon in 1981. It was an analysis of 125 photographs, on which Steckling pointed out “evidence” of buildings and other constructions on the surface of the Moon. Major parts of this publication, privately published, were later reused by David Hatcher Childress in his Extraterrestrial Archaeology. Indeed, many of the photographs that were used did seem to indicate anomalies that apparently did not belong on the surface of our Moon. In the late 1980s, Leonard’s research was handed over to James Sylvan, who reanalyzed Leonard’s material. Sylvan then handed his material over to Richard Hoagland, who had been writing about the strange objects that were visible in photographs of the planet Mars. Hoagland and co. used “fractal imaging” to analyze the photographs and identified the various anomalous structures as “the shard”, “the tower” and “the cube”. Specific attention was given to the Ukert crater, a crater which is the closest part of the Moon to Earth. Hoagland’s contact with geologist Dr. Bruce Cornet resulted in the observation that the crater apparently contained a triangle. Cornet confirmed that this could not be a natural event, but was proof of an artificial origin. Cornet also stated that the structure labeled “the Shard”, visible on a photograph of the Lunar Orbiter III in 1967, was the best available evidence that there were enigmatic – artificial – structures on the surface of our Moon. The Shard was apparently more than 1.5 kilometers high. He stated that if it was natural, it would be the miracle of the universe, defying all known patterns of erosion. But the Shard was apparently topped by “the Tower”, which rose no less than five miles above the surface of the Moon. The Shard, believed by some to be the best evidence of artificial structures on the Moon. Such massive construction projects were possible because of the lower gravitational pull that existed on the surface of the Moon. Hoagland and team stated that the Tower had been seen by Armstrong and team… and had even been filmed by them. All these structures were apparently made from glass. Though fragile on Earth, in the void environment of the Moon, glass would achieve the same rigidity as steel is known to have on Earth. What can we make of this?

Just tell me what you think. It is okay if you say that the square Tower blob is just a piece of floating jelly, I'll believe that. And that the Shard is part of many ordinary peoples imaginations of which it is a photographic defect that casts a 3 mile shadow on the lunar surface (without enhancements or enlargements as found in the first link), and that it appears in 5 different photographs from different missions to the Moon. It is probably the only unexplainable photograph, I just type in "the moon shard" in Google images and walla! Thank you.--24.23.160.233 (talk) 05:55, 24 October 2009 (UTC)[reply]

I'm not allowed to tell you what I think - to do so would be a gross violation of our "No Personal Attacks" rule. We've answered your question comprehensively - with demonstrations provided that clearly demonstrate the principles behind these bogus claims. I've been in the digital imagery business for an awful lot of years and I've looked into a bunch of these things - they are ALL bogus. Every single one of them - no exceptions. If you persist in believing this pile of crap - there is nothing we can do about it. The truth - boring though it is - is that all of these bizzare features are very simple image processing artifacts and when you go back to the original source material, that's easy to prove. There are a lot of idiots in the world who don't understand this simple fact - and sadly they make a lot of noise about it. Please - don't be another one of them. SteveBaker (talk) 14:40, 24 October 2009 (UTC)[reply]
If it really were true that unexplainable geological features/structures built by extraterrestrials had been observed on the moon, astronomers would be competing to be the first to characterize them. Suggesting otherwise, would be to subscribe to a conspiracy theory. Extraordinary claims require extraordinary evidence. I find no extraordinary evidence in the images you linked to. Calling the dark area to the right of the blob the blob's "shadow" is a human interpretation, not a fact. --NorwegianBlue talk 15:35, 24 October 2009 (UTC)[reply]
A; most of the general public does not know that these photographs exist (therefore they will not look for it with their telescopes, its too small anyways). B; not every image of a tower on the Moon is a photographic defect (especially the shard as you can see it with its shadow WITH NO enlargements-I did not come up with that originally). C; its very possible that you did not look at all the photos and text I provided, plus its pretty ignorant of you to say that all these claims are bogus, forget about the claims, what about the photograph's themselves, the source of the material can't be questioned, unless you think NASA planted the tower in the picture? Especially as we also have dialogue between mission control and the astronauts from most of the Apollo, and later space missions, even above Earth. With pictures and video, which the astronauts themselves said they saw UFO's and weird formations on the Moon. But you can forget about that, because that is a whole other subject with enough material to fill a hole article with, which I will not go to at this present time. What I don't understand is why every geologic anomaly on the Moon is a defect, it could just be a natural formation. I don't care if you have a million years experience in image analysis (extraordinary claims of experience need extraordinary evidence), at least 5-10 other scientists, 2 from the space programs themselves have commented on the shard, and they say its there. I later proposed we include the shard in the Lunar Orbiter 3 article, just like the Face on Mars is in the Viking 1976 article, there your welcomed to debunk and bash the photograph as much as you want. But its important to have it in a article so more people can investigate it, without more investigation there is no way to prove or disprove that the shard exists. Here is list of links that are from online articles to books of which they range from really reliable to barely reliable sources, what you might find interesting is that the last link is Italian but with pictures pinpoints the Shards location using newer photographs that show there is some kind of hill there (the pictures where taken right above the object, so we now have new evidence that something is there), plus I am concerned about the moon hoax section below this message, I hope this is not an attempt to derive attention from this to stupid moon landing hoax theories. Also, obviously Hoagland's conspiracy theories are wrong about NASA, and only half of the photographs he claims there are structures in is worth looking at, this is the only thing that you and me disagree on, you believe that all are bogus, while I believe a small percentage shows something-and therefore I have found support from scientists to crazy researchers of what I say, but I'm not claiming I'm 100% true, I'm simply saying more needs to known so we can be sure above these subjects; [15], [16],[17],[18], [19], [20], [21], [22], [23], [24], [25], [26], [27]. Video links, you will like these; [28], [29], [30], [31], [32]. Your right, there is no evidence huh?--24.23.160.233 (talk) 04:20, 25 October 2009 (UTC)[reply]
Before you start trying to put these ridiculous pictures into articles - I strongly suggest you get familiar with our policies at WP:FRINGE, WP:NOTE and WP:V. You're going to have a hard time getting acceptance for this nut-job stuff. SteveBaker (talk) 04:55, 25 October 2009 (UTC)[reply]

Thank you Steve, I just want to say I understand that even if these subjects are true, they have to follow Wikipedia guidelines like you said-so I will try my best and only include things acceptable to Wikipedia. To be fair I'll include sources that will even try to debunk my nonsense. So I think its best to have a small section about it, so people will at least become aware that such things might exist, its a start. I appreciate your honest input, I myself have trouble believing in this nut-job stuff too. I might, and there is a very small possibility that I was thinking to add anything on the Shard, if I do, it will be a tiny section, and no original research, only what I find in reputable articles and books. I'll try to find the opinion of more than one scientist. I got a lot of verifiable evidence, I just need to organize it and take out the loony stuff. I want to thank you again, and respect your criticism (it is good to have it), I think being a skeptic for these subjects is always a good thing.--24.23.160.233 (talk) 08:05, 25 October 2009 (UTC)[reply]

alternative line of argument for moon hoax

If China were to claim that it is CURRENTLY landing something on the moon, and for some reason we doubted this claim, then the easiest way to TEST it -- something that doesn't require TRUSTING them at ALL -- would be to actually take some telescopes to it and LOOK. Obviously we can do the same thing with the past, as the light rays from the supposed landing event have been streaming continuously outward like waves from a pebble thrown into a very still lake: the original event is simply still radiating outward. Only space is not flat, but very curved, and there are all sorts of places where the 40-light year distance is traced around large gravity wells. By finding the longest of the paths tracing these wells, we can calculate where the light from the event will be at a certain time, catch up to it (by taking a more direct path), and look at that light. But we don't do that. Why don't we do that? Is there a single reason not to do the ONE thing that would SHOW, definitively, the event right as it's taking place? the one thing that simply CANNOT be faked? I humbly submit that there is one and only one reason why we do not do that: there is no moon. Never has been. 92.224.205.52 (talk) 17:17, 24 October 2009 (UTC)[reply]

spoon - "There is no spoon" - please, get it right! SteveBaker (talk) 22:29, 24 October 2009 (UTC)[reply]
That's an interesting idea though - find where light from an event in the past was bent by a gravity well - and catch up with it at subliminal speeds by taking a short cut - and take another look at it! Sadly - none of the gravity wells within 40 light years are going to bend the light by more than a fraction of a degree - so you'd still have to be traveling at very nearly the speed of light to catch up with them. Then, by the time you got back to earth, time dilation would ensure that there would be no earth...or moon...or anyone to care about whatever you found out. It's an interesting thought-experiment though. Thanks! SteveBaker (talk) 22:29, 24 October 2009 (UTC)[reply]
you're welcome! would you like to collaborate with me on a science-fiction book, containing about one hundred similarly infeasible ideas (which I am happy to share with you by e-mail)? 92.230.68.6 (talk) 00:49, 25 October 2009 (UTC)[reply]
There is an Asimov short story on this very subject - The Dead Past. Original ideas in SF are a bit thin on the ground these days. :) Tevildo (talk) 08:06, 25 October 2009 (UTC)[reply]
"il n'y a rien de nouveau sous le soleil". If we take that attitude in the arts, we might as well pack up our shit now and close down shop, because I guarantee you that the Greeks already knew all the best ways to tell a story. 92.224.206.209 (talk) 18:28, 25 October 2009 (UTC)[reply]

Maximum electron orbital speed

What's the maximum expected speed of an electron in its atom? Can I apply Bohr Model electron speed rule directly with Bohr radius, or shall I take into account the number of protons in that atom? --Email4mobile (talk) 14:25, 23 October 2009 (UTC)[reply]

What purpose do you have for calculating the velocity? Unfortunately, your approach will not yield a "correct answer" (in that it will have subtle consequences which do not make sense theoretically or empirically). To accurately answer your question requires a full quantum-mechanical treatment of the electron. Because its momentum is defined by a probability function, its velocity is also similarly "spread out" - the single electron does not have a well-defined exact velocity. But, the level of complex quantum-mechanical treatment you need to apply depends on your purposes (e.g. are you trying to determine optical effects or scattering, or some other atomic interaction? If so, you need a very thorough QM treatment). If you just want to know this for "trivia", you can just pretend the Bohr model applies, and forget the QM details, and assume a circular orbit... (but this velocity will have no practical useful meaning). Nimur (talk) 14:34, 23 October 2009 (UTC)[reply]
I was just curious know the maximum possible speed an electron would ever achieve inside an atom and compare it to the speed of light. For instance is it possible to have electrons rotating at speeds of 0.08c or even higher? I'm not looking after precise measurements but the worst case to know the upper limit for these velocities. I am afraid I am not that skilled in quantum and modern physics.--Email4mobile (talk) 15:53, 23 October 2009 (UTC)[reply]
Well, a back of envelop ball park calculation shows that within the Bohr model the innermost electron of an atom with Z protons is given by which obviously breaks down for Z>137! Dauto (talk) 17:51, 23 October 2009 (UTC)[reply]
That limit is actually discussed in End of the periodic table and Bohr model. DMacks (talk) 18:00, 23 October 2009 (UTC)[reply]
For instance the 1s electron of gold (Au) has a velocity 58% of light, and the "orbital radius" of the 6s orbital is contracted by ~13%.[33] Franamax (talk) 00:14, 24 October 2009 (UTC)[reply]

Process Control

(Modified by Email4mobile), I a think this should be an appropriate title for the question posted by 85.115.52.180: What is valve hysterics in relation to process control85.115.52.180 (talk) 15:36, 23 October 2009 (UTC)[reply]

Hysteresis, I think you mean. :) See the linked article, and also Control theory. Tevildo (talk) 17:34, 23 October 2009 (UTC)[reply]
Specifically for mechanical valves, Backlash would be a more usual term than hysteresis. Basically, if you're trying to control the flow of a liquid using a valve, your process has to take into account the fact that the valve needs to be turned backwards by some distance before the flow starts to reduce. Tevildo (talk) 17:48, 23 October 2009 (UTC)[reply]

Ford Explorer Injectore Over Fueling

What causes 1998 Ford Explorer injectors to over fuel?

Thank you —Preceding unsigned comment added by 86.138.99.179 (talk) 15:45, 23 October 2009 (UTC)[reply]

Nine times out of ten, the Lambda sensor. However, only a competent mechanic who can look at the vehicle can give you a definite answer. Tevildo (talk) 23:14, 23 October 2009 (UTC)[reply]

Rain measurements

Why is rainfall measured in inches? Isn't that an inaccurate measurement since inches can vary depending on the size of the container? Like in a small cup 3 in comes very fast, but in a bowl it takes awhile for the some measurement.Accdude92 (talk) (sign) 16:13, 23 October 2009 (UTC)[reply]

It is measured in inches (or in millimetres in many parts of the world) since that is the best way to describe how much rain has fallen. If your cup or bowl have parallel sides, instead of being cup-shaped or bowl-shaped, then they would both fill up at the same rate. Rainfall is decribed as the depth of rain that would accumulate in a parallel sided container. --Phil Holmes (talk) 16:39, 23 October 2009 (UTC)[reply]
If the complaint is that a cylinder with a diameter of 3 inches fills up faster than a cylinder with a diameter of, say, 10 inches, a bit of thought will make it clear that they will fill up at the same rate if they are really cylinders and their tops are the same diameter and shape as their bottoms; and if the fall of rain is consistent — a cylinder with a diameter of 100 kilometers will fill up at a slower rate if only half of it is under the rain cloud. Comet Tuttle (talk) 17:08, 23 October 2009 (UTC)[reply]
Another way of thinking about it is that the "real" measurement is "volume of rain per collection area" (e.g. something like cubic feet per square mile). It's a little more obvious to see in the metric system, but you can convert to common units (e.g. inches cubed per inches squared), and cancel out the units from the bottom (e.g. (in.*in.*in.)/(in.*in.) => in.) Numbers stated that way are equivalent to stating how deep the rain would be over the (level) square mile if none of it soaked in or ran off. Twice the area would get twice the volume of rain, but it would still cover to the same depth, since there's twice the area to cover. It works when you shrink the area too. A little thought will show that as long as the opening of the container is level, it doesn't matter how small the observed area is - it'll be about as accurate as the full acre, even if the rain isn't coming straight down. -- 128.104.112.179 (talk) 21:52, 23 October 2009 (UTC)[reply]
I believe that it is common to use a funnel to collect the rain from a larger area into a container with a smaller cross-section, then scale the markings on the container accordingly. Eg if the funnel has a collection area of twice the cross-section of the container, then the container would have "1mm" marks every 2mm on the container. This gives more accurate readings. (See "How does it work" on this page.) The Rain gauge article mentions funnels, but not the scaled marking. (Probably it should.) Putting the smaller container inside a larger container (eg with the same area as the funnel top, and non-scaled markings) such that the smaller container overflows into the larger container allows for measuring more rain (with less accuracy). Mitch Ames (talk) 01:06, 24 October 2009 (UTC)[reply]

wireless LAN justification

I have a question that "which of the method used in wireless LAN either CSMA/CD OR CSMA/CA?" i found the answer that CSMA/CA used in wireless LAN ,but now how can i justify my answer.this is the problem.220.225.244.114 (talk) 16:14, 23 October 2009 (UTC)[reply]

Have you read our article Carrier sense multiple access? Comet Tuttle (talk) 17:10, 23 October 2009 (UTC)[reply]
It's not clear to me what you mean 'justify my answer'. If you need to explain why CA or CD is used then I think the article Comet Tuttle provides will provides the basics, enough at least for it to be easy to do further research. If you need to prove that CA or CD is used then I guess the best thing to do is to go to the official spec sheet and cite that. Failing that, look for a few journal articles which discuss it. Wireless LAN is of course a fairly imprecise term. I presume you want 802.11g or some such Nil Einne (talk) 20:33, 23 October 2009 (UTC)[reply]

List of objects in the solar system ordered by volume

Is there such a list anywhere? And what about ordered by mass - perhaps the mass of the gas giants is not so great. Since what is and what is not a planet has become so confusing recently, including some new planets being discovered, then this would help clarify things. 78.149.146.34 (talk) 18:35, 23 October 2009 (UTC)[reply]

List of Solar System objects by size. The list there can be sorted by various attributes, including mass and volume. Tevildo (talk) 18:38, 23 October 2009 (UTC)[reply]

October 24

Why don't cells make carbon fibers?

With all that carbon and all that elaborate machinery in the cell, why don't living things produce graphitic fibers or carbon nanotubes, which seem to have such nice properties? (and by the way should graphitic fibers redirect somewhere?) Thanks — Walrus heart (talk) 00:32, 24 October 2009 (UTC)[reply]

Well, for one thing, evidence is accumulating that suggests that carbon nanotubes are a pretty nasty health hazard. But evolution can't just magically "do the right thing". That's why animals don't roll around on wheels or communicate via radio waves or have digital circuitry in their brains. At each point, only teeny-tiny steps can happen and this very often results in amazingly evolved but hideously illogical solutions to life's little problems. For example, non-human mammalian babies reach an age where if they continued to consume their mother's milk, it would deprive their younger newborn siblings. Yet they have not evolved an efficient brain mechanism that makes them think "Hmm - I really don't want mommy's milk anymore because it would deprive my younger brother" - but instead we wound up with a mechanism that gives them indigestion and worse every time they try to drink milk of any kind! (lactose-intolerance) When you think about it - that's an incredibly dumb way to achieve the intended goal! Who wants baby animals getting sick for no particularly good reason? But that's all that the random shuffling of genes could do when the need arose. SteveBaker (talk) 01:14, 24 October 2009 (UTC)[reply]
The article Weaning implies that the initiative to end breastfeeding is taken by the mother. There are many cases where children continue happily to breastfeed up to age 8 and more. Lactose intolerance arises after weaning; about 25% of adults develop none and there is some evidence that there has been evolution in humans in the direction of lactose tolerance that allows the high modern consumption of milk and dairy products. Apropos sibling competion, mamma's standard-issue maternal endowments provide dual nourishment outlets for tandem feeding (triplets however present her a scheduling challenge). Cuddlyable3 (talk) 14:10, 24 October 2009 (UTC)[reply]
I did very carefully state "non-human mammals". Humans are very much the exception to the rule in this regard. SteveBaker (talk) 14:17, 24 October 2009 (UTC)[reply]
I apologise, I thought you were just being rude about other people's babies. Cuddlyable3 (talk) 14:41, 24 October 2009 (UTC)[reply]
Weaning is a little bit more complicated than that, Steve. There is compelling evidence that there is extensive rewiring on the locus coeruleus which facilitates the weaning process. The LC is thought to mediate the attachment of newborns to their mother's milk. This early sensory "imprinting" appears to be the mechanism through which young suckle. Around weaning, the LC undergoes neural changes which results in that attachment being weakened. Thus the young are less compelled to suckle and start to investigate other forms of sustenance.
Whats cool about this is the mechanism through which the LC works: it is linked to the young's ability to undergo positive and negative conditioning. During the suckling phase even negative stimuli (such as being bitten or trampled on by the mother) will result in positive reinforcement for suckling. This is important, because most mothers are not particularly good. But a bad mother is better than no mother, and it would be catastrophic if one developed an aversion to one's only form of sustenance. But after LC rewiring, the young will now respond to negative stimuli with aversion. So now when the mother bites or kicks the older young to chase them away while she suckles her new litter, the older young will now "get the message" and avoid her. We do not know exactly what precipitates this LC rewiring, but it appears to be developmentally regulated. Thus evolution appears to favored quite an elegant solution to the suckling problem. Even better, its all tied in with the ability for young to learn crucial avoidance behaviors (which are necessary for post-weaning survival, but catastrophic during suckling) at exactly the right time. Lactose intolerance has a slightly later onset in most mammals, and while it clearly functions to stop further suckling, it is unlikely to be the mechanism through which weaning occurs. Rockpocket 17:02, 24 October 2009 (UTC)[reply]
Second what SteveBaker stated. It's why living things can be amazingly complex; evolution often results in highly inelegent solutions to simple problems since not only must the current function be useful (or at worst, not harmful), every function along the way at each stage has to be useful (or at worst, not harmful). Furthermore, living things DO make some fantastically amazing polymers, consider silk. Spider silk, for example, has a tensile strength that, gram for gram, excedes that of steel, and is as strong as just about any synthetic fiber out there.--Jayron32 01:57, 24 October 2009 (UTC)[reply]

Mercury "burn"

In an episode of Diagnosis Murder, someone splashed a spot of mercury (liquid elemental mercury, that is) on their hand. They immediately wiped it away, but later a red "burn" mark appeared where the mercury had been in contact with the skin. I think it was termed a "mercury burn" in the script. I seem to remember coming into direct contact with mercury in the past and not suffering from any such problems. Is there any basis to this idea that mercury "burns" the skin, or is it just a fiction? —Preceding unsigned comment added by 86.138.41.250 (talk) 01:02, 24 October 2009 (UTC)[reply]

Sounds like bullshit to me. Mercury isn't an acute poison in this manner, it does not cause direct instant damage like that. Mercury poisoning is a chronic problem that comes from long-term exposure to mercury vapors or consumption of some mercury compounds. I don't see where elemental mercury would ever do something like that. Neither our article on Mercury (element) or on Mercury poisoning makes any mention of contact burns from handling mercury. The latter article notes that "Quicksilver (liquid metallic mercury) is poorly absorbed by ingestion and skin contact. It is hazardous due to its potential to release mercury vapour." --Jayron32 01:51, 24 October 2009 (UTC)[reply]
Agreed. Long-term exposure might but a quick splash wouldn't do more than a little rash at the worst. - Draeco (talk) 04:26, 24 October 2009 (UTC)[reply]
Unless it was hot, of course. Looie496 (talk) 05:49, 24 October 2009 (UTC)[reply]
Unless there was some allergy. We used to roll the stuff around on our school desks in the old days --BozMo talk 07:07, 24 October 2009 (UTC)[reply]
... agreed - I spent many happy hours playing with mercury in my younger days (and before I heard of mercury poisoning), having regular skin contact without any marks appearing. The health risk was probably about the same as that from my many amalgam fillings (i.e. minimal). Hot of very cold mercury might produce a "burn" mark. Contact allergy to mercury does exist, but is very rare. Dentists in the UK didn't test for it before using amalgam fillings. Dbfirs 09:39, 24 October 2009 (UTC)[reply]
IIRC (which I probably don't), something very similar was used in an episode of Jonathan Creek...A skull made of frozen mercury that would burn to the touch? Although, I guess that was probably because it was so cold. Vimescarrot (talk) 12:14, 24 October 2009 (UTC)[reply]
Sitting on Mercury. National Geographic published this picture of a Mercury miner. Cuddlyable3 (talk) 13:24, 24 October 2009 (UTC)[reply]
For the record, the mercury in said episode of Diagnosis Murder was at room temperature. If this "burn" thing is "bullshit" then it makes me wonder about the authenticity of the rest of the medical "facts" in that show. Perhaps it's all mumbo-jumbo...! 86.133.243.75 (talk) 13:27, 24 October 2009 (UTC).[reply]
What?! A work of television fiction with a lack of authenticity? NO! Nooooooo!!!! I won't believe it! SteveBaker (talk) 22:20, 24 October 2009 (UTC)[reply]

Fixing of insect fat body tissues

What is the procedure for fixing insect fat body tissue —Preceding unsigned comment added by 59.93.255.135 (talk) 05:25, 24 October 2009 (UTC)[reply]

Use Hoyer's mounting medium, maybe? I'm not sure. --Dr Dima (talk) 05:52, 24 October 2009 (UTC)[reply]

Cows

Why does a cow have four legs? --41.185.88.129 (talk) 07:01, 24 October 2009 (UTC)[reply]

Becuase it's a vertebrate, and most vertebrates are quadrupedal. Apparently they just evolved that way to get around on land. Mitch Ames (talk) 07:52, 24 October 2009 (UTC)[reply]
So, vertebrates were not predisposed to be four-legged before they tried to crawl onto the land? —Preceding unsigned comment added by 86.133.243.75 (talk) 11:21, 24 October 2009 (UTC)[reply]
It is connected with the layout of fins on the lobe-finned fish from which all tetrapodomorphs (and hence land vertebrates) have evolved. Creatures that did not evolve from this line have all sorts of other numbers of legs, there is nothing special about four, it is just an accident. SpinningSpark 12:29, 24 October 2009 (UTC)[reply]
Four makes sense, though - it's the minimum number of legs that allows you to walk without worrying about balance. Stability without any kind of active balancing requires at least 3 legs (that's simple mathematical fact) and in order to walk you need to lift at least one leg and want at least 3 still on the ground, that means you need at least 4 legs. Any superfluous legs are a potential cause of problems, so an animal with precisely 4 legs is a good choice. There are exceptions (in both directions), but that isn't surprising - there are usually many ways of solving a particular problem, so some animals have ended using other ways. Birds (often even flightless ones) will occasionally flap their wings to stop themselves falling over, humans will hold out their arms to balance themselves, etc. (interesting, both those other solutions still require 2 legs and 2 other limbs for a total of 4 limbs). There are creatures with many legs (eg. centipedes and millipedes), which I guess is the best way to carry a long thin body only a small distance above the ground. There are insects with 6 limbs, but only 4 legs, so that fits with my explanation. The only really weird land animals I can think of are spiders - I don't know why they have eight legs... --Tango (talk) 12:52, 24 October 2009 (UTC)[reply]
Tango, I think you are suffering from a bad case of OR there, with an added dose of anthropomorphism. I don't know of any evidence that evolution favours four legs for stability or has difficulty producing active controls for two legs - do you?. Many major branches of the tetrapod line have gone to locomotion on two legs: dinosaurs, birds, kangaroos and other marsupials. Nor does there seem to be any particularly heavy demand for processing power with many legs, millipedes do not have much of a brain. Early robotic research also came to the conclusion that one leg was the very easiest system to program, see Robotics#Walking robots. SpinningSpark 13:16, 24 October 2009 (UTC)[reply]
It's pure OR but I don't see how I can be accused of anthropomorphism when I gave humans as an example of an exception to the rule... While some aspects of life are "accidental" (eg. we use D-Glucose not L-Glucose for respiration - at least, I've never seen an explanation for that choice), most have evolved for a reason. I may be completely wrong about why so many animals have evolved to have 4 legs, but I doubt it is entirely random. --Tango (talk) 15:01, 24 October 2009 (UTC)[reply]
Yes, I should have said tetrapodomorphism, sorry. It is incorrect to say "so many animals have evolved to have 4 legs", they all belong to the same clade, that is, they have a common ancestor and have inherited a common characteristic. I cannot think off-hand of any species that has separately evolved into a 4-pod. Other groups are quite happy with other numbers, notably the wildly succesful Insecta are 6-pods. To address your question on spiders, the reason you cannot think of a good reason why spiders need to be 8-pods for their lifestyle is that they probably don't and it will never make any sense trying to analyse it that way. However, it immediately makes sense when you consider that spiders are related to crustaceans, they have merely inherited a common characteristic. SpinningSpark 17:32, 24 October 2009 (UTC)[reply]
If you examine it in terms of clades you are obviously going to conclude that it is caused by the common ancestor having that characteristic because that is a feature of your examination. It makes more sense to examine it in terms of how widespread that characteristic is at the current time. If we restrict ourselves to land animals then we have small 6-legged, 8-legged and many-legged animals and large 4-legged (and a few tetrapods that walk on 2 legs). Why is it that only tetrapods have evolved to be larger than a few centimetres (at least, I can't think of any counter-examples)? It has been the case for an extremely long time so it isn't just that evolution hasn't had enough time to change it. Evolution is definitely capable of removing limbs (cf. aquatic mammals and snakes [which are a counter-example! I think this is an example of the exception proving the rule - snakes are unusual in their lack of legs]) and we see Polymelia quite often (although I don't know how often it has an inheritable cause) so it doesn't seem likely that there is some biological reason that prevents tetrapods from evolving more or fewer legs, that means that either by random chance it has never happened (which is unlikely, IMO) or there is some benefit to 4 legs over other numbers. --Tango (talk) 18:31, 24 October 2009 (UTC)[reply]
The point of looking at the clade is that inherited features within the clade are just neutral. If the same feature evolves in other clades then there is an indication that it is particularly advantageuous. I am fairly confident that 4-podism was not the driver to large size or that lack of it prevents groups evolving large size. Limitations in skeletal structure and vascular system are far more significant. Besides which, large examples of other clades can be found, eg coconut crab (10-pod). SpinningSpark 10:34, 25 October 2009 (UTC)[reply]
If a clade very rarely changes a particular feature (in this case only snakes, legless lizards and aquatic mammals have fewer legs and none have more legs, unless I've missed a species) that is also suggestive of an advantage. For example, tetrapods have a wide range of numbers of digits on their four legs, which suggests there is no great advantage to, say, 5 digits compared to 6 (some species have a number of digits that is advantageous to them, of course, but wouldn't be advantageous to tetrapods in general). The coconut crab is exceptional - in this kind of subject a single exception doesn't invalidate a rule, it just gives you a way to work out the reason for the rule by seeing what is different about the exception. --Tango (talk) 13:11, 25 October 2009 (UTC)[reply]
I'd have thought the more legs the merrier, since the more you have, the less your mobility is affected by damage to or loss of one or more of them. 86.133.243.75 (talk) 13:30, 24 October 2009 (UTC)[reply]
Yet very few animals have more than 4 legs. If more was better, one would expect most animals to have more legs. --Tango (talk) 15:01, 24 October 2009 (UTC)[reply]
Eight million species of insects (all of whom are 6 legged) would like to differ with your viewpoint! From a stability point of view, 2 and 3 legs clearly suck - 2 legged animals have to actively balance, which costs them energy on a continual basis. 4 legs solve the stability problem - but only at the cost of relatively slow locomotion. If (as you said) they gain this stability by moving one leg forward at at time (which they rarely do), then their top speed would be awful! Most quadrupeds don't actually take advantage of the stability you're talking about - except when standing still. 3 legs is plenty to do what pretty much all quadrupeds need to do. Insects (6 legs) DO get the benefit you claim - they stand stably on 3 legs while they move the other 3 forwards to get a new stable position. This certainly simplifies the control issues - exerting sinusoidal patterns onto the lift and move-forwards muscles with the right phase is really all you need to make a 6 legged animal run pretty fast. SteveBaker (talk) 22:15, 24 October 2009 (UTC)[reply]
Sorry, I should have said very few large animals. I realised I had forgotten the small animals and clarified that above. You are wrong that tetrapods rarely move one leg at a time - see Horse gait#Walk. When they have no need to go fast, they use a nice stable gait and only use less stable ones when there is a need for speed. I think that same is true for other animals. Now, I may be completely wrong about stability being the main benefit for 4 legs - the point I am trying to make is that there almost certainly is a benefit. While evolution does result in random choices that just get stuck, it doesn't happen very often with features as widespread as tetrapodism (for things like which isomer of glucose we use there is a benefit of using the same isomer as every other lifeform uses [ie. you can eat], even if there isn't a benefit to one isomer over another, that isn't the case for number of legs). --Tango (talk) 13:11, 25 October 2009 (UTC)[reply]
Start removing them and you'll see why. Vranak (talk) 13:19, 24 October 2009 (UTC)[reply]
I've seen a three-legged cow before. She seemed to be managing okay-ish. I'm guessing that she would probably struggle with only two. --Kurt Shaped Box (talk) 13:45, 24 October 2009 (UTC)[reply]

Okay, I'll bite. Why does a cow have 4 legs? Cuddlyable3 (talk) 13:26, 24 October 2009 (UTC)[reply]

to get to the other side. John Riemann Soong (talk) 13:35, 24 October 2009 (UTC)[reply]
except when said cow is already on the other side in which case the question is udderly moot. Cuddlyable3 (talk) 13:40, 24 October 2009 (UTC)[reply]
I want a reference that the even have four in the first place! SpinningSpark 13:42, 24 October 2009 (UTC)[reply]
I don't want to lower the tone of the desk, but I feel that you need to know that I just found this in an internet search for this question. SpinningSpark 13:54, 24 October 2009 (UTC)[reply]

I nominate that video for the Golden Globes Award. Cuddlyable3 (talk) 14:22, 24 October 2009 (UTC) [reply]
Ok, just one more, then I promise to leave this thread alone. What to you call a cow with no legs? Give up?
Ground beef! SpinningSpark 14:38, 24 October 2009 (UTC)[reply]
Because its Hox genes encode four legs. Rockpocket 15:36, 24 October 2009 (UTC)[reply]
Because we live in 3 dimensions. In two dimensions they have three legs and in four cows have five legs [citation needed] ;-) Dmcq (talk) 17:32, 24 October 2009 (UTC)[reply]
Well, that would be consistent with my proposed explanation. --Tango (talk) 18:31, 24 October 2009 (UTC)[reply]
Well, we seemed to have milked that for all its worth! Richard Avery (talk) 22:28, 24 October 2009 (UTC) [reply]
Four - four - any advance on four? yes I have a bid of five - no I don't need another bid of five - any advance on five ? yes I have a bid of six - going at six - going - GONE. Cuddlyable3 (talk) 22:50, 24 October 2009 (UTC)[reply]
I thought everyone knew that cows have 6 legs (the two in the rear plus the forelegs in front). StuRat (talk) 04:27, 25 October 2009 (UTC) [reply]

high dc voltage and current measurement

how are high dc voltage and current measured ??? —Preceding unsigned comment added by Waghmare (talkcontribs) 10:11, 24 October 2009 (UTC)[reply]

It depends what you mean by "high". A simple resistive divider probe (such as this) used with an ordinary Multimeter can measure voltages up to about 50kV DC - Hall effect ammeters (such as this) can measure up to about 2000A. Tevildo (talk) 13:27, 24 October 2009 (UTC)[reply]
Measurement of high voltage and high current. Cuddlyable3 (talk) 13:37, 24 October 2009 (UTC)[reply]

:You could use VTs (voltage transformers) and CTs current transformers. See also article on Transformers —Preceding unsigned comment added by 79.75.37.202 (talk) 22:47, 24 October 2009 (UTC) [reply]

phosphate (and diphosphates) as leaving groups

I see that a lot of Wikipedia likes to use the high school explanation of a "high energy bond" for why phosphorylation drives certain processes. Recently I learnt about role of ATP in fatty acid synthesis and now the use of ATP makes so much sense! But are there other uses of ATP besides using phosphates as leaving groups? Mechanistically, how does ATP drive a reaction forward? I get the idea that a triphosphate group is sort of like the phosphoric version of acid anhydride, and thus cleaving ATP into a phosphate ester and ADP is favourable (diphosphate is a good leaving group). And phosphorylating an already phosphorylated substrate is still thermodynamically favourable...? John Riemann Soong (talk) 13:46, 24 October 2009 (UTC)[reply]

Swine flu vaccine

On average, about how long does it take after being vaccinated for an individual to be fully protected from swine flu? --75.28.168.53 (talk) 14:08, 24 October 2009 (UTC)[reply]

The U.S. CDC estimates that it takes about 2 weeks. Note that kids under 10 years need 2 doses (spaced about 4 weeks apart) and your use of the word "fully" might reflect a misconception about its efficacy (perhaps not your misconception, but other readers might be confused) - the flu vaccine reduces incidence, severity, and mortality, but does not completely prevent infection in all recipients. -- Scray (talk) 14:49, 24 October 2009 (UTC)[reply]
And remember, it protects you from swine flu but not other strains of flu (including other strains of H1N1). So, just because some people get the flu after having the jab doesn't necessarily mean the vaccination hasn't worked. --Tango (talk) 15:21, 24 October 2009 (UTC)[reply]

Why would a portable radio's signal go out for one station?

I had a portable radio/CD player a few years back, where the station I listened to all the time gradually faded to where it wouldn't play anymore. Not sure if it was analog or digital (I got it around 2000, if that helps), but I was just wondering what would cause it to do that? It seemed to play other stations well. And, it wasn't the radio station, becuase I tried on other portable radios that I have.

I suppose the receiver part for that one could wear down, but I'm really unsure. SO, I thought I'd ask if anyone had any ideas.209.244.187.155 (talk) 14:20, 24 October 2009 (UTC)[reply]

Reception of FM stations can be affected by standing waves due to interference from reflected signal paths; the signal wavelength is about 3 meters. Even though a receiver is not moved signal reflections can change over time e.g. when a nearby building is put up. Maybe that happened with the OP's favourite program. Cuddlyable3 (talk) 14:36, 24 October 2009 (UTC)[reply]

sky

If water is blue because of sky reflection then how come water is still bright blue when they show photos of earth from outer space? it's not reflecting space, that's for sure. —Preceding unsigned comment added by 82.44.55.2 (talk) 14:30, 24 October 2009 (UTC)[reply]

Where did you get the idea that it is caused by reflection? According to our article Ocean#color it is not. SpinningSpark 14:49, 24 October 2009 (UTC)[reply]
I imagine the OP got the idea because it is an extremely common misconception. Water is usually only noticeably blue in large quantities, which means a lot of people don't realise that it genuinely is blue. --Tango (talk) 15:03, 24 October 2009 (UTC)[reply]
See also: Color of water. --NorwegianBlue talk 15:40, 24 October 2009 (UTC)[reply]
As for the sky, check out Rayleigh scattering#Reason for the blue color of the sky. ~ Amory (utc) 17:06, 24 October 2009 (UTC)[reply]

In addition, to the extent that sky reflection contributes to the water's color, this is just as true for water photographed from space as it is when photographed from the same angle from a point in the air. The fact that you're looking down from above sky doesn't change anything about the light that reaches and reflects off the water. --Anonymous, 18:44 UTC, October 24, 2009.

(OR) Many postcards show lakes having an intense emerald blue colour that one never actually sees. This looks good in tourist advertisements. An explanation I have received is that colour films are sensitive to invisible ultraviolet light reflected from the water, and render it as a saturated blue. Only a select few have actually seen earth from outer space. Cuddlyable3 (talk) 22:33, 24 October 2009 (UTC)[reply]
Photographs may have been taken using filters to enhance the colour. These days colour enhancements are likely to have been done digitally. Mitch Ames (talk) 00:15, 25 October 2009 (UTC)[reply]
"Momma, don't take my Kodachrome away". StuRat (talk) 04:16, 25 October 2009 (UTC)[reply]
Too late! --Anonymous, 21:10 UTC, October 25, 2009.
You can see the blue color of water in an indoor pool. StuRat (talk) 04:16, 25 October 2009 (UTC)[reply]
... but the faint turquoise-blue of the water is often swamped by blue paint on the sides, by reflection from the sky or ceiling, and by chemicals in the water. Interestingly, heavy water is not blue. Dbfirs 08:57, 25 October 2009 (UTC)[reply]
If by any chance it's green, then I have several tons of heavy water in my back-yard pool right now! Who should I call? SteveBaker (talk) 20:18, 25 October 2009 (UTC)[reply]
Is the OP sure that the sky isn't blue because it's relecting the water? B00P (talk) 04:52, 26 October 2009 (UTC)[reply]

Humans after extended time in outer space

What happens to humans after they land on earth after an extended stay in outer space, say 5 months? --Reticuli88 (talk) 14:40, 24 October 2009 (UTC)[reply]

A ticker-tape parade and pictures in the newspapers. Cuddlyable3 (talk) 14:45, 24 October 2009 (UTC)[reply]

I meant physically and mentally.--Reticuli88 (talk) 14:46, 24 October 2009 (UTC)[reply]

Of course you did, some people around here just like to muck about making unhelpful jokes. —Preceding unsigned comment added by 82.44.55.2 (talk) 18:09, 24 October 2009 (UTC)[reply]
See Human spaceflight#Medical issues. Rockpocket 15:20, 24 October 2009 (UTC)[reply]

Natural gas from the outer planets?

Would it be possible to commercially extract methane,hydrogen gas,hydrocarbons or helium from the outer planets or moons and parachute it back to Earth?80.0.99.159 (talk) 14:58, 24 October 2009 (UTC)[reply]

It would certainly be possible to do it (Uranus, Neptune or the moons of the outer planets would be the best choices for methane and other hydrocarbons - they exist in very small proportions in Jupiter and Saturn and the increased gravity makes extraction most costly even if you can isolate it). I'm not sure it would be economically viable to return it to Earth, though - big solar panels in space beaming energy to Earth by microwaves is at least as easy an engineering problem and probably far cheaper. It might make sense to extract it for use in the outer solar system, though - there is less solar energy out there and you don't have to move the extracted gasses as far, so the costs would be more favourable to the extraction. I'm not sure there is really any point extracting helium, though. If you can get Helium-3, it might be useful in a fusion reactor, but we don't have any viable fusion reactors yet. --Tango (talk) 15:12, 24 October 2009 (UTC)[reply]
To use These hydrocarbons as a source of energy in the outer solar system you would also need a source of oxydiser. There is no molecular oxygen out there. Dauto (talk) 16:37, 24 October 2009 (UTC)[reply]
That is a good point. We could, in theory bring the oxygen from Earth (or produce it in the inner solar system using solar power to electrolyse water). We would then only have to transport half our rocket fuel from the inner solar system. I'm not sure that would be more efficient than other means of generating energy in the outer solar system, though (tidal power is one option). --Tango (talk) 18:36, 24 October 2009 (UTC)[reply]
There's quite enough energy on earth without importing energy. Some solar panels in the Sahara or Utah or Gobi or some wave power and good electrical transmission could solve lots of problems for instance, and would be cheaper cleaner and safer than getting methane from other planets or beaming microwaves from space. Dmcq (talk) 17:29, 24 October 2009 (UTC)[reply]
I agree about the methane - we have enough problems with global warming as it is - without importing another source of carbon from outside! We're never going to run out of fossil fuels because we're going to have to stop burning them long before they run out.
However, having large amounts of Helium-3 (which is thought to exist in useful quantities on the Moon) available would make building super-clean, efficient fusion reactors much more feasible. Such reactors would need very little helium-3 to produce vast amounts of energy - so the "shipping cost" isn't such a concern as it would be for almost any other product mined in space. It isn't so hard to imagine some scheme whereby robotic mines on the moon could produce the stuff and ship it back here by hurling containers of it off the lunar surface using a linear induction motor launch track or something. The mines themselves could be powered using the same helium-3 fusion technology - and could be more or less self-sustaining.
The idea of beaming energy down from orbit (probably in the form of microwave laser beams) has it's attractions. Without all of that atmosphere in the way, being able to park yourself in an orbit where you'd get almost 24 hours of sunlight per day and with zero-g meaning you wouldn't need huge supporting structures - you could greatly increase the efficiency of solar panels. Whether that increase would be worth the enormous cost of getting them up there and getting the power back down again - is anyone's guess at this point. SteveBaker (talk) 22:02, 24 October 2009 (UTC)[reply]
Lunar He-3 mining is almost certainly a good idea if we can invent the fusion reactors. Getting things from the lunar surface to Earth's surface is far easier than getting it here from the outer solar system. --Tango (talk) 22:23, 24 October 2009 (UTC)[reply]
I'm not too sure about that. Overall considering the extraction costs it may be cheaper to get it from an outer planet. Travel is fairly cheap, the main problem is the money tied up in the time. Dmcq (talk) 22:53, 24 October 2009 (UTC)[reply]
The question with He3 fusion is whether you can develop the fusion technology without a sufficient supply of He3 to power it...and then, whether it's worth setting up all of the infrastructure to mine the stuff on the moon with the fusion plant design being merely a theoretical concept. It's a chicken and egg problem. The problem with mining He3 in the outer planets is that it's such a trek out there to fix problems, shuttle spare parts, etc. You can get to the moon in three days - if your lunar soil excavator needs a new reverse spurling frangulator, you'll not lose much production fixing it. But it's gonna take the best part of a year to get to Mars - and then only when the Earth/Mars system are in the right places in their respective orbits. SteveBaker (talk) 04:37, 25 October 2009 (UTC)[reply]

Using solar and RTG powered ion-electric engines might make it economic,but importing methane would not be green.However, forgetting about helium3, isn't normal helium running out and becoming very expensive?80.0.108.29 (talk) 13:21, 25 October 2009 (UTC)[reply]

Yes, regular helium is becoming scarce - but not sufficiently to warrant the spectacular costs associated with bringing it back from the Moon (or anywhere else for that matter!). The effort to produce (say) just 20 kilograms of helium-3 from the moon would be insane - but those few kilo's are enough to produce all the energy needed by an entire 1000 megawatt power station for an entire year...that's enough to replace about 10 million tons of coal. I could believe that the cost to mine and transport 10 million tons of coal could be more than the cost to mine and transport 20kg of Helium-3 from the moon. However, 20 kg of regular helium is unlikely to be enough to fill the kids party balloons for one major US city for a day and would probably take a comparable amount of effort to produce! Hardly likely! However, if we were mining He3 on the moon - then immense amounts of regular helium would be produced as a by-product - and it's possible that it might be worth shipping that back to earth if the stuff get really scarce down here. SteveBaker (talk) 20:14, 25 October 2009 (UTC)[reply]
Could you power an ion drive with an RTG? RTGs are good for low levels of power production for long periods of time (which makes them perfect for probes to the outer planets). Ion drives need quite a lot of power (existing ones use several kilowatts, I think) and existing RTGs generate at most a few hundred watts, and that reduces over the lifetime of the RTG. --Tango (talk) 23:19, 25 October 2009 (UTC)[reply]
Dunno - but once you get the processing pipeline 'primed', you don't really care how long it takes to get the stuff back to earth - there's really only a need to nudge it in the right direction and let the orbital dynamics do the rest. IMHO, you wouldn't want elaborate rocket motors on the shipping containers - hence my suggestion that you launch them with solar-powered linear induction motors or something. The containers could probably contain just a few kg of He3 apiece and be launched at spectacularly high g-forces from the lunar surface. Alternatively, a lunar version of the space-elevator could do the job very cheaply. SteveBaker (talk) 02:03, 26 October 2009 (UTC)[reply]
I was assuming we were talking about the outer planets - there is really no point using an RTG in the inner solar system, solar power is much better. I agree that getting things from the lunar surface to Earth orbit should probably be done using some kind of mass driver. --Tango (talk) 02:08, 26 October 2009 (UTC)[reply]

Pterodactyls

Were pterodactyls large and strong enough that a person could ride on its back, similarly to the way dragons are used in some fantasy settings? --70.129.184.103 (talk) 16:34, 24 October 2009 (UTC)[reply]

No. Large birds even today have difficulty taking off. Their weight is really at a premium. See bird flight.--Shantavira|feed me 16:47, 24 October 2009 (UTC)[reply]
Is there any large, flying animal (modern or extinct, it doesn't matter) that's capable of carrying a human? --70.129.184.103 (talk) 17:35, 24 October 2009 (UTC)[reply]
Unlikely. It takes a lot of energy to feed muscles and the animal wouldn't benefit from having big enough muscles to lift itself and a human when it has only ever needed to lift itself (and maybe its young or some food for its young). You might be able to intentionally breed such a bird, but it wouldn't be easy. (Land animals can carry humans since the extra muscle required to walk with extra weight is so much less than the extra muscle required to fly with it.) --Tango (talk) 18:40, 24 October 2009 (UTC)[reply]
What about Haast's Eagle? If the bird was indeed strong enough to carry off small/young humans, then it may also have been the case that it could've been used as a (probably extremely unwilling and likely short-range) mount. That said, the Ostrich is a much more substantial bird and is (AFAIK) the only living bird that can be ridden by adults - yet (from what I remember reading somewhere a long time ago) it struggles to even stand up with a human weighing more than 120lbs on its back. --Kurt Shaped Box (talk) 20:06, 24 October 2009 (UTC)[reply]
According to our article, the heaviest Haast Eagles weighed in at 33lbs. So, no - it couldn't have been ridden by a human. Even an anorexic 5 year old would weigh more than the largest Haast Eagle...and let's not forget the increase in drag by adding a very un-aerodynamic human to that smooth, slippery Eagle body. SteveBaker (talk) 05:18, 25 October 2009 (UTC)[reply]
No - the problem is that of volume versus area. In terms of flying, one critical number is the "wing loading" - the body-weight to wing-area ratio. When you double the size of an animal, you pretty much increase it's weight by a factor of 8 and it's surface area by a factor of 4. When you double the size of a bird, the wing loading doubles - making it twice as difficult to get off the ground. That's why insects seem to fly so effortlessly - they are tiny, and halving the size of an animal halves the wing loading. For an animal to have enough spare lift capacity to get a human into the air, it would have to be huge - and as you make the bird larger and larger in order to have that spare capacity - the drastic increase in wing loading rapidly overtakes their ability to fly. Birds also have to have a whole lot of adaptations for flight - including very lightweight bones - that makes them fragile creatures. Quite unable to bear lots of extra weight.
Pterodactyls had to operate with the same laws of physics - there is no reason to assume that they did any better - and their lack of fancy flight feathers strongly suggests that they did a lot worse. Modern aircraft have higher wing loading than birds and pterodactyls - which they have to compensate for by having high forward speed and powerful engines, but for animals that get their forward thrust from flapping - the mechanical difficulties of moving those gigantic wings up and down makes that solution impossible too.
When you look at the Gossamer Albatross (the first human-powered plane) - you can see just how much wing area you'd need in such a bird. That plane was incredibly fragile - and there is no way you could make those big flimsy wings actually flap!
Probably the best chance of such a creature existing would be if it survived only by gliding over steep, windy mountains where the slope lift would allow them to stay aloft despite crazily high wing loading. For a long time, scientists believed that the larger species of Pteranodons could only fly in this way. SteveBaker (talk) 21:45, 24 October 2009 (UTC)[reply]
Size comparison of the two known Quetzalcoatlus species and a human
Wait, why the comparison with the Gossamer Albatross? It's a very unusual craft. The GA was constructed like that,because it had such incredibly little horsepower. Aren't you making an assumption about the muscle strength of pterosaurs? Even professional paleontologists have not come to agreements about the flight mechanics or even the total body weight of pterosaurs because they're so different than any living animal.
If a pterosaurs wanted to launch from a cliff, and then ride thermals, it doesn't seem entirely beyond belief that it could carry a small human. Nine meters is an entirely reasonable wingspan for a hanglider, and Pterosaurs have been found with wingspans greater than 10meters. (And it's suspected that a complete Hatzegopteryx skeleton would be even larger if one is ever found!) APL (talk) 04:47, 26 October 2009 (UTC)[reply]


If you're interested in a theoretical creature that could carry an adult human in the air, I suspect that it would need to be something like a blimp, perhaps with many hydrogen "balloons", each made of highly elastic sacs, like a frog's throat. It might also have wings to control it's flight. I would suspect that the evolutionary path to such a flying animal would be from some fish or other marine animal, as many of them already use the concept of an expandable ballast "tank". StuRat (talk) 04:07, 25 October 2009 (UTC)[reply]
Yes - that would work - but it's hard to see how a proto-gasbag creature would evolve hydrogen production. SteveBaker (talk) 04:27, 25 October 2009 (UTC)[reply]
My knowledge of biochemistry is minimal, but plenty of process created hydrogen ions (eg. glycolysis). I expect some mechanism exists to get the extra electron. I agree that the most likely route is from a water dwelling animal with a gas bladder changing the gas used from air/oxygen to hydrogen. --Tango (talk) 13:29, 25 October 2009 (UTC)[reply]

Artificial Chocolate Flavour

Hello. Is there an ester that tastes like chocolate? Thanks in advance. --Mayfare (talk) 16:35, 24 October 2009 (UTC)[reply]

As I understand it, chocolate is a rather complex flavor, consisting of many different molecules. So, no, there is no single ester that tastes much like chocolate. The only chocolate substitute I'm aware of is carob, and it's not all that close. StuRat (talk) 03:52, 25 October 2009 (UTC)[reply]

Any chance colonoscopy can be replaced?

I just read about the preparation for the procedure. No wonder there're so many public service ads for it! Is there any chance that in the future this procedure will be replaced by something more agreeable? Imagine Reason (talk) 17:35, 24 October 2009 (UTC)[reply]

See Virtual colonoscopy I guess someone could swallow a camera and the footage viewed after it had passed through the person being examined, but as the article states, optical colonoscopy means you can do procedures while you have a look. Endocopic procedures would be preferable to open surgery, so even if virtual colonoscopy was used, if a procedure was to take place the unagreeable preparation would have to occur.MedicRoo (talk) 17:41, 24 October 2009 (UTC)[reply]
It's not that bad... you take a laxative, and don't eat for a day. Other than the laxative, it's not much different than the preparation you do before getting your blood drawn. As far as medical procedures go, it's pretty straightforward, and you are usually somewhat sedated during it. --98.217.71.237 (talk) 18:50, 24 October 2009 (UTC)[reply]
I'll go further and say the experience was more interesting than bothersome. I watched "my" picture and chatted with the nurses while it went on. I had no idea that the colon seen from the inside is not round but triangular. For anyone longing to know, my colon was declared to be in excellent condition. Cuddlyable3 (talk) 22:14, 24 October 2009 (UTC)[reply]
The laxative is the part that sounds really disagreeable. Plus I've read that it's more like a day and a half of little food ingestion. Imagine Reason (talk) 01:06, 25 October 2009 (UTC)[reply]
The laxative is to make sure one does what one does anyway. It's not a good idea to go straight from the colonoscopy to an important meeting because the colon will have been inflated with air that has to be expelled the way it went in. Cuddlyable3 (talk) 19:24, 25 October 2009 (UTC)[reply]
One of the main benefits of physical endoscopic procedures as opposed to "virtual" radiological imaging is the potential to take biopsies or perform therapeutic manoeuvres. Colonoscopy is unlikely to be replaced any time soon. Axl ¤ [Talk] 22:09, 25 October 2009 (UTC)[reply]

Can you identify this bug?

Unidentified bug in my home

I have seen this little guy and others like him in my home in Kentucky. Any idea what he is? -- KathrynLybarger (talk) 17:52, 24 October 2009 (UTC)[reply]

I think this is him!
The lighting is a bit bad, but it may be a species of Soldier beetle; there are many many kinds of soldier beetle, but this would be my first guess. --Jayron32 20:00, 24 October 2009 (UTC)[reply]
Looks like a true bug to me, maybe in Coreidae family. Hard to say, though. Can you please take a picture with the camera flash on (assuming the critter is still there)? --Dr Dima (talk) 00:43, 25 October 2009 (UTC)[reply]
Isn't that the rare Blind Venetian Beetle ? :-) StuRat (talk) 04:40, 25 October 2009 (UTC) [reply]

Good call, Dr Dima! He did look quite a bit like the guy in the Leptoglossus article. Thanks!! -- KathrynLybarger (talk) 21:07, 25 October 2009 (UTC)[reply]

Air composition

what is the percentage of o2 in the air? —Preceding unsigned comment added by 24.60.6.106 (talk) 18:37, 24 October 2009 (UTC)[reply]

We have articles about air and oxygen, both of which seem good places for you to learn to look up this type of information. DMacks (talk) 18:41, 24 October 2009 (UTC)[reply]
Roughly 21%. SteveBaker (talk) 21:25, 24 October 2009 (UTC)[reply]
This figure is also dropping at roughly just over twice the rate that CO2 concentrations are rising (roughly 100 ppm, or 0.01% in the past 100 years for CO2), mainly because each molecule of carbon dioxide is composed of two oxygen atoms. ~AH1(TCU) 00:14, 26 October 2009 (UTC)[reply]
Worrysome as the rise of CO2 levels is - the reduction in oxygen content as a result of that is truly negligable. The CO2 problem is measured in hundreds of parts per million (we're currently at about 390ppm) - the effect of that on oxygen levels is therefore also hundreds of parts per million - which is just nothing compared to 21%. SteveBaker (talk) 01:57, 26 October 2009 (UTC)[reply]
yeah, but it is a good rejoinder to those whose religious beliefs tell them that the additional CO2 is all released by volcanos. Gzuckier (talk) 03:29, 26 October 2009 (UTC)[reply]

What is this called?

I'm preparing for a trip to the literal other side of the world, and I'm trying to force my sleep schedule towards the time zone I'll be working in. Is there a term for this practice? SDY (talk) 19:41, 24 October 2009 (UTC)[reply]

Its been referred to in academic studies as Preflight sleep adjustment Rockpocket 20:12, 24 October 2009 (UTC)[reply]

Anabolism in Heterotrophs

In heterotrophs, anabolic processes synthesize organic compounds. Would the following be a true statement?

Heterotrophs are able to synthesize all their needed organic compounds solely through anabolic reactions.

Heterotrophs, of course, must get the carbon for these organic compounds and the glucose to undergo cellular respiration from other organisms, but isn't the synthesis solely anabolic? —Dromioofephesus (talk) 20:25, 24 October 2009 (UTC)[reply]

Not quite. Many organisms are unable to synthesize certain compounds, hence the classes of essential nutrients (including essential amino acids, among others), for example. DRosenbach (Talk | Contribs) 04:50, 25 October 2009 (UTC)[reply]
That's a good point. I didn't think about that. —Dromioofephesus (talk) 05:00, 25 October 2009 (UTC)[reply]

Eggs (the kind in a woman's body)

When was it first discovered that a man's "seed" was not enough -- that a woman's egg, too, was needed? —Preceding unsigned comment added by 75.40.136.211 (talk) 20:43, 24 October 2009 (UTC)[reply]

Leonardo da Vinci 1452-1519 drew dissections of a woman and embryos which may be the earliest documented investigation of woman's reproductive organs. Cuddlyable3 (talk) 22:08, 24 October 2009 (UTC)[reply]
What I mean was, when was it first discovered that a woman's reproductive organs were not simply a ground of sorts into which a man's "seed" was planted? —Preceding unsigned comment added by 76.210.248.231 (talk) 22:21, 24 October 2009 (UTC)[reply]
It wasn't really a Eureka! moment, in as much as it was the accumulation of lots of different scientists work. William Harvey proposed that all creatures arise from eggs ("ex ovo omnia") rather than just sperm, back in the 1600s. In 1826 Karl Ernst von Baer discovered the ovum by examining the ovarian follicle of the dog. In 1876, Oscar Hertwig reported that fertilization requires the penetration of a sperm cell into an egg cell, but it wasn't until October 6, 1928 that the first human ovum was recovered and described (by Allen at al). Rockpocket 00:42, 25 October 2009 (UTC)[reply]
Wow -- and I thought Hertwig was famous for his work on teeth. DRosenbach (Talk | Contribs) 01:22, 25 October 2009 (UTC)[reply]
But, of course, it should have been readily apparent even to the ancients that the mother contributed genetically to the resultant child, since the child will often at least partially resemble the mother. Of course, this is most obvious when the parents are quite different from one another. A white father and a black mother producing a brown child would be pretty good evidence. Those that insisted that the mother was only an "empty vessel", in the face of such obvious evidence to the contrary, must have been willfully ignoring the facts to push their POV. StuRat (talk) 03:48, 25 October 2009 (UTC)[reply]
The first generation doesn't actually demonstrate that the woman contributes any heritable factors to the child. It could easily have been argued that the father contributes all heritable factors, and the mother-like characteristics are caused merely by developing in her "environment" and consuming the nutrition provided therein. Although, the perpetuation of said characteristics to subsequent generations would quite surely kill that theory. Someguy1221 (talk) 03:56, 25 October 2009 (UTC)[reply]
What if it were a sex-linked recessive allele? —Dromioofephesus (talk) 05:14, 25 October 2009 (UTC)[reply]
It would kill the theory just as easily, except that the data would be more confusing to our pre-genetics observers. The key is multi-generational heritability, which an "allele" necessarily is, as opposed to an epigenetic situation. Although then there's still the possibility of pan-generational epigenetic inheritance, which would just have been damn confusing to anyone predating modern understanding of DNA regulation. Someguy1221 (talk) 05:23, 25 October 2009 (UTC)[reply]
It was rarely held that the mother contributed nothing (though since genes were not discovered for another few thousand years, I'm not sure how readily apparent it could be that the mother contributed genetically). Aristotle thought that the man's sperm mixed with the woman's blood to form the child. Menstrual blood was thought to be the fertile soil that the seed was planted in. Thus the Greeks realised brothers are blood relatives through the mother, a relationship that would be meaningless if the mother contributed nothing to their generation. Rockpocket 05:28, 25 October 2009 (UTC)[reply]
I thought somebody was going to get picky about my use of the word "genetically". I was using the word to mean "the observed similarity in features between parents and offspring", which existed long before the formal study of genetics. I believe the Bible even mentions it ("like shall beget like", or something along those lines). It you have a better term to use, please let me know. StuRat (talk) 21:35, 25 October 2009 (UTC)[reply]
The term for the historical concept you are referring to is heredity. Rockpocket 22:13, 25 October 2009 (UTC)[reply]
Also, remember that incorrect theories are suprisingly resiliant. People tend to massage the old system in suprisingly convoluted ways than to simply ditch it entirely to adopt a new, more elegent, theory. Consider the whole "geocentric-heliocentric" conflict in astronomy. The geocentric system had to devise some pretty rediculous explanations, like epicycles and things like that, just to avoid having to remove the Earth from the center of creation. So, any explanation of how a mother's traits could be passed to a child even though only the "sperm" was supposed to develop into the child needs no further explanation; it doesn't have to make sense with our modern understanding of genetics, it only needed to make sense to people back then and apparently, given such theory's resiliance, it did. --Jayron32 06:16, 25 October 2009 (UTC)[reply]
I think I will get Jayron a spell-checker for Xmas... :-) StuRat (talk) 21:39, 25 October 2009 (UTC) [reply]

hedron

what is 'the great unified theory' —Preceding unsigned comment added by Johny anu15 (talkcontribs) 21:10, 24 October 2009 (UTC)[reply]

May be you're looking for grand unified theory? Grand unified theories are models that atempt to explain all three interactions of the standard model of particle physics (electromagnetic, weak and strong interactions) as different aspects of a single interaction — Preceding unsigned comment added by Dauto (talkcontribs)
Also, a hadron (note spelling) is a particle made of quarks held together by the strong nuclear force. Jkasd 22:44, 24 October 2009 (UTC)[reply]

help

This question appears to be a request for medical advice. It is against our guidelines to provide medical advice. You might like to clarify your question. Thank you.

Responses containing prescriptive information or medical advice should be removed and an explanatory note posted on the discussion page. If you feel a response has been removed in error, please discuss it before restoring it.

Cuddlyable3 (talk) 21:52, 24 October 2009 (UTC)[reply]

in the context of quarrying, what is a stocking ground? —Preceding unsigned comment added by 86.128.188.238 (talk) 21:47, 24 October 2009 (UTC)[reply]

From this link [34], it seems to be an area where the quarried material is stored before being distributed. Richard Avery (talk) 21:54, 24 October 2009 (UTC)[reply]

Confused about transformers.

In the transformer article, it says that the flux in the core cuts the secondary winding to give secondary voltage. This voltage can then deliver a current to a secondary load. But if a load is connected to the secondary, the secondary current flow will introduce an equal and opposite flux in the core. So how is the flux in the core related to the power transfer when the transformer is working?--79.75.37.202 (talk) 23:10, 24 October 2009 (UTC)[reply]

No, the direction of the secondary current is such that it corresponds to the same flux in the same direction (which you can determine by the right hand screw rule. If the secondary current instead, was being sucked out of the transformer by a generator, exactly the same flux would result and a current would be induced at the primary equal to the original current as in the first case. SpinningSpark 02:32, 25 October 2009 (UTC)[reply]
But, of course, to fully answer this Q, we would need to know if the Transformer is in the form of a race car, jet plane, or rocket ship. :-) StuRat (talk) 04:45, 25 October 2009 (UTC) [reply]
The OP is right to be confused. His question cannot be dismissed as easily as SpinningSpark suggests. The flux due to the secondary current does in fact (neglecting leakage) cancel out the flux due to the primary. In an "ideal" transformer, you would be left with zero net flux and no power transfer. Luckily for electrical engineers, "ideal" transformers do not exist. In a real transformer, there is a non-zero flux called the magnetising flux in the core, but even this is not an indicator of the amount of power flowing between the windings. The magnetising flux is proportional to the primary EMF (if we're talking about sinusoidal AC), and does not depend on the load. The magnetising flux is a necessary condition for power transfer, but it is not what transmits the power. The power actually snakes its way around the windings outside of the core, guided by the magnetising flux in the core.
If you studied transformers at school then at this point you will be saying "No! No! No! That's not what they taught me!" and you would be right. What they teach at school is a theoretical model that gives the right answers in exams but does not describe physical reality. Please see the archived talk pages for the Transformer article for a lengthy and enlightening discussion on this, but be warned, it may mess with your head. --Heron (talk) 11:17, 25 October 2009 (UTC)[reply]
Suppose a transformer is built up as follows. A) The primary winding is just a coiled wire, a solenoid. B) Add a core of iron that is an easier magnetic path than air. Flux in the core links together the magnetic fields of each turn of the coil and that increases the self inductance of the primary. C) Apply a fixed alternating voltage to the primary and the current is limited by the primary inductance (and usually to a lesser extent by its resistance). D) Add the secondary winding. The alternating magnetic field induces an alternating voltage across the secondary terminals but until they are connected to a load there is no secondary current nor change in the primary current. E) When current flows from the secondary into a load the nett magnetic flux in the core gets reduced just as the OP says (though Spark denied). Less flux means the inductance seen at the primary gets reduced so more primary current flows. Thus power taken from the transformer secondary causes the primary to draw extra power. Transformers for power conversion can be made almost 100% efficient when power losses in wire resistance, eddy currents and stray magnetic field are minimised. Ideally there would be no field outside the core. To Heron, I don't see that "power snakes around the windings outside the core". Cuddlyable3 (talk) 18:48, 25 October 2009 (UTC)[reply]

HIV and RNA

The HIV virus contains two strands of RNA in its capsid. This video is pretty good and shows that it uses reverse transcriptase to form the RNA into DNA. My question is, why does the capsid contain two strands of RNA? Surely only one strand is needed? 81.154.211.84 (talk) 23:58, 24 October 2009 (UTC)[reply]

Great question!! I can start with a couple of possibilities: (i) RNA can be hydrolyzed easily, resulting in strand breaks - since the RT enzyme is really good at template-switching, a broken strand can be complemented by its companion, allowing completion of replication; (ii) recombination is a major mechanism that accelerates HIV evolution, and the two strands can be from different HIV genomes if the cell producing the virions is dually infected with two different genetic variants. This latter point is almost certainly the genesis of circulating recombinant forms of HIV that make up a significant fraction of HIV strains worldwide. -- Scray (talk) 01:05, 25 October 2009 (UTC)[reply]
Along the same vein, DNA doesn't have to be double-stranded either. While it's great to speculate, many things will remain just that -- speculation. DRosenbach (Talk | Contribs) 04:46, 25 October 2009 (UTC)[reply]

October 25

Kaposi sarcoma

Is it possible to contract Kaposi sarcoma without having the HIV/AIDS virus? If so, under what circumstances would it be under?72.95.174.219 (talk) 02:19, 25 October 2009 (UTC)[reply]

I know nothing about it, but the article on Kaposi sarcoma seems to state that it is caused by the Kaposi's sarcoma-associated herpesvirus, not the Human immunodeficiency virus (HIV). If that virus were transmitted, I guess it might be able to cause a tumor without the presence of HIV. —Dromioofephesus (talk) 02:44, 25 October 2009 (UTC)[reply]
(ec) Infection with HHV-8, the virus that causes Kaposi's sarcoma, is much more common than the disease - thus, the disease occurs in only a small minority of people infected with the virus. The disease does occur in people who don't have HIV; for example, "Classic KS" is described in certain populations of Mediterranean men, and "Endemic KS" is described in some parts of Africa. Both are described in our KS article. -- Scray (talk) 02:45, 25 October 2009 (UTC)[reply]
The Kaposi sarcoma was discovered in 1872 and this was before any HIV spread in humans.--Stone (talk) 14:24, 25 October 2009 (UTC)[reply]

Time machine coin strory

I've read an Arabic article here (Google translation) and also another story in Arabic Wikipedia (I marked as citation needed). The articles seemed interesting till I read about an experiment done in 1980s to transport a coin from one place to another via fiber optics. The story told scientists could make the coin vanish for about 1 hour and 6 minutes and then suddenly appeared in the other place. I know it's funny but I'd like to confirm if such stories were originally written in English and where can I prove/disprove about them?--Email4mobile (talk) 05:59, 25 October 2009 (UTC)[reply]

I don't know where you could find the stories in English, but for the stories to be true, there would have to be a fundamental change in the laws of physics as we know it or for an event to occur with a fantastically small probability. So I would say that as conclusively as possible, the stories are untrue by Occam's razor, and thus "disproved." Perhaps I am misunderstanding the article though, it is certainly possible for a coin to travel through space and time; just pick up a coin and throw it: See! it just moved through space and time in your reference frame. But I don't think that this is what the article meant. Jkasd 09:14, 25 October 2009 (UTC)[reply]
That's (rather slow) teleportation, isn't it? There have been no accepted successful attempts at teleportation other than transferring quantum information from one atom to another (see Quantum teleportation), that wouldn't allow for transporting coins. --Tango (talk) 15:20, 25 October 2009 (UTC)[reply]
We have to stick by the mantra that "extraordinary claims require extraordinary evidence". We don't see ANY evidence (apart from one shakey translation of what is evidently not a news or science journal - but perhaps a blog site or something). If this were true - or even CLOSE to being true - there would be immense repercussions. Such a thing could not be kept quiet for 20+ years - the experiment would have been repeated, re-repeated and studied until some kind of an explanation emerged. It is overwhelmingly likely that this is a hoax or some piece of fiction being misrepresented as truth. The translation of that Arabic piece does not suggest anything remotely like a proper scientific experiment - the rambling at the end about Stephen Hawking and others stinks of someone with little scientific knowledge seeking to make their story look more believable by dropping some real physicists' names into the story. I don't believe it for one moment.
If I had to be super-charitable, there is a thin possibility for confusion. The nearest thing I could find to this claim was the success in "teleporting the quantum properties of one atom to another across several feet" - which is an interesting physics experiment - but a heck of a lot different than teleporting a coin...plus I found a news report where the reporter used the analogy of transferring the head/tails state of a coin to that of another coin several feet away. I suppose that a non-English reader might misread that as the actual teleportation of the coin...when in fact, no coin was involved, not even a single atom was teleported - only it's "quantum state"...which is a heck of a lot different from the head/tails state of a coin! However, if that's what really happened then the arabic article has CONSIDERABLY embroidered the story with bullshit details about bell jars and the time delay and all sorts of other things that never happened.
So we may conclude that either: (a) this never happened...or... (b) there is no evidence that it ever happened...or... (c) the story in arabic is mere confusion of the experiment I explained above...or... (d) that it's a hoax. If one or more of those things is true (which is surely the case) - then this should be deleted from the Arabic Wikipedia immediately. A little blue "citation required" tag is only appropriate when the fact in question is not seriously disputed but a citation is needed. In this case, the "fact" is very seriously disputed and should be deleted until/unless at least one solid reference can be found (ie a Peer-reviewed scientific paper in respectable journal). Nothing less than that standard of proof would be acceptable for such an outrageous claim! SteveBaker (talk) 19:58, 25 October 2009 (UTC)[reply]
If someone could actually do this, they would be the most famous scientist in the world and the experiment would have been world news - it's clearly bollocks. --Cameron Scott (talk) 20:00, 25 October 2009 (UTC)[reply]
Some joker is trying to pull the fiber over your optics. Clarityfiend (talk) 23:05, 25 October 2009 (UTC)[reply]
heck, happens in my house every day. I'm always not finding something where i'm sure i put it, and then it turns up somewhere else. Gzuckier (talk) 03:24, 26 October 2009 (UTC)[reply]

What is this boat.

Doing some google earth, I came across a boat at 40°48.655′N 14°5.602′E / 40.810917°N 14.093367°E / 40.810917; 14.093367 (near Italy). The google maps image is the one I'm referring to (although other provides may use the same image. There's a large ship (290 feet (88 m)*) and in the middle of the ship is what looks like a giant screw, or a tank, like a door in a ship. What is the purpose of that central structure, and what kind of ship is this? I imagine it's some sort of liquid transport, but it doesn't look like most oil barges I've seen. Shadowjams (talk) 07:57, 25 October 2009 (UTC)[reply]

Probably a Pipe-laying ship. See the seventh drawing from the top at this site - the drawing shows a ship with two reels rather than one, but the basic arrangement is the same. Tevildo (talk) 08:20, 25 October 2009 (UTC)[reply]
Wow! Neat image. I agree with Tevildo. Or it might be a Cable-laying ship. —Steve Summit (talk) 17:07, 25 October 2009 (UTC)[reply]
Thanks. Those both look like matches. It's a curious ship design I'd never seen before. Shadowjams (talk) 20:07, 25 October 2009 (UTC)[reply]

lipids

How does trans fatty acids and unsaturated fatty acids corelate to LDL? —Preceding unsigned comment added by 122.169.11.39 (talk) 08:41, 25 October 2009 (UTC)[reply]

Trans fat raises your LDL levels, lowers your HDL levels, and is even worse for you than saturated fat. Replacing part of the saturated fat and trans fat in one's diet with naturally occurring unsaturated fat will generally result in lower LDL levels. Trans fat is actually a type of unsaturated fat, but it's a different kind of isomer than the kind of unsaturated fat (cis fat) that occurs naturally in vegetable oils, oil that hasn't been artificially hydrogenated. Red Act (talk) 13:12, 25 October 2009 (UTC)[reply]
yeah. as the diagrams in trans fat and saturated fat show, they both have nice orderly carbon chains that pack tightly and are both solid at room temp, while cis fat unsaturated fats have a kink in them that inhibits packing so they're fluid at room temp. so, it appears that it's not so much the saturated or unsaturated nature of the fats that affects the body as their fluidity; less fluid fats drive up the bad LDL and down the good HDL, more fluid fats the opposite. Gzuckier (talk) 03:15, 26 October 2009 (UTC)[reply]

Mixture of gases

I have been working on the following exercise:

A mixture of gases He and O2 has a density of 0.518 g/L at 25ºC and 721 mmHg. What is the percentage in mass of He in this mixture?

I have tried both using the ideal gas formula (which failed, because I know neither V nor n) and the following proportions formula:

Where 32 and 4 are the molar masses of O2 and He, and x is the decimal proportion of O2 in the mixture.

Nevertheless, I just keep getting rubbish results, instead of the actual result, which is 19.9% He. What am I doing wrong? Should I try another approach? Thank you. Leptictidium (mt) 09:48, 25 October 2009 (UTC)[reply]

The ideal gas law (pV = nRT) is the right approach. You know R and T. Assume a fixed volume (say, 1 litre). The overall mass of the system is then ρ V (and you're given ρ, the density). nHe = mHe / MHe, where mHe is the mass of helium in the system (which you don't know), and MHe is the molar mass of helium (4.0 g/mol) - similarly for the O2. pHe + pO2 is the total pressure of the system, by Dalton's law, and you're given that value (721 mmHg). Substitute the ideal gas law values for pHe and pO2 in the Dalton's law equation, and you should have enough to calculate the ratio mHe/mO2. Tevildo (talk) 10:50, 25 October 2009 (UTC)[reply]
Thank you, but... How am I supposed to calculate pHe without knowing nHe, or vice versa? Leptictidium (mt) 11:29, 25 October 2009 (UTC)[reply]
The Dalton's Law equation is . From the ideal gas law, we get and The two equations you need to solve are: R, MHe, and MO2 are known physical constants. ptotal, T, and ρ are given in the question. If we assume V = 1.0, we can solve the equations for mHe and mO2. (Perhaps start by rearranging the second equation to give mHe in terms of mO2?) Tevildo (talk) 12:07, 25 October 2009 (UTC)[reply]

Coffee Stain

When coffee stains, water colour and other ink stains start out with lot of water and then dry out, why does the dye accumulate at the periphery? I am trying to simulate coffee stain effect. If the dye is supposed to diffuse, then it should spread evenly, right? Is there a wiki article on the physical mechanism behind this ink spreading? —Preceding unsigned comment added by 59.93.1.138 (talk) 10:55, 25 October 2009 (UTC)[reply]

Basically, what happens is that the dye gets carried along with the solvent. The articles I can think of that come the closest to addressing this kind of phenomenon are paper chromatography and thin layer chromatography. Unfortunately, those articles don't go into a whole lot of technical detail about the mechanisms involved. Red Act (talk) 11:40, 25 October 2009 (UTC)[reply]
Umm. Is this obvious or am I missing something? It doesn't seem to me to be quite like paper chromatography because all the coffee is added well mixed (versus solvent washed past the solute). However, even for initially well mixed coffee the solvent will eventually be fairly evenly spread but the concentration in it won't be. All the time the stain is spreading the leading edge is both thinner than the bulk of the fluid behind it (so evaporating proportionately upwards more) and evaporating in the coplanar direction into the carpet (so it has an extra surface). So it is continually getting more concentrated than the bulk of the coffee. Hence a darker ring. That make sense? --BozMo talk 21:04, 25 October 2009 (UTC)[reply]
Whoops. You're right; the situation is rather different from paper chromatography. Please disregard my answer. Red Act (talk) 22:38, 25 October 2009 (UTC)[reply]
The physical basis of the phenomenon is described in Contact line deposits in an evaporating drop, Deegan et al , Phys Rev E, 2000. Looie496 (talk) 21:07, 25 October 2009 (UTC)[reply]

Was gentian violet ever a commonly used antiseptic in the U.S?

Was gentian violet ever a commonly used antiseptic in the U.S.? If it was, when did its use begin to decline? —Preceding unsigned comment added by 98.114.98.236 (talk) 14:39, 25 October 2009 (UTC)[reply]

Well, the first thing that pops to mind was that various characters in the novel Catch-22 were always getting their toes and gums painted with the stuff, and in fact this tidbit is mentioned in our article on gentian violet. [Don't tell the notability police, though; they've got it in for "in popular culture" sections.] Does that article give you any useful information on your question? —Steve Summit (talk) 17:01, 25 October 2009 (UTC)[reply]

Hippo teeth

I'm trying to help a six year old with homework (so this is homework, but not mine!). While I've found plenty of descriptions of hippo teeth (and pictures, yuck!), I've not been able to answer a question of why their teeth are like this! Anyone? --Dweller (talk) 17:06, 25 October 2009 (UTC)[reply]

The general rule with teeth is 1. consider what they eat, and 2. if they have things like tusks, consider how they fight. --Mr.98 (talk) 17:17, 25 October 2009 (UTC)[reply]
Like what, exactly? From a evolutionary perspective, utilizing natural selection, favorable random mutations are passed onto offspring at a greater rate than unfavorable random mutations and non-mutations, thus establishing greater and greater diversity between species. So it must have been that such teeth favored the species leading up to and including the modern-day hippo. As a dentist, I can say with some experience and context that hippo teeth, and for that matter, the teeth, skulls and skeletal patterns of almost all organisms are a real wonder. When taken on a whole, most everything about everything is actually a wonder, if considered in that context. DRosenbach (Talk | Contribs) 17:38, 25 October 2009 (UTC)[reply]
Sorry -- I was assuming you meant your question on a very general level. If, though, by why you meant "how does it come about" or "what processes occur in the periodontium and related structures such that the teeth exist in the manner and environment that they do", you might want to contact the chair of vertebrate zoology at your local natural history museum -- the answer will likely be too complex for a six year old's homework, though...with references made to structures and processes the likes of dental lamina, cementum, odontoblasts and amelogenesis. DRosenbach (Talk | Contribs) 18:41, 25 October 2009 (UTC)[reply]
I think... you have missed the point about the "hippo", here. It is a 6-year-old's question regarding a specific animal, not a general question about "what is evolution, and what does it mean?" --Mr.98 (talk) 20:26, 25 October 2009 (UTC)[reply]
My point was that although the teacher may not understand this (and it might often be that 1st grade teachers lack understanding of this caliber), the father should understand that it's a silly question -- once one asks about a hippo's teeth, why not ask about a manatee's fluke and a walrus's tusks. Then let's ask about the innumerable other specialty items that most animals possess. The kid wasn't asking the question -- it was the father, so I figured the father could use a response to sort of smooth over the intensity of the question's focus on hippos -- the same answer applies for all animals' specialty items. DRosenbach (Talk | Contribs) 01:24, 26 October 2009 (UTC)[reply]
For a 6 year-old's homework - it would be plenty good enough to say that the hippo's teeth evolved the way they did to allow them to eat the food that's available for them in the places where they live. So look up what a hippo eats - and try to understand why their teeth make that easier for them. SteveBaker (talk) 19:23, 25 October 2009 (UTC)[reply]
In addition to the topic of eating, I think Mr. 98 was right to highlight the role that tusks might play in "social interaction". -- Scray (talk) 19:29, 25 October 2009 (UTC)[reply]
Yes, I wasn't trying to be obtuse... I just wanted to structure the inquiry a bit better, and I thought that was clear. Hippo's teeth do two things: they let it chew the leafy material that they eat (those rough, herbivore molars), but more notably (what distinguishes their jaws from, say, cows, in the most obvious way), the front tusks are used for sparring with other hippos (sexual selection), and for self-defense. These are both clear from the hippo page on here, if you are looking for the right things. --Mr.98 (talk) 20:14, 25 October 2009 (UTC)[reply]
(ec)In our article, Hippopotamus, it says that the large canines are used for fighting (either other hippos, or other animals such as crocodiles), and that Hippos mostly eat grass. They need rough molars to chew that. 86.144.144.110 (talk) 20:17, 25 October 2009 (UTC)[reply]
The OP might consider explaining to his child that hippo teeth are as they are - gross and ugly - because hippos don't brush. Perhaps this might encourage a lifetime of better dental hygene in the child. B00P (talk) 05:19, 26 October 2009 (UTC)[reply]

Membrane fluidity. What is it good for?

I was taught in class that the fluidity of lipid membranes is a result of the type of lipids of which the membrane is constructed (saturated vs unsaturated, length) and the presence of cholesterol. I was also told that the fluidity of a membrane is important to a cell, that being to rigid is bad. I was never told however, why exactly this is bad. Wikipedia's article on membrane fluidity doesn't really cut it for me.PvT (talk) 17:26, 25 October 2009 (UTC)[reply]

I'll tell you that after learning this quite a number of times (high school, college, dental school and now again in post-grad), I found only one or two professors who know enough to actually answer such a question. It is merely explained that membrane fluidity is proportional with the cells' ability to be flexible (in the figurative sense, not only the literal sense) in the face of insult. Rigidity, they explain, is detrimental to the cell's ability to adapt to mechanical forces and mechanical changes in ECM of immeditate vicinity. DRosenbach (Talk | Contribs) 19:37, 25 October 2009 (UTC)[reply]
The interesting thing is that I did a thesis a while back on lipid producing algae. Funny thing is they tend to produce more unsaturated fatty acids in cold waters. Presumably this was to maintain the same level membrane fluidity. These algae would most likely not endure high shear forces in nature nor do they need to phagocytize their food source. To me it appears that something more is going on here then just being able to handle mechanical stress.PvT (talk) 20:08, 25 October 2009 (UTC)[reply]
Even if you don't need to handle sheer forces or phagocytize food, you still need to turnover your membrane proteins and lipids. This means that any cell will be continuously exporting and importing material from its membrane, and this will mean extreme local curvature. Cell division will also require a degree of membrane fluidity. Someguy1221 (talk) 22:01, 25 October 2009 (UTC)[reply]
yep. warm blooded animal fats: saturated, solid at room temp but fluid at body temp. fish fats: unsaturated, fluid at the cold temps fish live at. plant oils: unsaturated, fluid at room temps where plants live except...... tropical plants like palm or coconut, whose saturated fats are solid at temps up north here, but fluid at the slightly higher temps where they live. which suggests that cell membranes have to not be too solid, but also not too fluid. i don't think we've made more than a dent in the question of membrane function, but there are issues of permeability (gotta keep the inside separate from the outside or you're not a cell), mobility of membrane structures and proteins etc., which have to sort of float around from where they get made to where they need to be, how proteins, which typically have moving parts, function when embedded in a fluid of whatever consistency, and as some guy points out, just the physics of the bending a membrane goes through. maybe when we get a better handle on this stuff we'll be better able to understand the role of cholesterol in the body than we do now. Gzuckier (talk) 03:03, 26 October 2009 (UTC)[reply]
Thanks for the answers guys :).PvT (talk) 11:36, 26 October 2009 (UTC)[reply]

mental and cognitive development

Hi everyone, I've been given an assignment on child development and the criteria includes mental and cognitive development of a child. My question is, is there a significant difference between a child's mental and cognitive development or are the terms interchangeable? If not, any hints about what I should be including when writing about each of these things? Thanks to anyone who helps! RichYPE (talk) 19:26, 25 October 2009 (UTC)[reply]

Those sound like technical jargon words to me -- likely, they require specific definition on the level at which you are being educated. DRosenbach (Talk | Contribs) 19:40, 25 October 2009 (UTC)[reply]
Mental is a general term for anything to do with the mind. Cognitive refers to reasoning and logic. --Tango (talk) 19:43, 25 October 2009 (UTC)[reply]
In my opinion (as a behavioral neuroscientist), you should treat the two as meaning the same thing. Looie496 (talk) 20:46, 25 October 2009 (UTC)[reply]
Ok, so what do they mean to you? --Tango (talk) 21:19, 25 October 2009 (UTC)[reply]
Both of them basically mean development of the ability to think. This is a very broad and loosely defined question. You could stretch for a distinction between them but it seems most likely to me that the person who wrote that wasn't trying to be so subtle. Looie496 (talk) 21:28, 25 October 2009 (UTC)[reply]
Would you consider developing, for example, the ability to empathise to be part of mental and cognitive development? I would consider it part of mental development but not cognitive. --Tango (talk) 00:38, 26 October 2009 (UTC)[reply]
Looie, I have to disagree with you despite your expertise. Cognition is conscious mental reasoning, and mental includes unconscious mechanisms and heuristics, like recognizing shapes and fine motor coordination. Vranak (talk) 22:12, 25 October 2009 (UTC)[reply]
According to our Mental retardation article, (lack of) cognitive development is one part of mental retardation, as distinguished from "[an] individuals' functional skills in their environment". Mental development redirects to neural development, but I don't think that's what your teacher means. Developmental psychology might be a useful reference. Tevildo (talk) 21:12, 25 October 2009 (UTC)[reply]

Gravitys effect on radioactive decay

Does radioactive elements decay at a different rate in different fields of gravity? The measure given for known elements decay rates are they for G=0 or maybe G does not mean anything in relation to radioactive element decay?

And what happens in layman turns to elements properties in extreme gravity fields (high Gs)? —Preceding unsigned comment added by 85.81.121.107 (talk) 20:05, 25 October 2009 (UTC)[reply]

Any effect will only happen where the force of gravity exceeds that of the weak or strong forces involved in the nucleus. You are looking at G=1040 or something like that. In neutron stars extreme pressure affects the atoms. Graeme Bartlett (talk) 20:57, 25 October 2009 (UTC)[reply]
Radioactive decay will happen slower (from the perspective of a distant observer) in higher gravitational fields due to gravitational time dilation. --Tango (talk) 21:18, 25 October 2009 (UTC)[reply]

Me again..

G=1040 in relation to the above, is that some kind of point where the forces 'melt' together?

So outside of elements (atoms) traveling near a black hole (horizon) I can assume that decay rates are somewhat universal? Is time dilation accounted for... Neutron stars is not really different elements (atoms) so what is the rule that governs decay rates in a gravity field. Gotta read up on neutron stars. —Preceding unsigned comment added by 85.81.121.107 (talk) 22:40, 25 October 2009 (UTC)[reply]

DNA stability vs. RNA

Which contributes more greatly to the chemical stability of DNA when compared to RNA: the fact that it contains thymine (and not uracil, which spontaneously deaminates into cytosine, which would lead to A-U -> G-C mutations), or the fact that it is double-stranded, which, at least to me, seems to protect the bases in a hydrophobic core. Although RNA may form extensive irregular double-stranded secondary structure, it remains single stranded to a significant degree, and so may be more labile because of this. Thanks for the help. —Preceding unsigned comment added by Lashyn (talkcontribs) 21:21, 25 October 2009 (UTC)[reply]

Most of the stability differences between RNA and DNA in vivo arise from the abundance of ribonucleases in the cell, which post-dates the "choice" of DNA over RNA as the genetic material of all living organisms. RNA can also exist in a double stranded state, although this is especially unstable due to dsRNA targetting RNases. As for what stability issues may have lead to the initial choice of DNA over RNA long ago, I'd assume (I'm just guessing here) it may have to do with the ability of RNA to self-hydrolyze its phosphodiester bond through a reaction with its 2' hydroxyl group, which is lacking in DNA. As an example of the inherent instability of RNA outside of all other concers, even high purity RNA, stored at -80oC, may be noticably degraded within years, while similarly purified and stored DNA, even single stranded, will stay good more or less forever. Someguy1221 (talk) 21:53, 25 October 2009 (UTC)[reply]
right. not to repeat what you said, but single stranded DNA is more stable because the 2' hydroxyl participates in the hydrolysis of RNA, so its absence makes DNA more stable in general; presumably that led to evolutionary supremacy of organisms using it, and the biological things like T vs U, double stranded, etc. got added later. Gzuckier (talk) 02:48, 26 October 2009 (UTC)[reply]

Radioactive Mercury

Which Radioactive Mercury were used in old Analog Telephone Sets Grammar-Phone Turn-table Player? —Preceding unsigned comment added by Fred Emagoro (talkcontribs) 21:57, 25 October 2009 (UTC)[reply]

I have a hard time interpreting your Q. Are you perhaps talking about a Gramaphone ? StuRat (talk) 00:49, 26 October 2009 (UTC)[reply]
There is no such thing as radioactive mercury. All of the naturally occurring isotopes are stable. The only way to get radioactive mercury is to make it synthetically - like in a nuclear reactor or something. The cost of the stuff would be spectacular. So I'm pretty sure you're mistaken about the radioactive part. Non-radioactive mercury is used in certain types of switches and relays - it's certainly found in old telephones and lots of other pre-modern electronic systems - dunno about gramophones though. SteveBaker (talk) 02:38, 26 October 2009 (UTC)[reply]
Perhaps poster makes the common mistake of hearing "isotope" and thinking "radioactivity"? DMacks (talk) 03:14, 26 October 2009 (UTC)[reply]

NaBH4 versus LiAlH4

Costs, environmental concerns and economics aside, what would be an advantage of using sodium borohydride over lithium aluminum hydride? Is it mainly the lack of reactivity with possible protecting groups, or is there more? John Riemann Soong (talk) 23:50, 25 October 2009 (UTC)[reply]

NaBH4 is mild enough it can be used in aqueous solution. That's a huuuuge gain for industry, for lab-safety, and for organic-waste issues. The idea of "milder" is a whole world itself, not just "doesn't touch a protecting group". Using a milder reagent, one can get selectivity between two unprotected carbonyls (even easily reduce aldehyde in presence of ketone if you're careful). LiAlH4 is also much more basic as well as more hydride-nucleophilic, so there are base-induced side-reaction issues to consider. DMacks (talk) 03:10, 26 October 2009 (UTC)[reply]
I considered that, but pragmatic considerations generally aren't a factor in grading examinations. No one awards you more points if your reaction is more environmentally-friendly! John Riemann Soong (talk) 04:35, 26 October 2009 (UTC)[reply]
Maybe not in your class.... DMacks (talk) 04:50, 26 October 2009 (UTC)[reply]

October 26

Does acid hydration work with enones?

If so, what caveats should be issued? Now the alkene bond has a polarity -- right? What does that mean for how Markovnikov's rule is applied, etc.? John Riemann Soong (talk) 00:07, 26 October 2009 (UTC)[reply]

Markovnikov's rule is a load of crap, a crutch (albeit a convenient one) to teach one basic pattern of reactivity. It is never the "cause" to be applied, but only an observed way certain reactions go. Consider why they go that way...look for carbocation stability. DMacks (talk) 03:12, 26 October 2009 (UTC)[reply]
Well yeah I know about carbocation stability. (I use Markovnikovity as an indicator of "default thinking".) Basically, the alkene bond is now polarised, isn't it? Enones are Michael acceptors? So H+ might want to add to the nucleophilic carbon? (Which also happens to be an alpha carbon?) Is it enough to drive the reaction to be anti-Markovnikov, if the reaction occurs at all? John Riemann Soong (talk) 04:31, 26 October 2009 (UTC)[reply]
"α to carbonyl (Cδ+)" is usually more electronically significant than most 1°/2°/3° alkyl carbocation effects. Googling for "hydration of acrolein" says that 3-hydroxy is the product. DMacks (talk) 04:37, 26 October 2009 (UTC)[reply]
Also, I'd like to know about hydrogenation of conjugated pi systems. Will LiAlH4 react with conjugated systems, but not sodium borohydride? John Riemann Soong (talk) 04:36, 26 October 2009 (UTC)[reply]

short snooze hunched over, always wake up with tremendous amounts of gas in my stomach

Preface - if directly answering the following breaks the No Medical Advice rule, I'd appreciate someone instead directing me to the most appropriate anatomy page on Wikipedia.

For as long as I can remember, if I fall asleep hunched over - say at a desk or low table - when I wake up I have a tremendous amount of gas in my stomach. I'm talking like an almost immediate 4 or 5 second sustained belch. I've often wondered at the physical processes at work here. Given my posture, I suspect there's some sort of relaxing of the throat? Or contortion of a sort that alters the position of that flappy thing (the name escapes me at the moment) which controls trachea vs esophagus entry? But even if the entry to the esophagus is partly ajar while I slowly respire, what draws such a large amount of gas into my stomach? 218.25.32.210 (talk) 01:32, 26 October 2009 (UTC)[reply]

The flappy thing is your epiglottis. And I'm like you in that if I fall asleep with my head on my desk, I have a long "exhalation" of gas from my stomach too. Dismas|(talk) 03:04, 26 October 2009 (UTC)[reply]

Girls born first

How come it is usually girls born first, then boys born second?--Mikespedia (talk) 06:16, 26 October 2009 (UTC)[reply]

It isn't. The sex ratios of first borns is not particularly different from that of subsequent children. Some studies have found a very slight female bias, but its marginal and still near 50:50. Rockpocket 07:02, 26 October 2009 (UTC)[reply]
It may be so in certain families because of random chance - just like the family I heard about on Paul harvey's news back in the 1990s or so where one brance hadn't had a girl born in a century or more. But, if that's what you're seeintg, it would be observational bias; a normal condition where you expect to see things one way becuase that's how they may be in each situation you've experenced.209.244.187.155 (talk) 12:13, 26 October 2009 (UTC)[reply]

plant viruses.

what is the mechanisum of viruses to infect a plant? —Preceding unsigned comment added by 121.52.144.162 (talk) 06:40, 26 October 2009 (UTC)[reply]

We have an article, amazingly enough, called Plant virus. That should answer your question at a superficial level, if you have more detailed questions, do come back. Rockpocket 06:51, 26 October 2009 (UTC)[reply]

Jones oxidation to COOH

I read that Jones oxidation will oxidise aldehydes to COOH by relying on a geminal diol intermediate (hydration of the aldehyde). The problem I have with this is that some aldehydes might not be very hydratable, even at catalytic level, and also, isn't the alcohol more acidic than an C-H proton? (I'm looking at http://www.organic-chemistry.org/namedreactions/jones-oxidation.shtm). Basically, wouldn't the chromate possibly complex to both alcohol groups, thereby becoming "stuck" most of the time? John Riemann Soong (talk) 07:06, 26 October 2009 (UTC)[reply]

Some aldehydes are indeed not very hydratable, which makes them not easily oxidized under these conditions. Maybe that's why your ref notes "The oxidation of primary allylic and benzylic alcohols gives aldehydes."? The alcohol is much more acidic than a C-H, but that is already covered in the mechanism: the OH does react first, and only later does the C-H get removed. And your ref even illustrates it as a cyclic mechanism, so it is not literally "C-H acting as an H+ donor". But if it were (and some refs list an acyclic mechanism in which an external Lewis base removes that H+), Cr(VI) is very electrophilic, which makes it an excellent leaving group in a simple E2 mechanistic sense. Having a Cr atom complex both O at the same time is unlikely: 4-membered ring. But even if it did happen, it would still not prohibit an elimination mechanism leading to a carbonyl with loss of Cr off one of the O (would leave it on the other O, as some sort of a carboxylate/chromite mixed anhydride). DMacks (talk) 07:31, 26 October 2009 (UTC)[reply]

In theory

What is the largest ball of absolutely pure iron that can be formed and at that point or beyond what would happen if more iron were added? 71.100.9.185 (talk) 08:29, 26 October 2009 (UTC)[reply]

I believe that at a critical mass the ball would collapse into a neutron star. It seems that before that however, it would tend towards pure Iron-56. Jkasd 08:56, 26 October 2009 (UTC)[reply]
In that case would it begin to spew protons and electrons into space or form more neutrons from them? 71.100.9.185 (talk) 09:01, 26 October 2009 (UTC)[reply]
According to Degenerate matter#Neutron degeneracy, the protons and electrons form into neutrons through electron capture, although some "normal" nuclei may be present near the surface of a neutron star, including iron nuclei according to Neutron star#Structure. Jkasd 09:17, 26 October 2009 (UTC)[reply]
So the conversion would follow a normal decay path from an atom of iron back through the lighter elements overcoming the stable isotopes back finally to a bunch of neutrons? 71.100.9.185 (talk) 10:06, 26 October 2009 (UTC)[reply]

Titan or Mars

Is it possible to place Titan or Mars into a solar orbit within the Habitable zone 180 degrees from the position of Earth? 71.100.9.185 (talk) 08:57, 26 October 2009 (UTC)[reply]

It seems that any one of the five Lagrange points would do nicely, if you could figure out how to move such a large object. Perhaps with a gravitational tractor? Also see Counter-Earth. Jkasd 09:25, 26 October 2009 (UTC)[reply]
I was thinking perhaps more in line with the idea of a lot of rocket motors on one side of the planet. 09:45, 26 October 2009 (UTC) —Preceding unsigned comment added by 71.100.9.185 (talk)
Well both ways would take a long time to complete and enormous amounts of energy. Of course, there's also methods that push the boundaries of physics such as worm holes or something from the Breakthrough_Propulsion_Physics_Program. Jkasd 09:54, 26 October 2009 (UTC)[reply]
Also keep in mind that both Mars and Titan are rotating, so the "one side" of the planet the rockets are on is always changing. Although this website is intended to be humorous and not technically accurate, it might lead insight into the enormous difficulty of moving a planet (or moon). Jkasd 10:08, 26 October 2009 (UTC)[reply]

Strong interaction and distance

The article Strong interaction states "The strong force acting between quarks, unlike other forces, does not diminish in strength with increasing distance, after a limit (about the size of a hadron) has been reached. It remains at a strength of about 100 000 newtons, no matter how far away from each other the particles are, after this limiting distance has been reached." This threw me for a loop. There is a force that does not diminish with distance?!? Could someone explain why the strong force doesn't diminish in strength beyond that distance? I can understand complicated answers but don't have a background in quantum physics. Wikilinks to related concepts in your explanation would be greatly appreciated. Thank you. 152.16.16.75 (talk) 09:46, 26 October 2009 (UTC)[reply]

The color confinement article contains a brief (and no doubt hugely over-simplified) explanation. In short, the particles that carry the strong force (called gluons) also have a colour charge themselves. So if two quarks are separated, the gluons that are exchanged between them to create the strong force interact amongst themselves, creating more and more gluons, which tend to be attracted to one another. This creates a force field which is concentrated into narrow tubes, and so the force does not diminish with distance. By contrast, photons, which carry the electromagnetic force, do not have an electric charge, so (a) the photons exchanged between two particles with electric charge do not spontaneously create more photons and (b) these photons are evenly distributed in all directions, so the electromagnetic force between the particles decreases with the square of distance. Gandalf61 (talk) 11:23, 26 October 2009 (UTC)[reply]

over population control

Could doctors without our permission or knowledge splice GMO's capable of rendering us sterile until we paid the government a large fee for a deactivator spray to have children or as a means to create a "master" race? 71.100.9.185 (talk) 10:11, 26 October 2009 (UTC)[reply]

No. Dbfirs 10:27, 26 October 2009 (UTC)[reply]
Why not knowing how Monsanto handled farmers over GMO flax, soybeans, corn and peanuts? 71.100.9.185 (talk) 10:42, 26 October 2009 (UTC)[reply]
One of the skills embodied in scientific skepticism is assessing ideas like this one, to find a reasoned position as to whether the idea is more likely to be sound or unsound. One example of an unsound idea is the conspiracy theory. Conspiracy theories sometimes can be shown to be unsound because they rely on large, disparate groups of people all acting in concert to hide vital information and promote unethical behaviour. Individuals, and even small groups, often act unethically but the larger the group the less likely it is that every member of that group will conform to the group agenda. The larger the group the more likely it is that some of them will rebel, break ranks, talk to the media etc.
In the question above it is proposed that a large group of professionals ("doctors") would know about the use of GMOs capable of rendering a population sterile for monetary purposes, but that the population would not know about it because not one member of that large group would spill the beans, talk to the media, whisper to friends and family members, challenge the group's leaders etc. Sounds to me like a classic conspiracy theory.
I recommend that everyone develops at least a little skill in scientific skepticism. Dolphin51 (talk) 11:34, 26 October 2009 (UTC)[reply]
All countries have secrets, which they keep secret by common bond, indoctrination and having a realistic and agreed upon goal. The potential benefit offered by GMO as opposed to possible opposition based on tampering with God or His creation beyond what God approved (see Genesis 1:28-30). Monsanto may not be put out of business by the NON-GMO project just yet but I ask why does the NON-GMO even exist if what you are saying is universally applicable? Their are evil people and groups in the would capable of hiding behind medicine or food which led me to ask the question. I think your answer a bit naive. 71.100.9.185 (talk) 12:22, 26 October 2009 (UTC)[reply]
Could they ? Perhaps, although probably not with current technology and knowledge (and, in any case, the public water supply would be a much more effective means of distribution). Would they ? No - not because of any ethical scruples, but because it would be economic and political suicide. Monsanto, like any commercial company, have economic incentives to exploit their market as much as they can get away with. A country that introduced non-voluntary reversible sterilisation of all of its citizens would see its economy collapse immediately. Gandalf61 (talk) 11:43, 26 October 2009 (UTC)[reply]
Ever hear about the fog that jumped out of the boiling pot of water while his companion fell asleep in the ever increasing temp of the water in the pot he was in? 71.100.9.185 (talk) 12:22, 26 October 2009 (UTC)[reply]
Yes, we have an article on it, actually. But hey, if you're convinced it's likely, don't let us dissuade you. But you're probably not going to convince us here. It sounds very science fiction and far-fetched for a number of reasons. It presupposes much closer cooperation between the government and large biotech companies than exists, where both would mutually be willing to let the other one take on activities of such risk that they could kill millions and lead to political revolution. This is rather improbable. --98.217.71.237 (talk) 12:33, 26 October 2009 (UTC)[reply]

ascetic acid

Sorry for so many questions but when they come it doesn't just rain it pours.

My question is since a wart soaked in 5% ascetic acid solution for about two weeks will be killed has this ever been used to treat malignant cancers caused by viruses that produce malignant cancers? 71.100.9.185 (talk) 10:31, 26 October 2009 (UTC)[reply]

If I am not mistaken, which I may well be, viruses cause cancer by altering the DNA of a cell and making it malignant. Once this is done, the virus is no longer important to the cancer, and killing any such virus probably won't kill the cancer. Falconusp t c 11:23, 26 October 2009 (UTC)[reply]
I'm assuming that after the virus is dead that when the body recognizes there is a problem the immune system will kill the altered cells without worry of the cycle repeating. This seems to be the wart killing mechanism. Kill the virus (by penetrating cells) so the immune system can kill the altered cells. 71.100.9.185 (talk) 11:31, 26 October 2009 (UTC)[reply]